Vous êtes sur la page 1sur 323

01-Financial Accounting An Introduction Structure: 1.1 Introduction Objectives 1.2 Meaning of Accountancy, book-keeping and Accounting 1.

.3 Accounting Process 1.4 Objectives for accounting 1.5 Differences between book-keeping and accounting 1.6 Users of accounting information 1.7 Limitations of Accounting 1.8 Basic terminologies 1.9 Summary 1.10 Terminal Questions 1.11 Answers 1.1 Introduction All of you at one point of time would have visited a grocery shop or a medical shop. You might have wondered how the business person maintains the record of all the transactions done during a particular period of time say a year. You might have also thought why he or she has to maintain a record, how is it beneficial and whether it is mandatory or not? As against this, imagine the role of a business organization. They provide goods that might range from simple safety pin to fighter aircrafts. Those who are in service industry provide various services such as transportation services, hospitality services, developing complex software programmes etc. To make sound decision a business enterprise need accounting information. This information is also needed by government agencies, regulatory bodies, analyst and individuals at various point of time and at different levels. Accounting is perhaps one of the oldest, structured management information system. It has evolved in response to the social and economic needs of society. Accounting as an information system is concerned with identification, measurement and communication of economic information of an organization to its users who may need the information for rational decision

making. The accounting system is a means to provide relevant and reliable financial information to all the interested parties. In this unit we are dealt the meaning of accounting, book-keeping and accountancy, the steps involved in accounting process. It explains the various objectives of accounting, discusses the difference between book-keeping and accounting and how accounting information is used by various users of accounting information. This unit concludes with basic terminologies in accountancy. Objectives: After going through this unit, you should be able to: 1. Define book keeping, accounting and accountancy 2. Describe the accounting process 3. Explain the objectives of accounting 4. Distinguish between book keeping and accounting 5. Categorize various users of accounting information 6. Acquaint with the basic terminology used in the subject. 1.2 Meaning of Book-keeping, Accounting and Accountancy Accounting as a discipline was introduced to have permanent and systematic record of business transactions. This would help a business person to record all relevant business transactions, to ascertain the profit earned during a particular period and finally evaluate the financial position of his/her business. Book keeping, accounting and accountancy are the terms used in the science of financial accounting. Book-keeping is defined as the science and art of recording business transactions in a systematic manner in a certain set of books known as books of accounts. It identifies the transactions and events, measures the identified transactions and events in a common measuring unit, records them in proper books of accounts and finally classifies them in another book called the ledger. Accounting is termed as language of business which records all events and transactions that are of monetary value and facilitates communication among individuals in a society. Accountancy refers to a systematic knowledge of accounting. It explains why to do and how to do of various aspects of accounting. It tells us why and how to prepare the books of accounts and how to summarize the accounting information and communicate it to the interested parties. 1.3 Accounting Process

Accounting is the process of identifying the transactions and events, measuring the transactions and events in terms of money, recording them in a systematic manner in the books of accounts, classifying or grouping them and finally summarizing the transactions in a manner useful to the users of accounting information.

1. Identifying the transactions and events: This is the first step of accounting process. It identifies the transaction of financial character that is required to be recorded in the books of accounts. Transaction is transfer of money or goods or services from one person or account to another person or account. Events happen as a result of internal policies or external needs. Events of non financial character cannot be recorded even though such events may have an impact on the operational results of the firm. 2. Measuring: This denotes expressing the value of business transactions and events in terms of money (in terms of rupees in India). 3. Recording: It deals with recording of identifiable and measurable transactions and events in a systematic manner in the books of original entry that are in accordance with the principles of accountancy. 4. Classifying: It deals with periodic grouping of transactions of similar nature that appear in the books of original entry into appropriate heads by posting or transfer entries. For Eg: All purchases of goods made for cash or on credit on different dates are brought to purchase account. 5. Summarizing: It deals with summarizing or condensing transactions in a manner useful to the users. This function involves the preparation of financial statements such as income statement, balance sheet, statement of changes in financial position and cash flow statement. 6. Analyzing: It deals with the establishment of relationship between the various items or group of items taken from income statement or balance sheet or both. Its purpose is to identify the financial strengths and weaknesses of the enterprise. The above six process in the present day scenario are generally performed using software packages. 7. Interpreting: It deals with explaining the significance of those data in a manner that the end users of the financial statement can make a meaningful judgment about the profitability and financial position of the business. The accountants should interpret the statement in a manner useful to the users, so as to enable the user to make reasoned decision out of the alternative

course of action. They should explain various factors on what has happened, why it happened, and what is likely to happen under specific conditions. 8. Communicating: It deals with communicating the analyzed and interpreted data in the form of financial reports/ statements to the users of financial information eg Profit and loss account, Balance Sheet, Cash flow and Funds Flow statement, Auditors report etc. The Accounting Information System

Source: Adapted from Financial Accounting, Management perspective by R. Narayaswamy. Self Assessment Questions 1. Book keeping _________ the transactions and events, _________the identified transactions and events in a common measuring unit, records them in proper books of accounts and finally classifies them in the ledger. 2. Accounting in addition to book keeping involves ________ the classified transactions and ________ the summarized results. 3. ____________ interprets the analyzed results and communicates the interpreted information to the interested parties.

1.4 Objectives of Accounting From the above paragraphs, it can be concluded that accounting involves the following functions and objectives.

a) Accounting helps in systematic recording of all business events or transactions. Written records are more preferable to memorizing (oral recording) because the latter may fade away with time. Also systematic records can be used by different persons for different decision making purposes. b) Accounting measure the financial performance of the enterprise. The results of operations are ascertained by preparing profit and loss account, balance sheet and cash flow statements. This will enable the business person to ascertain what the business owes to others, what others owe them and whether his/her capital remained same or increased or decreased. c) Accounting facilitates in reporting the results to both internal and external users. The management requires information for internal purpose at various levels of operations. They need to prepare various reports such as production report, idle time report, cash budget report, receivable report, accounts payable report, project appraisal report, capital budgeting report etc. The management reports the financial performance of the firm to external users such as shareholders, creditors, bankers, investors, stock brokers, stock exchanges, employees, governments etc., d) Accounting is required to fulfill the statutory requirements of various regulatory bodies such as Registrar of Companies, SEBI (Securities Exchange Board of India) income tax authorities and the Government. e) Accounting helps in internal control by holding the concerned persons responsible for any errors, lapses or under performance. Equally it helps to identify the strong /weak areas of each unit or department. Accounting is a tool for effective planning. Current years financial performance becomes the basis for future predictions and estimations. Since it is tool for planning, it also acts as tool for controlling. Preparation of budgets, cost analysis, tax planning, auditing are some of the functions of accounting. Self Assessment Questions: 4. Accounting is a tool for _______________ and ________ 5. Expand SEBI. 6. Mention any five stakeholders. 1.5 Distinction between Book-keeping and Accounting Accountancy is the profession and the practitioners of accountancy are called accountants. Book keeping is the basic activity of recording. On recording the transactions and events in the books of accounts, accounting does the role of analysis and reporting. Accountancy is the profession of carrying the activities of book keeping and accounting. Accounting enjoys wider scope and includes not only book keeping but also analysis, interpretation and reporting of financial

information. The later part of accounting is the core function of accounting. In the present day environment, sophisticated software packages are available, which facilitate entry of transactions and preparation of ledger accounts. Book keeping Accounting It is a process of identifying, It involves summarizing the classified measuring, recording and transaction, interpreting the analyzed analyzing the transactions in books results and communicating the of accounts information to the users of financial statement. Adopt principles of accounting Analyzing and interpreting requires for recording skill, knowledge and experience Book keeping is the first stage of Accounting follows book keeping. It is accounting process the secondary stage The objective is to prepare final The objective is to ascertain net results accounts and balance sheet in a of financial operations and communicate systematic manner at the end of the results to all stakeholders in a accounting period manner they understand. Accounts executives who perform Accountants who perform this function this function may not require need higher analytical skills to interpret higher level of knowledge. the data and to take appropriate decisions. The nature of job is routine and The nature of job is non routine but clerical analytical. 1.9 Users of Accounting Information Accounting reports are designed to meet the common information needs of most decision makers. These decisions include when to buy, hold or sell the enterprise shares. It assesses the ability of the enterprise to pay its employees, determine distributable profits and regulate the activities of the enterprise. Investors and lenders are the most obvious users of accounting information. a) Investors: Investors may be broadly classified as retail investors, high net worth individuals, Institutional investors both domestic and foreign. As chief provider of risk capital, investors are keen to know both the return from their investments and the associated risk. Potential investors need information to judge prospects for their investments. b) Lenders: Banks, Financial Institutions and debenture holders are the main lenders and they need information about the financial stability of the borrower enterprise. They are interested in information that would enable them to determine whether their borrower has the capability to repay the loans along with the interest due on it. They also use the information for monitoring the financial condition of the borrowers. They may stipulate certain restrictions (known as covenants) such as upper limit on the total debt borrowed from all sources or ask for additional security etc. Short term lenders (trade creditors) who provide short term financial support need

information to determine whether the amount owing to them will be paid when due and whether they should extend, maintain or restrict the flow of credit. c) Regulators, Rating Agencies and Security Analyst: Investors and creditors seek the assistance of information specialist in assessing prospective returns. Equity analyst, bond analyst and credit rating agencies offer a wide range of information in the form of answering queries on television shows, providing trends in business newspapers on a particular stock, offer valuable information in seminars, discussion groups, meetings and interviews. Security analyst obtain valuable information including insider information by means of face-to-face meetings with the company officials, visit their premises and make constant enquiry using e-mails, teleconference and video conference. Firms build a good rapport with such type of information seekers to gain visibility in the market. d) Management: Management needs information to review the firms short term solvency and long term solvency. It has to ensure effective utilization of its resources, profitability in terms of turnover and investment. It has to decide upon the course of action to be taken in future. Management may also be interested in acquiring other business which is undervalued. When managers receive a commission or bonus related to profit or other accounting measures, they have a natural interest in understanding how those numbers are computed. Further when faced with a hostile takeover attempt, they communicate additional financial information with a view to boosting the firms stock price. e) Employees, Trade Union and Tax authorities: Employees are keen to know about the general health of the organization in terms of stability and profitability. Current employees have a natural interest in the financial condition of the firm as their compensation will depend on the financial performance of the firm. Potential employees may use financial information to find out the future prospects of the firm. Trade unions use financial reports for negotiating wage package, declaration of bonus and other benefits. Tax authorities need information to assess the tax liability of the firm. f) Customers: Customers have an interest in the accounting information about the continuation of company especially when they have established a long term involvement with or are dependent on the company. For Eg. Car owners, buyers of white goods, electronic gadgets, depend on the manufacturer for warranty service support, continued supply of spare parts. The sales of Matiz car was badly affected due to the abrupt closure of Daewoo Motors. g) Government and regulatory agencies: Government and the regulatory agencies require information to obtain timely and correct information, to regulate the activities of the enterprise if any. They seek information when tax laws need to be amended, to provide institutional support to the lagging industries. The regulatory agencies use financial reports to take action against the firm when appropriate returns are not filed in time or when the returns fails to provide true and fair position of the business or to take appropriate action against the firm when complaints / misappropriation are being lodged. Stock exchange has a legitimate interest in financial reports of publicly held enterprise to ensure efficient operation of capital market.

h) The Public: Every firm has a social responsibility. Firms depend on local economy to meet their varied needs. They may get patronage from local government in the form of capital subsidy, cheap land or tax sops in the form of tax holidays for certain period of time. Prosperity of the enterprise may lead to prosperity of the economy both directly and indirectly. Growth in software industry in Bangalore, Karnataka State, led to boom in housing sector, education sector, entertainment sector, travel sector and tourism sector in and around Bangalore. Published financial statement assist public by providing information about the trends and recent developments of the firm.

Self Assessment Questions 7. ________ as chief provider of risk capital is keen to understand both the return from their investments and the associated risk. 8. _________ use financial reports for negotiating wage package, declaration of bonus and other benefits. 9. ___________ has a legitimate interest in financial reports of publicly held enterprise to ensure efficient operation of capital market. 10. The regulatory agencies use _____________ to take action against the firm when appropriate returns are not filed in time or when the returns fails to provide true and fair position of the business or to take appropriate action against the firm when complaints / misappropriation are being lodged. 1.7 Limitations of Accounting 1) Though accounting system is the only source for extracting financial information of the firm, it grossly lacks qualitative elements. Qualitative resources could include leadership of top brass, highly talented human resource, highly motivated team, best products, the power of resource and development, brand image etc. 2) Accounting is not free from bias. The accountants have some leeway or freedom on the methods of depreciation charged, inventory valuation etc. Though the convention says consistency has to be maintained on the policies adopted, there is considerable room for bias, favourism and personal judgment.

3) Accounting reveals the estimated position and not the real position of the firm. Generally financial statements are prepared on separate entity concept, conservatism concept etc. which are based on the estimates that may lead to over valuation or under valuation of assets and liabilities. The exact picture of the financial situation can be ascertained only on the liquidation of an enterprise. 4) Accounting ignores the price level changes when financial statements are prepared on historical cost. Fixed assets are shown in the balance sheet at historical cost less accumulated depreciation and not at their replacement value. Land value is shown at historical cost but the replacement value could be far higher than the value stated in the balance sheet due to appreciation of land value over the period of time. 5) The danger of window dressing arises when the management decides to incorporate wrong figures to artificially inflate revenue or deflate losses or when there is a threat of hostile takeover. In such a situation the management fails to provide true and fair view of the financial position to the various users of the financial statement. Satyam Computer Services, the fourth largest software firm went into bust when the information on inflated income to the extent of Rs.7000 crore was revealed. Self Assessment Questions 11. Accounting grossly lacks ____________elements 12. The exact picture of the financial situation can be ascertained only on the ________of an enterprise. 13. The danger of ___________ arises when the management decides to incorporate wrong figures to artificially inflate revenue or deflate losses or when there is a threat of hostile takeover. 14. Accounting ignores the price level changes when financial statements are prepared on __________. 1.8 Basic Terminologies To understand the subject, proper understanding of the following terms is essential. 1. Transaction: It is transfer of money or goods or service from one person or account to another person or account. There are cash transactions, credit transactions and paper transactions. In all cash transactions, cash is paid or received immediately. Credit transaction is one where there is a promise to pay/receive cash at a future date. Paper transaction is one where there is no cash inflow or outflow but adjustment is made in the records only. (Bad debts of previous year are written off; depreciation provided on fixed assets etc.), 2. Capital: Funds brought in to start business, by the owner/s. In the case of a company, capital is collected by issue of shares. Capital used to purchase fixed assets is called fixed capital and

that capital used for day to day affairs of business is known as working capital. From business point of view, Capital is a liability. 3. Share: A share in a company is one of the units into which the total capital of the company is divided. 4. Assets: An asset is a resources legally owned by the enterprise as a result of past events and from which future economic benefits are expected to flow to the enterprise. Eg: Land and buildings, plant and machinery, furniture and fixtures, cash in hand and at bank, debtors and stock etc., are regarded as assets, Assets may be fixed, current, liquid or fictitious. 5. Fixed assets are those which are held for use in the production or supply of goods and services. Ex: plant and machinery, which is used fairly for long period. 6. Current assets are those which are held or receivable within a year or within the operating cycle of the business. They are intended to be converted into cash within a short period of time. Ex: Stock in trade, debtors, bills receivable, cash at bank etc., 7. Liquid assets are those which can be easily converted into cash and for instance, cash in hand, cash at bank, marketable investments etc., 8. Fictitious assets are in the form of such expenses which could not be written off during the period of their incidence. For example, promotional expenses of a company which could not be treated as expenditure in the year of incidence are shown as fictitious asset. 9. Liability: It is a financial obligation of an enterprise arising from past event the settlement of which is expected to result in an outflow of resources embodying economic benefit. Eg. Loans payable, salaries payable, term loans. 10. Current liability is that obligation which has to be satisfied within a year. For example, payment to be made sundry creditors for the goods supplied by them on credit; bills payable accepted by the businessman; overdraft raised by the businessman in a bank etc. 11. Equity: Equity is the residual interest in the asset of the enterprise after deducting all its liabilities. The equity of a company is called shareholders equity. Its components include share capital, share premium and retained earnings. 12. Entity: It is an economic unit that performs economic activities. 13. Sole trader: A single individual carrying on business with or without the help of his kith and kin is called sole trader. 14. Partnership: It is a relationship between partners to contribute capital to start business, agree to distribute profits and losses in an agreed proportion and the business being carried on by all or any one acting for all. Partnership firm refers to business where as the partnership refers to relationship caused by agreement.

15. Joint Stock Company: It is an organization, for which the capital is contributed by shareholders to carry on business and it is registered under Companies Act and it has a legal entity, having perpetual existence and a common seal. 16. Goods: Goods refer to merchandise, commodities, products, articles or things in which a trader deals. It is the commodities or things meant for resale. Goods account is divided into six heads viz: purchase account, sales account, purchase return account, sales return account, opening stock account and closing stock account. Let us get the meaning of each one. Purchase: Goods purchased by a business are called purchase. Sales: Goods sold by a business are called sales. Purchase Return or Return Outward: Goods returned by the business to its suppliers out of the purchases already made from them are called purchase return. Sales Return or Return Outward: Goods returned to a business by its customers out of the sales already made to them are called Sales Return. Opening Stock: Unsold goods lying in a business at the beginning of a year, are called opening stock. Closing Stock: Unsold goods lying in a business at the end of a year, are called closing stock. 17. Inventory: Inventory refers to goods held by a business for sale in the ordinary course of business or for consumption in the production of goods or service for sale. It includes stock of raw materials, stock of work in progress and stock of finished goods. 18. Drawings: It refers to cash, goods or any other asset withdrawn by the proprietor from his business for his personal or domestic use. In short, amounts the owner withdraws from his business for living and personal expenses. 19. Debtor: A debtor is a person who owes money to the business. A debtor may be of 4 types. Trade debtor is a person who owes money to the business for the goods supplied to him on credit. A loan debtor is a person who owes money to the business for the loan advanced to him. Debtor for asset sold is a debtor who owes money to the business for any asset sold to him on credit. A debtor for service rendered is a debtor who owes money to the business for the service rendered to him on credit.

20. Debt: the amount due from a debtor to the business is called a Debt, generally debt may be of three types: Good debt refers to fully recoverable debt. Bad debt refers to debt, which is not recoverable (irrecoverable). Doubtful debt refers to debt whose recovery is doubtful. 21. Creditors: A creditor is a person to whom the business owes money. A creditor also may be of 4 types. Trade creditor is a person to whom the business owes money for goods purchased from him on credit. Loan creditor is a person to whom the business owes money for the loan borrowed from him. Creditor for asset purchased is a creditor to whom the business owes money for any asset purchased from him on credit. Expenses creditor refers to a creditor to whom the business owes money for any service received from him on credit. For e.g.: salaries unpaid, commission unpaid etc. 22. Loss: It refers to money or moneys worth given up without any benefit in return. For e.g. loss of cash by theft, loss of goods by fire etc. it is a situation where in the expenses of the business exceeds revenues. An expense brings some benefits, but loss does not bring any benefit. 23. Profit: It is a situation where the revenue of a business exceeds its expenses. In other words, the amounts we earned were greater than our expenses. 24. Journal: A journal is a daily record of business transactions. It is a book of original, prime or first entry in which all the business transactions are first entered in the specified manner in the order of dates. A preliminary record where business transaction is first entered into the accounting system. 25. Ledger: A ledger is an account book in which all the accounts are maintained. It is the books of final entry as well as principal book of accounts. 26. Entry: It is the record of a transaction made in any book of account, either in the book of original entry or in the books of final entry. 27. Narration: It is a brief explanation to a journal entry, given below the journal entry, with in brackets. It gives the explanation for the particular journal entry. 28. Posting: Posting is the process of entering in the ledger the information already recorded in the journal or in any of the subsidiary books. In other words process of transferring balances

from bookkeeping records called journals to a "final" bookkeeping record called the general ledger. 29. Voucher: It refers to any written document in support of a financial transaction. 30. Trial Balance: A worksheet listing of all the accounts appearing in the general ledger with the dollar amount of the debit or credit balance of each, used to make sure the books are "in balance" total debits and credits are equal. 31. Balance Sheet: It is the financial statement, which shows the amount and nature of business assets, liabilities, and owners equity as of a specific point in time. It is also known as a Statement of Financial Position or a Statement of Financial Condition. 32. Carried Forward: The term carried forward or its abbreviation [c/f] is used at the foot of a page to indicate that the total amount at the foot of that page has been carried forward to the head of the next page. 33. Brought Forward: The term brought forward or its abbreviation [b/f] is used at the head of page to indicate that the total amount at the head of that page has been brought forward from the foot of the previous page. 34. Carried Down: The term carried down or its abbreviation [c/d] is written in a ledger account at the time of its closing to indicate that the balance in that account has been carried down to the next period. 35. Brought Down: The term brought down or its abbreviation b/d is written in a ledger account at the time of its opening to indicate that the opening balance in that account has been brought down from the previous period. 36. Bill of exchange: It is documentary evidence in writing containing an unconditional order signed by the maker, directing a certain person to pay a certain sum of money only to, or to the order of, a certain person or the bearer of the instrument. 37. Bills Payable: It is a bill of exchange stating an obligation to pay a certain sum of money at a specified date. In case of purchase of raw materials on credit the supplier or the creditor draws bills of exchange on the business entity. When the entity accepts the bill it becomes bills payable for the entity. The same bill for the supplier is termed as bills receivable. 38. Bills Receivable: It is a bill of exchange containing an acceptance from the drawee (or Payee) a certain sum of money at a specified date. On sale of goods on credit the entity draws a bill of exchange on the customer. When the customer or debtor accepts the bill it becomes bills receivable for the firm. Bills receivables can be discounted with banks or discount houses. 1.9 Summary

Accounting is the process of identifying the transactions and events, measuring the transactions and events in terms of money, recording them in a systematic manner in the books of accounts, classifying or grouping them and finally summarizing the transactions in a manner useful to the users of accounting information. The main objective of accounting is to determine income, financial reporting and disclosure of relevant and pertinent information to the users of financial information. The users of accounting information are investors, lenders, regulators, rating agencies, security analysts, management, employees, trade unions, tax authorities, customers, government and the general public. Accounting ignores qualitative aspects while providing information. It is not free from bias. It ignores price level changes and pose the danger of window dressing. Management accounting refers to the use of financial data for the purpose of planning and decision making, performance evaluation etc. 1.10 Terminal Questions 1. Explain the process involved in accounting. 2. What are the objectives of accounting? 3. Distinction between book-keeping and accountancy. 4. How accounting information is used by investors and lenders? 5. How Government and Regulatory agencies use accounting information to regulate the activities of the firm? 6. Distinguish between financial accounting and management accounting 1.11 Answers to SAQs and TQs

1. Identifies, measures 2. Summarizing, analyzing 3. Accounting 4. Effective planning, controlling 5. Securities Exchange Board of India

11. Qualitative 12. Liquidation 13. Window dressing 14. Historical Cost

6. Shareholders, Creditors, Bankers, Government, Employees 7. Investors 8. Trade Union 9. Stock Exchange 10. Financial Reports Answers to Terminal Questions 1. Refer 1.3 2. Refer 1.4 3. Refer 1.5 4. Refer 1.6 5. Refer 1.6 6. Refer 1.8

MB0041-Unit-02-Accounting Concepts, Principles, Bases and Policies


Unit-02-Accounting Concepts, Principles, Bases and Policies Structure: 2.1 Introduction Objectives 2.2 Accounting Concepts, Principles, Policies and Standards 2.3 Types of accounting concepts Business Separate entity concept Going concern concept Money measurement concept Periodicity concept Accrual concept 2.4 Accounting Principles Principle of Income recognition Principle of expense Principle of matching cost and revenue Principle of Historical costs Principle of full disclosure Double aspect principle Modifying Principle Principle of materiality Principle of consistency

Principle of conservatism or prudence 2.5 Accounting Policies Changes in Accounting Policies Disclosure in case of changes in Accounting Policies 2.6 Accounting Standards Scope and functions of Accounting Standards Board International Financial Reporting System 2.7 Summary 2.8 Terminal Questions 2.9 Answers Annexures I and II 2.1 Introduction In the previous unit we had discussed on the meaning, objectives of accounting, the accounting process involved and the distinction between book keeping and accounting. Accounting is a reflection of all business transactions expressed in terms of money pertaining to a definite period of time. The objective of accounting is to find out profit or loss arising out of transactions and finally to judge the financial position of the business organization. Accounting is based on certain postulates, concepts and policies. In this Unit, we have dealt with various types of concepts, principles and policies of accounting with suitable examples. A brief introduction is made on accounting standards issued by Accounting Standards Board of ICAI and International Financial Reporting System (IFRS). Objectives: After going through this unit, you should be able to: 1. Define the meaning of concepts, principles and policies 2. Explain the different types of accounting concepts 3. Explain the different types of accounting principles.

4. Explain the major consideration governing accounting policies, changes in accounting policies and the disclosure if there is change in accounting policies. 5. Define the scope and functions of Accounting Standards 6. State the meaning and objectives of International Financial Reporting System. 2.2 Meaning of Accounting Principles, Concepts & Policies Accounting information is used by various stakeholders. Since all the stakeholders should understand the accounting language in the same sense, certain principles, concepts and policies of accounting have been laid down. Accounting Principles: Accounting Principles are basically the rules of action adopted by the accountants universally while recording accounting transactions. The principles are doctrines associated with theory and procedures and current practices of accounting. These principles may be classified as concepts and conventions. Concepts: Concepts take the form of assumptions or conditions, which guide the accountants while preparing accounting statements.

Based on this assumption, business person purchases fixed assets, uses long term source to fund the fixed assets etc. This strong assumption that the business will continue for a long period of time is called a concept.

Conventions: Conventions are those customs and traditions which guide the accountants while preparing the financial statements.

Accounting Policy: Accounting policy refers to the specific accounting principles and methods of applying those principles adopted by the enterprise in the preparation and presentation of financial statements.

The choice of selecting straight line method of depreciation or any other is the policy of the management. No management can exercise discretion regarding fundamental presumptions of accounting. But every management has a choice of making an accounting policy. Generally Accepted Accounting Principles: The double entry system of accounting is based on a set of principles which are called generally accepted accounting principles. It incorporates the consensus at a particular time as to: Which economic resources and obligations should be recorded as assets and liabilities by financial accounting, Which changes in assets and liabilities should be recorded, When these changes are to be recorded, How the assets and liabilities and changes in them should be measured, What information should be disclosed and Which financial statement should be prepared For example, an entity having research and development department may follow the policy of deducting all the R&D expenses incurred in a year as revenue expense while for the same situation another entity may classify R&D expenses into projects and may write off only when the project is not expected to offer any future benefits. Annexure 1 given at the end of this unit provides you details on the difference between US GAAP norms and Indian GAAP norms. Accounting Standards To bring uniformity in terminology, accounting concepts, conventions, and assumptions, the Institute of Chartered Accountants of India (ICAI) established Accounting Standards Board (ASB) in 1977. An Accounting Standard is a selected set of accounting policies or broad guidelines regarding the principles and methods to be chosen out of several alternatives. There are altogether 32 accounting standards issued by ASB out of which, one standard (AS8) has been withdrawn pursuant to AS26 becoming mandatory. Annexure 2 given at the end of this unit provides you the details of 32 Accounting Standards (AS).

Self Assessment Questions: 1. Accounting principles are _______, associated with theory and practice of accountings. 2. Accounting Principles are classified as ________ and ________. 3. Assets may be depreciated on fixed installment method or reducing balance method. Is it a concept or a convention? 4. A business is started with an assumption of making profit. Is this assumption, a concept or a convention? 5. The purpose of establishing ICAI and ASB is to ________. 6. How many accounting standards are issued by ASB so far? 2.3 Types of Accounting Concepts As said earlier, concepts are the basic assumptions or conditions upon which the science of accounting is based. There are five basic concepts of accounting, namely business entity concept, which is also termed as separate entity concept, going concern concept, money measurement concept, periodicity concept and accrual concept. Each concept is discussed below.

2.3.1 Business Separate Entity Concept The essence of this concept is that business is a separate entity and it is different from the owner or the proprietor. It is an economic unit which owns its assets and has its own obligations. This enables the business to segregate the transactions of the company from the private transactions of the proprietor(s).

This legal separation between business and ownership is kept in mind while recording the transactions in the books of business. Self Assessment Questions:

7. State true or false:


1. If the household expenses of Rs 25,000 of a proprietor are shown as business expenses, the profit of the business will be understated to the extent of Rs.25,000. 2. If a proprietor invests Rs.1,00,000 in the business, it is deemed that the proprietor has given Rs.1,00,000 to the business and it is shown as an asset in the books of the business.

8. Business and its owner are _______________ entities. 9. Profits earned in business form an addition to the _____________ of the owner. 2.3.2 Going concern concept The fundamental assumption is that the business entity will continue fairly for a long time to come. There is no reason why an enterprise should be promoted for a short period only to liquidate the business in the foreseeable future. This assumption is called going concern concept. This concept forms the basis for the distinction between expenditure that will yield benefit over a long period of time (Fixed Assets) and expenditure whose benefit will be exhausted in the short term (Current Asset). Similarly liabilities are classified as short term liabilities and long term liabilities. According to AS 1 issued by ICAI, if this concept is followed, this fact need not be disclosed in the financial statement since its acceptance and uses are assumed. In case this concept is not followed, the fact should be disclosed in the financial statement along with the reasons.

Self Assessment Questions: 10. Accounting of a small calculator as an expense and not as an asset is the application of principles of prudency. State true or false. 11. Classification of assets as current and fixed assets is the application of going concern concept. State true or false. 12. Purchase a building for your business is made under the assumption that it would last for a long period. This is in accordance with the materiality principle. State true or false.

13. What is the underlying intention in making a provision every year when an asset is purchased? 2.3.3 Money Measurement Concept All transactions of a business are recorded in terms of money. An event or a transaction that cannot be expressed in money terms, cannot be accounted in the books of accounts.

Self Assessment Questions: 14. An event or a transaction expressed in monetary value is measured but inflation or changes in the purchasing power are ignored in money measurement concept. Say yes or no. 15. Transactions or events should be expressed in ___________. 16. Revenues are matched with expense in accordance with money measurement principle. State true or false 2.3.4 Periodicity Concept The time interval for which accounts are prepared is an important factor even though we assume long life for a business.

The accounting period could be half year or even a quarter. The financial statements should be prepared at the end of each accounting period so that income statement shows profit or loss for that accounting period. So also a balance sheet is prepared to depict the financial position of the business. Self Assessment Questions:

17. The economic life of the entity is artificially split into periodic intervals in accordance with periodicity concept. State true or false 18. The accounting data must disclose all relevant information in accordance with periodicity concept. State true or false 19. The accountants are free to submit financial statement at arbitrary points in time during the life of the entity. This is in accordance with periodicity concept. State true or false. 2.3.5 Accrual Concept Profit earned or loss suffered for an accounting period is the result of both cash and credit transactions. It is possible that certain incomes are earned but not received and similarly certain expenses incurred but not yet paid during an accounting period. But it is relevant to consider them while computing the financial results just because they are related to the specific accounting period.

Similarly the expenses that are incurred for the accounting period could be paid after the accounting period. Such accrued expenses are deducted while calculating the profit for the accounting period. This is the accrual concept. Self Assessment Questions: 20. Interest earned but not received within an accounting period is called _______. 21. Following straight line method of depreciation of a particular asset year after year adhere to consistency concept. State true or false. 22. Accrued income should be _________ to compute profit and prepaid expenses should be _____ according to accrual concept of accounting. 23. Accrual concept considers not only cash transactions but also ______ transactions. 2.4 Accounting Principles Accounting Principles are the rules basing on which accounting takes place and these rules are universally accepted.

There are ten such basic principles, namely principle of income recognition, principle of expense, principle of matching cost and revenue, historical cost principle, principle of full disclosure, double aspect principle, modifying principle, principle of materiality, principle of consistency and principle of conservatism. A brief description is in the following paragraphs. 2.4.1 Principle of Income Recognition According to this concept, revenue is considered as being earned on the date on which it is realized, i.e., the date on which goods and services are transferred to customers for cash or for promise. It should further be noted that it is the amount which the customers are expected to pay which shall be recorded. In effect, only revenue which is actually realized should be taken to profit and loss account. Unrealized revenue should not be taken into consideration for determining the profit. Example:

Self Assessment Questions: 24. Income is considered as earned only when it is ____________. 25. Income is realized whether it is actually received in cash or promised to be received. Is it True or False? 26. Income realized is different from cash received. Is it true or false? 27. A sale is made on credit. Does it constitute income realization? 28. An order is received for sale of goods. Is it realization of income? 29. An order is received with an advance of Rs.100000 cash. Can this be called income? 2.4.2 Principle of Expense

Expenses are different from payments. A payment becomes expenditure or an expense only when such payment is revenue in nature and made for consideration.

Therefore all revenue expenses are transferred to profit and loss account to ascertain profit or loss of the business undertaking. In other words, there are revenue expenses and capital expenses. While revenue expenses are charged against profit, capital expenses are shown in the balance sheet as assets. Self Assessment Questions: 30. A cash payment may be a revenue payment or capital payment. Is it true or false? 31. A payment which is revenue in nature is expenditure. Is it true or false? 32. Plant is purchased and payment is made. Is it an expenditure or acquisition of asset? 33. All revenue expenses are charged against ___________. 34. Capital payments resulting in acquisition of assets appear in the balance sheet. True or False? 2.4.3 Principle of Matching Cost and Revenue Revenue earned during a period is compared with the expenditure incurred to earn that income, whether the expenditure is paid during that period or not. This is matching cost and revenue principle, which is important to find out the profit earned for that period. Here costs are reported as expenses in the accounting period in which the revenue associated with those costs is reported.

While preparing the final accounts adjustments are made for outstanding expenses, prepaid expenses, outstanding income and income received in advance. Self Assessment Questions: 35. Matching concept of accounting considers only revenue incomes and expenses relating to a particular accounting period. True or False? 36. Incomes and expenses for an accounting period are considered to compute _____.

37. Expenditure paid or payable and revenue earned whether realised or not in cash are taken into account to find out profit or loss. True or False? 38. For the actual revenue received, outstanding incomes are ________ and income received in advance are_________________ to find out the revenue income for the given period. 39. For the actual revenue expenses (costs) paid during the accounting period, outstanding expenses are _____ and prepaid expenses are _____ to find out expenses for the accounting period. 2.4.4 Principle of Historical Costs This is called cost principle. All assets are recorded at the cost of acquisition and this cost is the basis for all subsequent accounting for the assets. The expenses and the goods purchased are shown at the value at which they are incurred. The value of the assets is constantly reduced by charging depreciation against their cost to present their book value in the balance sheet.

However, on account of inflationary situations, this cost concept does not portray correct picture of the business and so inflation accounting has emerged. Self Assessment Questions: 40. All assets are shown at historical cost in balance sheet. True or False? 41. Depreciation is charged against the historical cost of assets. True or False? 42. Historical cost is the cost at which an asset is actually purchased. True or False? 43. Machinery is bought for Rs.200000 and its market value is Rs.80000. Which of these values do you consider to mention in the balance sheet according to cost principle? 44. Inflation accounting has emerged as a result of limitation of historical cost concept. True or False? 2.4.5 Principle of Full Disclosure The business enterprise should disclose relevant information to all the parties concerned with the organization. It means that any information of substance or of interest to the average investors will have to be disclosed in the financial statements.

The Companies Act, 1956 requires that income statement and balance sheet of a company must give a fair and true view of the state of affairs of the company. Self Assessment Questions: 45. The principle of full disclosure implies that information which is of ___________ should be stated in financial statements. 46. The material information that is disclosed should be of great interest to the average investors. True or False? 47. Non-disclosure of material information amounts to ___________. 48. Disclosing about assets without disclosing about liabilities is against the principle of full disclosure. True or False? 2.4.6 Double Aspect Principle This concept is the most fundamental one for accounting. A business entity is an independent unit and it receives benefits from some and gives benefits to some other. Benefit received and benefit given should always match and balance.

The total liabilities are equal to the total of assets. This is dual aspect of accounting. The established principle of accounting is that for every debit there is an equivalent credit and this is called double entry principle of accounting. Self Assessment Questions: 49. Under dual aspect principle, total benefits received by business should match with total benefits given. True or False? 50. Total liabilities should be equal to ___________ as per dual aspect principle. 51. For every debit, there should be an equivalent credit. This is called _________ of accounting. 2.4.7 Modifying Principle The modifying principle states that the cost of applying a principle should not be more than the benefit derived from. If the cost is more than the benefit, then that principle should be modified. This is called cost-benefit principle. There should be flexibility in adopting a principle and the advantage out of the principle should over weigh the cost of implementing the principle.

Self Assessment Questions: 52. Modifying principle is also known as _____________. 53. The modifying principles states that benefit derived should over weigh the cost of implementing it. State true or False? 54. A firm plans to establish costing department. By doing so it was estimated that the cost of the product would increase by 50%. Is it advisable to have cost-department? 2.4.8 Principle of Materiality While important details of financial status must be informed to all relevant parties, insignificant facts which do not influence any decisions of the investors or any interested group, need not be communicated. Such less significant facts are not regarded as material facts. What is material and what is not material depends upon the nature of information and the party to whom the information is provided. While income has to be shown for income tax purposes, the amount can be rounded off to the nearest ten and fraction does not matter. The statement of account sent to a debtor contains all the details regarding invoices raised, amount outstanding during a particular period. The information on debtors furnished to Registrar of Companies need not be in detail. Self Assessment Questions: 55. Principle of materiality states that relevant information should be given to relevant parties. True or False? 56. Details of debtors should be given to creditors. True or False? 57. The material information to one party need not be so for another party. True or False? 58. The method of depreciation adopted should be disclosed to Income Tax Authorities. True or False? 2.4.9 Principle of Consistency Consistency is required to help comparison of financial data from one period to another. Once a method of accounting is adopted, it should not be changed. For instance if stock is valued under FIFO method in first year it should be valued under the same method in the subsequent years also. Likewise if the firm chooses to depreciate assets under diminishing balance method, it should continue to do so year after year, unless the management takes a policy decision to change the depreciation method. Any change in the accounting methods should be informed to the concerned authorities with justification. Self Assessment Questions:

59. The purpose of principle of consistency is to help for ______ from one period to another period. 60. Consistency principle helps for proper assessment of profit or loss. True or False? 2.4.10 Principle of Conservatism or Prudence Accountants follow the rule anticipate no profits but provide for all anticipated losses . Whenever risk is anticipated sufficient provision should be made. The value of investments is normally taken at cost, even if the market value is higher than the cost. If the market value expected is lower than the cost, then provision should be made by charging profit and creating investment fluctuation fund. This is the principle of conservatism and it does not mean that the income or the value of assets should be intentionally under stated. Self Assessment Questions: 61. Provision should be made whenever _____________ is anticipated. 62. The underlying spirit of principle of conservatism is __________ . 63. State the name of the relevant accounting principles for the following statements. a. Following FIFO method of stock valuation year after year b. Appending notes to the financial statements. c. Anticipate no profit but provide for all probable losses 2.5 Accounting Policies It refers to specific accounting principles and methods of accounting adopted by the enterprise while preparing and presenting the financial statements. The management of each enterprise has to select appropriate accounting policies based on the nature and circumstances of the business they are in. Some of the areas in which different accounting policies may be adopted are: Methods of depreciation, amortization, Treatment of expenditure during construction, Conversion or translation of foreign currency items, Valuation of inventories, Treatment of goodwill, Valuation of investments,

Treatment of retirement benefits, Recognition of profit on long-term contract, Valuation of fixed assets and Treatment of contingent liabilities. The major considerations governing the selection and application of accounting policies are: a. Prudence: Uncertainties are a fact and it is inevitable. This should be recognized by exercising prudence in preparing financial statements. b. Substance Over form: Transactions and events should be accounted for and presented in accordance with their substance and financial reality and not merely with their legal form. c. Materiality: Financial statement should disclose all material items which might influence the decision of the users of the financial statement. 2.5.1 Change in Accounting Policies: The change in accounting policy is recommended only in the following circumstances: a. If it is required by statute for compliance with an accounting standard b. If is considered that the change would result in a more appropriate presentation of the financial statements of an enterprise. 2.5.2 Disclosure in case of change in Accounting Policy: If change has a material effect in current period and the effect of change is ascertainable the amount of change should be disclosed. If the change has a material effect in current period and the effect of change is not ascertainable wholly or in part, the fact should be disclosed. If change has no material effect in current period but which is reasonably accepted to have a material effect in later periods, the fact of such change should be appropriately disclosed. 2.6 Accounting Standards It is a selected set of accounting policies or broad guidelines regarding the principles and methods to be chosen out of several alternatives. Standards adhere to certain laws, customs, usage and business environment in which it operates.

The basic purpose of accounting standards is to harmonize the diverse accounting policies and practices that are currently used in India. The adoption of accounting standards ensures uniformity, comparability and qualitative improvement in the preparation and presentation of financial statement. Accounting standards facilitates more disclosure of financial information that is beyond the statutory limits. 2.6.1 Scope and functions of Accounting Standard Board The Accounting standards board (ASB) was constituted by The Institute of chartered Accountants of India on 21st April, 1977. The aim of the board was to bring in uniformity among various accounting policies and practices that were practiced in India. To determine the broad areas in which accounting standards need to be formulated and hold a dialogue with various representatives of the government, industry and other organization to ascertain their views. On the basis of the discussions they hold, the board prepares and issues a draft proposal for comments by the members of the Institute and the public. After modification (if any) in the draft proposal a final draft is submitted to the council of the institute. The accounting standard will then be issued under the authority of the Council. At present there are 30 accounting standards issued by the Council. 2.6.2 International Financial Reporting System: IFRS are standards, interpretations and framework for the preparation and presentation of financial statements. IFRS was framed by International Accounting Standards Board (IASB). The objective of financial statement is to provide information about the financial position, performance and changes in the financial position of an entity. It should also provide the current financial status of the entity to all the users of financial information. IFRS follows accrual basis of accounting and the financial statements are prepared on the basis that an entity will continue for the foreseeable future. IFRS helps entities access global capital market with ease. Under IFRS, we need to submit a statement of financial position (Balance Sheet), Comprehensive income statement (Profit & Loss/ Income and Expenditure account), either a statement of changes in equity or statement of recognized income or expenses, cash flow statement and notes including summary of significant accounting policies. 2.7 Summary Accounting Principles are basically the rules of action adopted by the accountants universally while recording accounting transactions. Accounting conventions are those customs and traditions which guide the accountants while preparing the financial statements.

Accounting concepts are the basic assumptions or conditions upon which the science of accounting is based. There are five basic concepts of accounting, namely business entity concept (separate entity concept), going concern concept, money measurement concept, periodicity concept and accrual concept. Accounting policy refers to the specific accounting principles and methods of applying those principles adopted by the enterprise in the preparation and presentation of financial statements. Different types of Accounting Principles are Principles of Income recognition, Principles of expense, Principles of matching cost and revenue, Principles of Historical cost, Principle of full disclosure, Principles of double aspects, Modifying Principles, Principles of Materiality, Principles of consistency, and Principles of conservatism or prudence. An Accounting Standard is a selected set of accounting policies or broad guidelines regarding the principles and methods to be chosen out of several alternatives. There are altogether 28 accounting standards issued by ASB which have to be adopted by management of different enterprises to improve the quality of presentation of financial statements in our country. 2.8 Terminal Questions 1. What are the basic principles of Accountancy? 2. The salaries paid in 2004 Rs.500000; Salaries outstanding Rs.20000; Salaries paid in advance for 2004 Rs.30000; What is the actual salary expenditure for 2004? What is the accounting principle involved in this? 3. What is wrong if assets like buildings are shown at market value in the balance sheet? 4. A business receives capital of Rs.100000 and a loan is raised for Rs.50000. This is represented by cash Rs.15000; Machinery Rs.85000; Furniture Rs.20000 and goods Rs30000. Find the total of debits and credits from business point of view. What principle of accounting is underlying in this case? 5. What is substance over form?
2.9 Answer for Self Assessment Questions

SAQ 1: 1. Doctrines

20. Accrued interest 21. True

45. Substance 46. True

2. Concepts, conventions 22. Added, deducted 47. Fraud 3. Convention 4. Concept 23. Credit 24. Realized 48. True 49. True

5. Bring uniformity in 25. True accounting terminology and principles 26. True 6. 32 7. a. True, b. False 8. separate 9. capital 10. False 11. True 12. False 13. To replace it after a certain period. 14. Yes 36. Profit or loss 15. Monetary value 37. True 16. False 17. True 18. False 19. False 27. Yes 28. No 29. No 30. True 31. True

50. Total Assets 51. Double entry principle 52. Cost-benefit principle 53. True 54. No 55. True 56. False

32. Asset Acquisition 57. True 33. Profit 34. True 35. True 58. True 59. Comparison 60. True 61. Risk

62. Anticipate no profit but provide for all anticipated 38. Added, Deducted losses 39. Added, Deducted 63. a. Principles of consistency 40. True b. Principles of full disclosure 41. True 42. True 43. Rs.200000 44. True c. Principles of conservatism

Answers for Terminal Question: 1. Income recognition, principle of expense, matching of cost and revenue, historical cost principle, full disclosure principle, double aspect principle, modifying principle, materiality principle, consistency principle and conservatism principle.

2. Rs.490000 (500000 + 20000 30000); Matching cost and revenue principle. 3. If assets like building are shown at market value instead of historical cost in the balance sheet, the profit or loss arising out of such valuation is against to the principle of income recognition. The profit or loss is said to arise only when the asset is sold or revalued for a specific purpose. The day when the assets are valued, the market value may be high and later the prices may fall. Therefore it is wrong to consider the unrealized or anticipated profit. Hence the assets should be shown at historical cost in the balance sheet. 4.

It is as per double aspect principle. 5. Refer unit 2.5 ANNEXURE-1

US GAAP It is established under FASB and AICPA

INDIAN GAAP It is established under ICAI

Balance Sheet, Income Statement Balance Sheet and income statement & Funds Flow Statement are are alone mandatory mandatory Any change in foreign exchange Any difference in foreign exchange fluctuations cannot be capitalized can be capitalized. but the difference can be shown or debited to Income statement Financial accounting, Management Only financial accounting and accounting and income tax Income tax accounting are prepared. accounting are prepared separately The basic tenets is globalization of The basic tenet is localization business

Any long term loan repayable is Long term loans maturing in the the current financial year is shown current financial year need not be separately disclosed separately In lease contract, lessee is more beneficiary because he can claim depreciation allowance In lease contract, lessor is eligible for depreciation allowance and not the lessee.

It is more transparent and accepted It is comparatively less transparent. worldwide. More disclosure is For listing the securities in other required countrys stock exchange USGAAP is mandatory ANNEXURE 2 ACCOUNTING STANDARDS

AS No Title

Recommendary or Mandatory

Mandatory from accounting period beginning on or after 1.4.1991

AS-1

Disclosure of Accounting Mandatory Policies Valuation of Inventories Mandatory Cash Flow Statement Mandatory

AS-2 AS-3 AS-4

1.4.1999 1.4.2000 1.4.1995

Contingencies and events Mandatory occurring after the Balance Sheet date (revised) Prior Period and extraordinary items and changes in Accounting Policies Depreciation Accounting (revised) Mandatory

AS-5

1.1.1987

AS-6

Mandatory

1.4.1995

AS-7

Accounting for construction contracts Accounting for Research and Development Revenue Recognition Accounting for Fixed Assets

Mandatory

1.4.1991

AS-8

Mandatory

1.4.1991

AS-9 AS-10

Mandatory Mandatory

1.4.1991 1.4.1991

AS-11

Accounting for the effects Mandatory of changes in foreign exchange rates (Revised) Accounting for Government Grants Mandatory

1.4.1995

AS-12

1.4.1994

AS-13 AS-14

Accounting for Investment Accounting for Amalgamation Mandatory

1.4.1995

AS-15

Accounting for retirement Mandatory benefits in the financial statement of employers Borrowing Costs Segment Reporting Related party disclosure Leases Earnings Per Share Consolidated financial statement Accounting for taxes on Mandatory Mandatory Mandatory Mandatory Mandatory Mandatory

1.4.1995

AS-16 AS-17 AS-18 AS-19 AS-20 AS-21

1.4.2000 1.4.2001 1.4.2001 1.4.2001 1.4.2001 1.4.2001

AS-22

Mandatory

1.4.2001

income AS-23 Accounting for investments Mandatory in associates in consolidated financial statements Discontinuing operations Mandatory 1.4.2002

AS-24 AS-25 AS-26 AS-27

1.4.2002 1.4.2002 1.4.2003 1.4.2002

Interim financial reporting Mandatory Intangible assets Financial reporting of interests in joint venture Impairment of assets Provisions, Contingent Liabilities & Contingent Assets Mandatory Mandatory

AS-28 AS-29

Mandatory Mandatory

1.4.2004 1.4.2004

AS-30

Financial Instruments: Mandatory from 1.4.2009 Recognition, Measurement 1.4.2011 and Limited Resources Financial Instruments: Presentation Mandatory from 1.4.2009 1.4.2011

AS-31

AS-32

Financial Instruments: Mandatory from 1.4.2009 Disclosure and limited 1.4.2011 revision to AS 19 (leases)

AS 8 was withdrawn in pursuant to AS 26 becoming mandatory. 29 accounting standards are issued as of date and only 28 is applicable. AS 30, 31, 32 are published but they will come into effect from 1.4.2009. It is mandatory on or after 1.4.2011 Copyright 2009 SMU

Powered by Sikkim Manipal University


.

MB0041-Unit-03-Double Entry Accounting


Unit-03-Double Entry Accounting Structure: 3.1 Introduction Objectives 3.2 Meaning of double entry accounting 3.3 Classification of accounts under Traditional approach 3.4 Classification of accounts under Accounting Equation approach 3.5 Comparison of traditional approach with Modern approach equal approach 3.6 Accounting Trail 3.7 Transactions and events 3.8 Meaning and roles of debit and credit 3.9 Accounting equation 3.10 Summary 3.11 Terminal Questions

3.12 Answers 3.1 Introduction In the previous unit we had dealt with different types of accounting principles, concepts, policies and accounting standards. Accounting Principles are basically the rules of action adopted by the accountants universally while recording accounting transactions. Accounting concepts are assumptions which guide the accountant in preparation of financial statements while accounting conventions are customs and traditions followed in preparation of the financial statements. Accounting policies are those concepts and conventions adopted by the management based to the situations but more importantly, consistency is needed while adopting them. Accounting standards is a select set of accounting policies, methods chosen by the firm. In this unit, we have dealt the meaning of double entry accounting, the classification of accounts under traditional approach and accounting equation approach. This classification is needed because business entity may not be confined to the geographical boundaries of the nation. Accounting trial is a sequential order in which the accounting process flows. The eight process involved in accounting trail is discussed briefly in this unit. Finally a brief explanation of the rules of debit and credit on various types of accounts is dealt. Finally a brief explanation of the rules of debit and credit on various types of accounts is dealt. Objectives: After going through this unit, you should be able to: 1. Define Double entry book keeping. 2. Classify different types of accounts under traditional approach 3. Classify different types of accounts under Accounting equation approach 4. Compare traditional approach with modern approach 5. State the process involved in accounting trail. 6. State the meaning of transactions and events 7. Explain the rules of debit and credit The students should be able to appreciate the double entry system and know the accounting process. 3.2 Meaning of Double Entry Accounting

We have learnt that the dual aspect recording is the most important accounting concept. According to the concept, every business transaction involves receiving aspect and giving aspect. A transaction is a business activity involving transfer of money or moneys worth. Double Entry Book-Keeping System: Every account has two sides: 1) One account is the receiver of the benefit 2) Other account is the giver of the benefit. It must be noted that the amount of benefit received by one account is equal to the amount of benefit given by the other account. This enables us to record the two effects of any busines transaction. If capital is brought in by the owner of the business unit, the owner is the giver of the benefit and the business unit is the receiver of the benefit. It is a liability to the business unit and it is equally balanced by an asset in the business unit, in the form of cash received towards capital. Therefore every liability is represented by an asset. This is also expressed as every debit has an equivalent credit. Illustration1: We shall consider five transactions and show how they are accounted for in the books of the business. 1. Mr. Abhi brings Rs.100000 cash as capital into his business. 2. He purchases furniture to his shop Rs.10000 3. He buys goods for cash Rs.50000 4. He sells goods worth Rs.30000 for Rs.40000 on credit to Arjun 5. He pays wages to servants Rs.1000 Transaction 1: The business receives capital in cash. Capital is a liability and cash is an asset to the business.

Liability Capital 100000

Asset Cash 100000

Transaction 2: Furniture is purchased for cash. This transaction can be reflected as under

Capital

100000

Cash Rs. (100000- 10000)

90000

Furniture Total 100000 Total

10000 100000

Transaction 3: Purchased of goods for cash. This can be reflected in the statement as under.

Capital

100000

Cash Rs (90000 50000) Furniture Stock of goods

40000 10000 50000 100000

Total

100000

Total

Transaction 4: Sold goods to Arjun on credit for Rs.40000, the cost of which is only Rs. 30000. In this transaction the affected accounts are Goods account, Arjun account and Profit & Loss account. Since the profit belongs to the owner it is fair to add it to the owners capital. The effect of this transaction can appear on the statement as shown below:

Capital Profit

100000 10000

Cash Furniture Stock of goods (50000-30000) Arjun (Debtors)

40000 10000 20000

40000 110000

110000

Transaction 5: Payment of wages Rs.1000.The cash balance gets reduced in the asset side and profit gets reduced as a result of the expenditure (wages account) on the liability side. This changes the statement as shown below:

Capital

100000

Cash (40000 1000) Furniture

39000 10000

Profit (10000-1000) 9000

Stock of goods Arjun (debtors) 109000

20000 40000 109000

From the above illustrations, it is clear that every transaction has dual effect. Recording these aspects is the fundamental idea behind double entry system of book keeping. Self Assessment Questions 1: 1. The system of recording transactions based on dual aspect concept is called a) Double account system b) Double entry system c) Single entry system 2. Show the dual aspect effect of the following transactions on the assets and liabilities of business. a. Purchased goods for cash Rs.80000 b. Purchased delivery van on credit for Rs.400000 c. Paid Rs.5000 to a supplier of goods on credit d. The proprietor withdrew Rs.20000 from the bank account of business for Personal expenses.

3.3 Classification of accounts under Traditional Approach The identification of the accounts affected in the transactions is a major task. There are three types of accounts, namely personal accounts, real accounts and nominal accounts.

Personal Account

Deals with accounts of individuals like creditors, debtors, bank account, outstanding/prepaid accounts It represents various asset accounts both tangible and intangible.

Real Account

Nominal Account

It consists of various types of expenses or incomes or loss or profit.

Personal Account:

Real Accounts: Real accounts are those which may be tangible real accounts and intangible real accounts. Tangible real accounts relate to things that can be touched, felt, physically measurable. Building account, furniture account, stock account, cash account etc are tangible real accounts. Intangible real accounts are such that they cannot be seen or touched. They can be measured in terms of money such as goodwill, patent rights etc. Nominal Accounts: Nominal accounts are also known as impersonal accounts. They are in the form of expenses or losses, incomes or gains. They do not really exist in physical form, but behind every nominal account cash is involved. For example, salary account is a nominal account and when salary is paid cash goes out and there is nothing in physical form. Therefore salary account is regarded as nominal account. Similarly all expenses and losses and all incomes and gains accounts are regarded as nominal accounts. 3.4 Classification of accounts according to Accounting Equation Approach: The preparation of financial statements is the objective of accounting. Accounting as an information processing system, should facilitate in the preparation of income statement and balance sheet. This in turn, would require that the terms in the accounting equation should be the summary of all the accounting records from an accounting system. Accounting equation approach classifies different types of accounts into assets account, liabilities account, capital account, revenue account and expenses account.

Types of Accounts Asset account

Meaning

Examples

Deals with tangible and Land a/c, Building a/c, Plant intangible real assets. & Machinery a/c, Cash a/c, Good will a/c, Trademark a/c, Patents a/c, Investments a/c

Liabilities account

Deals with the financial Long term loans, Debentures, obligations of the firm Bank loans, Trade creditors, on outsiders. Outstanding expenses. Deals with accounts of Capital a/c, Drawings a/c the owners of the company Deals with amount Sales a/c, Royalty received charged for goods sold a/c, interest received a/c, or service rendered, and dividend received a/c other incomes. Deals with expenses Purchases a/c, Discount incurred in the process allowed a/c, Interest paid a/c, of earning revenue Loss by fire a/c.

Capital account

Revenue account

Expenses account

3.5 Comparison of traditional approach with Modern approach or Accounting Equation approach:

Traditional approach Personal account: (other than those relating to owner) having debit balance (other than those relating to owners) having credit balance Those relating to owners Real account Nominal account: Relating to expenses Relating to revenue Illustration 2:

Modern approach Asset a/c Liabilities a/c Capital a/c

Asset a/c Expenses a/c Revenue a/c

Classify the following accounts according to Traditional and Modern approach

1. Capital a/c 4. Purchases account 7. Carriage outward 10. Interest paid 13. Comm received

2. Drawings a/c 5. Sales account 8. Cash received 11. Interest received 14. Discount allowed

3. Building purchased 6. Carriage inward 9. Cash paid 12. Commission paid 15. Conveyance charges 18. Subscription paid.

16. Sales promotion exp17. Entertainment expenses 19. Subscription received 22. Repairs incurred 20. Light,Power & Electricity

21. Telephone, postage and telegram

23. Insurance premium 24. Bad debts written off paid 27. Postage & stationery purchased 30. Wages & Salaries paid 33. Loan a/c of a partner

25. Bad debts recovered26. Discount received

28. F&F purchased 31. Travelling expenses 34. Sales return 37. O/s salaries a/c 40. Interest received in advance Solution:

29. Bank a/c 32. current a/c of the partner

35. Bank overdraft a/c 36. Loan a/c of the partner 38. Prepaid rent a/c 39. Interest accrued a/c

According to Traditional Approach:

Personal a/c Real a/c Nominal a/c

1, 2, 29, 32, 33, 35, 37, 38, 39, 40 3, 8, 9, 28 4, 5, 6, 7, 10, 11, 12, 13, 14, 15, 16, 17, 18, 19, 20, 21, 22, 23, 24, 25, 26, 27, 30, 31, 34, 36

According to Accounting Equation or Modern Approach:

Asset a/c Liabilities a/c Capital a/c Revenue a/c Expenses a/c

3, 8, 9, 28, 29, 38, 39 33, 35, 37, 40 1, 2, 32 5, 11, 13, 19, 25, 26, 34 4, 6, 7, 10, 12, 14, 15, 16, 17, 18, 20, 21, 22, 23, 24, 27, 30, 31, 36

Self Assessment Questions: 3. State yes or no a) State Bank of India is a Nominal Account. b) Machinery is a Real Account. c) Life Insurance Corporation is a Personal Account. d) Proprietors Capital Account is a Personal Account. e) Loan Account is a Real Account. f) Postage and Telegram Account is a Nominal Account. g) Interest on investment Account is a Nominal Account. h) Carriage on Goods Account is a Real Account i) The giver of a benefit must be debited.

j) Nominal Accounts fall in the category of impersonal accounts. k) Every debit has an equal and corresponding credit. l) Loss by fire is a Nominal Account. m) Outstanding Wages Account is a Nominal Account. n) Brokerage Account is a Real Account. o) Investment Account is a Personal Account. 3.6 Accounting Trail Accounting Trial is a sequential order in which the accounting process flows. All transactions are recorded first in a book called journal. The transactions are posted to the respective accounts, maintained in a separate book called ledger. Later, all adjustments such as opening entries, closing entries, adjusting entries are made in a book called journal proper and thereafter, the ledger balances are summarized to form a trial balance. From trial balance, trading account, profit and loss account and balance sheet are prepared. Accounting trail is the process of:

Self Assessment Questions: 4. Accounting trial is a process starting from identifying the transactions or events to preparation of final statement of accounts. True or False 5. There are three types of accounts namely ____________ and ________________. 6. A trial balance is the summarized form of ledger balances. True or False

3.7 Transactions and Events A transaction is a business activity involving transfer of money or moneys worth. It may be cash transaction or credit transaction. In cash transaction cash flows immediately where as in credit transaction cash will be paid or received at future date. Assets acquired or sold, liabilities incurred or paid, expenses paid or payable, incomes received or receivable are all business transactions. But there are events which are neither cash nor credit transactions but it has an impact on the financial position of a business. These events may include provision for bad debts, provision for repairs, depreciation, taxation, transfer of profit towards reserve fund or sinking fund or investment fluctuation fund, etc., Events happen as a result of internal policies or external needs. In accounting, transactions and events have equal relevance and they must be recorded to arrive at the financial results of the business concern. Self Assessment Questions: 7. A transaction is a business activity which involves transfer of money or moneys worth. True or False 8. An event happens as a result of internal policy of an organization. True or False 9. Business transactions and events have equal importance in finding the financial results of the business concern. True or False? 10. Identify the following as transactions or events as the case may be. i) Depreciation of assets :___________ ii) Tax rates announcement :___________ iii) Acquisition of assets :___________ iv) Selling an asset :____________ v) Transfer of profits to Reserve Fund:______________

3.8 Meaning and rules of debit and credit Debit and credit are the two words basic for accounting. Debit represents receiving aspect and credit represents giving aspect. The rules of debit and credit when accounts are classified on traditional approach:

Type of accounts

DEBIT

CREDIT

Personal account Real account Nominal account

The receiver What comes in Expenses & losses

the giver what goes out Income and gains

The rules of debit and credit when accounts are classified on Accounting Equation Basis:

Types of accounts Asset account Liabilities account Capital account Revenue account Expenses account

Rules for Debit Debit the increase Debit the decrease Debit the decrease Debit the decrease Debit the increase

Rules for credit Credit the decrease Credit the increase Credit the increase Credit the increase Credit the decrease

Illustration 3: Analyze the following transactions according to Traditional approach and modern approach: a. Subramanya started his business with cash b. Borrowed from Mahesh c. Purchased furniture d. Purchased furniture from Mohan on credit e. Purchased goods for cash f. Purchased goods from Ram on credit g. Sold goods for cash h. Sold goods to Shyam on credit i. Received cash from Shyam j. Paid cash to Ram

k. Deposited into bank l. Withdrew cash for personal use m. Withdrew from bank for office use n. Withdrew from bank for personal use o. Received a cheque from a customer, shyam at 5 pm p. Deposited Shyams cheque next day q. Bank intimated that shyams cheque was dishonored r. Paid Ram by cheque s. Paid salary t. Paid rent by cheque u. Goods withdrawn for personal use v. Paid an advance to suppliers of goods w. Received an advance from customers x. Paid interest on loan y. Paid installment of loan z. Interest allowed by bank Solution: Analysis of Transaction under Traditional Approach

Sl. no Accounts involved Cash a/c a Capital a/c Cash a/c b Loan from

Nature of account Real Personal Real Personal

How affected Cash is coming in

Debit/ Credit Debit

Subramanya is the giver Credit Cash is coming in Debit Mahesh is the giver Credit

Mahesh Furniture a/c c Cash a/c Furniture a/c d Mohan a/c Purchase a/c e Cash a/c Purchase a/c f Rams a/c Cash a/c g Sales a/c Shyams a/c h Sales a/c Cash a/c i Shyams a/c Rams a/c j Cash a/c Bank a/c k Cash a/c Drawings a/c l Cash a/c Cash a/c m Bank a/c Drawings a/c n Bank a/c Cash a/c o Shyam a/c

Real Real Real Personal Nominal Real Nominal Personal Real Nominal Personal Nominal Real Personal Personal Real Personal Real Personal Real Real Personal Personal Personal Real Personal

Furniture is coming in Debit Cash is going out Credit Furniture is coming in Debit Mohan is the giver Credit Purchase is an expense Debit Cash is going out Credit Purchase is an expense Debit Ram is the giver Cash is coming in Sales is revenue Shyam is the receiver Sales is revenue Cash is coming in Shyam is the giver Ram is the receiver Cash is going out Bank is the receiver Cash is going out Subramanya is the receiver Cash is going out Cash is coming in Bank is the giver Subramanya is the receiver Bank is the giver Cash (cheque) is coming in Credit Debit Credit Debit Credit Debit Credit Debit Credit Debit Credit Debit Credit Debit Credit Debit Credit Debit Credit

Bank a/c p Cash a/c Shyam a/c q Bank a/c Rams a/c r Bank a/c Salary a/c s Cash a/c Rent a/c t Bank a/c Drawings a/c u Purchase a/c Advanced to v Suppliers a/c Cash a/c Cash a/c w

Personal Real Personal Personal Personal Personal Nominal Real Nominal Personal Personal Nominal Personal Real

Shyam is the giver Bank is the receiver

Debit

Cash (cheque) is going Credit out Shyam is the receiver Debit Bank is the giver Ram is the receiver Bank is the giver Salary is an expense Cash is going out Rent is an expense Bank is the giver Subramanya is the receiver Decrease is stock Suppliers are the receivers Cash is going out Credit Debit Credit Debit Credit Debit Credit Debit Credit Debit Credit

Real

Cash is coming in Customers are givers Interest is expense Cash is going out Lender is the receiver Cash is going out Bank is the receiver Bank interest is an income

Debit Credit Debit Credit Debit Credit Debit Credit

Advance from Personal customers a/c Interest on loans Nominal a/c Real Cash a/c Loan a/c Personal Real Personal

y Cash a/c Bank a/c z Bank interest a/c Nominal

3.9 Accounting equation The preparation of balance sheet is the final step in accounting process. The accounting equation indicates that the sources of funds should be equal to uses of funds. In other words, proprietors equity and liabilities to outsiders should be equal to assets.

Where L is liabilities, P is Proprietor equity and A is assets. Steps involved in developing accounting equation: Ascertain the variables (Assets, Liabilities or capital) of an equation affected by the transaction. Find out the effect (in terms of increase or decrease) of a transaction on the variables of the equation Show the effect on the appropriate side of an equation.

Illustration 3: Transaction 1: Started business with Rs.1, 00,000.

Variables affected

Asset and capital

Effect of the transaction Increase in asset and capital Asset = Liabilities + Capital Accounting Equation 1,00,000 = 0 + 1,00,000 Transaction 2: Purchased Goods for cash Rs.20,000

Variables affected Effect of the transaction

Asset Increase in asset (Stock) and decrease in another asset (cash) Asset = Liabilities + Capital -20,000 +20,000 = 0 + 0

Accounting Equation

Transaction 3: Sold goods costing Rs.10,000 for cash Rs.12,000.

Variables affected Effect of the transaction

Asset and capital Increase in asset (Cash) and decrease in another asset (Stock) and increase in capital Asset = Liabilities + Capital - Stock + Cash -10,000 +12,000 = 0 + 2,000

Accounting Equation

Self Assessment Questions: 11. Liabilities plus Equity is equal to ____________________________. 12. Assets minus liabilities to outsiders are equal to __________________. 13. If assets are Rs.5 lakhs, liabilities are Rs.3 lakhs, find out the equity. 14. If Owners equity is Rs.3 lakhs, Outsider liabilities are Rs.2 lakhs, Owners share of profit is Rs.1 lakhs, find out the total value of assets. 15. Every transaction influences balance sheet and it is shown by accounting equation True or False?

Illustration 4: Show what accounts are affected in the following transactions. Also show the accounting equation for the transactions 1. Madan commenced business with cash Rs. 70000 2. Purchased goods on credit 14000

3. Withdrew for private use 3000 4. Goods purchased for cash 12000 5. Paid wages 5000 6. Paid to creditors 10000 7. Sold goods on credit (cost price Rs18000) 22000 8. Sold goods for cash (Cost price Rs.3000) 6000 9. Purchased furniture for cash 5000 10. Received from debtors 11000 Solution:

Transaction Accounts affected Account to be debited and account No in the books of the to be credited business 01 Capital account and Cash account being real account is cash account debited and Capital account being personal account is credited Goods account and Goods account being real account is creditors account debited and creditors account being personal account is credited Personal drawings Drawings account being personal account and cash account is debited and cash account account being real account is credited Goods account and Goods account being real account is cash account debited and cash account being real account is credited Wages account and Wages account being nominal account cash account is debited and cash account being real account is credited Cash account and Creditors account being personal

02

03

04

05

06

creditors account

account is debited and cash account being real account is credited

07

Goods account, Debtors account being personal Debtors account account is debited, profit transferred to and profit account capital account being personal account is credited and goods account being real account is also credited Furniture account and Cash account Furniture account being real account is debited and cash account being real account is credited Cash account being real account is debited and debtors account being personal account is credited.

09

10

Cash account and debtors account

Accounting equations for the transactions

3.10 Summary Double entry system has two effects: 1) One account is the receiver of the benefit, 2) Other account is the giver of the benefit. Every transaction has dual effect and recording these two

aspects which are known as debit and credit aspects is the fundamental idea behind double entry system of book keeping. Under traditional approach accounts are classified into personal account, real account and nominal account. Under accounting equation or modern approach, the accounts are classified into asset a/c, liability a/c, capital a/c, revenue a/c and expenses a/c. A transaction is a business activity involving transfer of money or moneys worth. It may be cash transaction or credit transaction. Events are neither cash nor credit transactions but it has an impact on the financial position of a business. Events happen as a result of internal policies or external needs. Accounting Trial is a sequential order in which the accounting process flows. The accounting equation indicates that the sources of funds should be equal to uses of funds. In other words, proprietors equity and liabilities to outsiders should be equal to assets.
3.11 Terminal Questions

1. The accounting equation is Assets = _______________ + _______________. 2. State the meaning of double entry book keeping. 3. What is accounting trail? 4. Find the value of the following: a. If the total assets are Rs87000 and the liabilities are Rs47000, find out the amount of capital. b. If the capital of proprietor is Rs400000 and the total assets are Rs600000, what is the amount of liabilities to outsiders? c. If creditors are Rs56000, bank overdraft is Rs100000 and outstanding expenses are Rs.8000, what is the total amount of assets? d. Fixed assets are Rs.70000 and current assets are Rs.100000 and the creditors are Rs.30000. What is capital? 3.12 Answer for Self Assessment Questions

Answers for Terminal Questions: 1. Liabilities + Owners capital 2. Every transaction has two aspects, debit and credit and for every debit there is equivalent credit. 3. Refer Unit 3.6 4. a) Rs.40000 b) Rs.200000 c) Rs.164000 d) Rs.140000 Answer to Activity 1: Analysis of the transaction using Modern approach

Accounts involved Cash a/c A Capital a/c Cash a/c B Loan from Mahesh a/c Furniture a/c C Cash a/c Furniture a/c D Mohan a/c Purchase a/c E Cash a/c

Nature of account Asset Capital Asset Liability Asset Asset Asset Liability Expenses Asset

How affected Debit/ credit Increased Increased Increased Increased Increased Decreased Increased Increased Increased Decreased Debit Credit Debit Credit Debit Credit Debit Credit Debit Credit

Purchase a/c F Rams a/c Cash a/c G Sale a/c Shyams a/c H Sales a/c Cash a/c I Shyams a/c Rams a/c J Cash a/c Bank a/c K Cash a/c Drawings a/c L Cash a/c Cash a/c M Bank a/c Drawings a/c N Bank a/c Cash a/c O Shyam a/c Bank a/c P Cash a/c Shyam a/c Q Bank a/c Rams a/c R Bank a/c Salary a/c S Cash a/c

Expenses Liability Asset Revenue Asset Revenue Asset Asset Liability Asset Asset Asset Capital Asset Asset Asset Capital Asset Asset Asset Asset Asset Asset Asset Lability Asset Expenses Asset

Increased Increased Increased Increased Increased Increased Increased Decreased Decreased Decreased Increased Decreased Decreased Decreased Increased Decreased Increased Decreased Increased Increased Increased Decreased Increased Decreased Decreased Decreased Increased Decreased

Debit Credit Debit Credit Debit Credit Debit Credit Debit Credit Debit Credit Debit Credit Debit Credit Debit Credit Debit Credit Debit Credit Debit Credit Debit Credit Debit Credit

Rent a/c T Bank a/c Drawings a/c U Purchase a/c Advanced a/c V Suppliers a/c Cash a/c W

Expenses Asset Capital Expenses Asset Asset Asset

Increased Decreased Increased Decreased Increased Decreased Increased Increased Increased Decreased Decreased Decreased Increased Increased

Debit Credit Debit Credit Debit Credit Debit Credit Debit Credit Debit Credit Debit Credit

Advance from Liability customers a/c Interest on loans Expenses a/c Asset Cash a/c Loan a/c Liability Cash a/c Bank a/c Asset Asset

Z Bank interest a/c Revenue

Answer to Activity 2

Copyright 2009 SMU

Powered by Sikkim Manipal University


.

MB0041-Unit-04-Secondary Books
Unit-04-Secondary Books Structure: 4.1 Introduction Objectives 4.2 Secondary books 4.3 Purchases Book/Purchases Day book Cash discount, Trade discount Difference between cash discount and trade discount 4.4 Sales Book or Sales Day book Purchase Returns Book

Sales Returns Book 4.5 Bills receivable book Bills payable book Cash book 4.6 Posting to Ledger accounts Posting to Ledger Summary 4.7 Terminal Questions 4.8 Answers
4.1 Introduction

In the previous unit we discussed the meaning of double entry accounting, the various classifications of accounts both under traditional approach and accounting equation approach. Accounting trial is a sequential order in which the accounting process flows. Journal is a book of original entry. Journal is basically a day book in which transactions are first entered in a systematic manner adopting the principles of debit and credit. Journal is subdivided into several books of original entry, namely purchases, sales, cash, bills receivable, bills payable, returns inwards, returns outwards books. They are also regarded as subsidiary books. When once the transactions are recorded in the journal or other subsidiary books, posting is made to ledger. In this unit we are discussing the meaning of secondary books, various types of secondary books and posting of entries to ledger accounts.
Objectives:

After going through this unit, you should be able to: 1. List various primary books containing original entries. 2. Explain the meaning of trade and cash discount and the distinction between them. 3. Describe purchase book, purchase return book, sales book, sales return book and the posting of transactions to ledger accounts. 4. Recall the format of Bills receivable and Bills Payable books

5. Know the method of preparing single column, double column and triple column Cash Book. 6. Know the method of preparing Petty Cash Book. 4.2 Secondary Books Journal is a book of original entry and only one journal is maintained if the business is very small in size and the transactions are limited. However, if the transactions are multifarious, then subsidiary books which are known as books of original entry are prepared. The types of subsidiary books include: 1. Purchases book 2. Sales book 3. Purchase returns book 4. Sales returns book 5. Bills receivable book 6. Bills payable book 7. Cash book and 8. Journal Proper. The entries are made in these books straight without recording in usual journal. From the respective books, posting is made to ledger. In fact, from the entries made in the subsidiary books, journalizing can be done. A detailed note is given in the following paragraphs on each of the subsidiary books. 4.3 Purchases Book Purchases book is also called purchases journal. Only credit purchases of goods are recorded in this journal. Goods mean items or commodities procured for resale. Cash purchases are recorded in cash book and credit purchases are recorded in purchases book. The form of a purchases book is given below.

Trade Discount

It is a reduction granted by a supplier from the list price of goods or services on business consideration (such as quantity bought, trade practices etc). For prompt payment cash discount is allowed. Example: If 5 gold coins are sold at the list price of Rs.15000 each subject to trade discount of 12%. The trade discount will be calculated as under:

Cash Discount It is the reduction granted by the supplier from the invoice price in consideration of immediate payment or payment within a stipulated period. Example: If 5 gold coins are sold at the list price of Rs.15000 each subject to trade discount of 12%. The invoice price after trade discount is Rs.66,000. Cash discount terms are 2%, 30 days. This denotes the buyer will get 2% cash discount if he makes payment within 30 days. The cash discount is calculated as follows:

Difference between Trade Discount and Cash Discount: 1. Trade discount is a reduction granted by a supplier from the list price on goods or services on business considerations such as quantity bought, trade practices etc while cash discount is a reduction granted from the invoice price in consideration of immediate payment or payment within a stipulated period. 2. Trade discount is allowed to promote the sales while cash discount is allowed to encourage early or prompt payment 3. Trade discount is shown by the way of deduction in the invoice itself. Hence no further entry is required in the books of accounts. Cash discount is shown as an expense in profit and loss account. 4. Trade discount may vary with the quantity purchased while cash discount varies with the period. Illustration 1: From the following transactions, prepare the purchase book of Adithya Bros and post the transactions recorded in the Purchase book to the Ledger:

Solution: Purchase Book

Date

Purchase Name of the Invoice No. supplier Goyal Bros 5.3.20X1 442 Less Trade Discount 10% Adikari Mills 8.3.20X1 450 Less: Trade Discount 10%

L.F.

Details 5500

Amount

4950 550 _____ 25000 22,500 2500 ______

Cash purchases from Greg Mac will be recorded in cash book.

Observe that in every case of credit purchase, the suppliers account is credited and goods account is debited. At the end of the day or week or month, the total of purchases is transferred to one ledger account known as Purchases account in the ledger. Self Assessment Questions:

1. All purchases cash or credit is entered into purchases day book. (State True/False) 2. Purchases of goods and other assets can also be recorded in purchase book. (State True/False) 3. Inwards Invoice is a document to verify the quantity, price and other details of goods purchased. (State True/False) 4. Purchases made from Mr. Ganesh on credit for Rs 6000, entered in the purchases book. What is the journal entry? 4.4 Sales book Sales book or sales day book contains the details of credit sales of goods made during a particular period. The total of the sales book is transferred to ledger to an account called sales account. The parties to whom credit sales are made are known as trade debtors. All debtors are classified as personal accounts and for each party; ledger account is prepared in the ledger. Sales account shows credit balance and debtors account shows debit balance. A pro forma of sales book is as given under. Illustration 2: From the following transactions, prepare the Sales Book of Adithya Bros and post the transactions recorded in the Sales book to the Ledger:

Solution: Sales Book

Date

Sales Invoice Name of the supplier L.F. No. Goyal Bros 5.3.20X1 442 Less Trade Disc 10%

Details Amount 5500 4950 550 _____ 25000 22,500 2500 ______

Adikari Mills 8.3.20X1 450 Less: Trade Disc10%

Cash Sales from Greg Mac will be recorded in cash book.

Outward/Sales Invoice number is the number of the invoice issued by the businessman to its customer. Similarly the respective ledger accounts of the customers will be prepared in the ledger.
Self Assessment Questions:

5. Sales day book contains only credit sales of goods made. (State True/False) 6. Sale of any other asset other than goods is also recorded in sales day book. (State True/False) 7. Persons to whom sales are made on credit are called ________. 8. Outward invoice is a document issued to customer, when the goods are sold on credit. (State True/False) 4.5 Purchase Returns Book When the business person purchases the goods and finds they are damaged or not as per the specifications he /she decide to return the goods to the supplier from whom the goods were purchased. All such purchase returns are recorded in a journal called purchase returns book. Normally the suppliers account is credited when the purchases are made. If the goods are returned, then a debit note will be sent and the number of debit note is recorded in the purchase returns book. A debit note is a document prepared by the purchaser to inform the supplier that his account has been debited with the amount mentioned and for the reason stated therein.

The total of the items is transferred to ledger to an account called purchase returns account, which shows credit balance. The respective personal accounts of the suppliers/creditors are debited in their respective ledger accounts.

Self Assessment Questions:

9. Purchase returns are also called returns outwards. (State True/False) 10. Purchase returns take place when the goods bought are not as for the specification. (State True/False) 11. When goods bought are returned the suppliers account is _________ and purchase return account is _____________. 12. Debit note is a document to show the supplies account is being debited. (State True/False) 4.6 Sales Returns Book Sales returns book (also known as Return Inward book) is opened for the purpose of recording the return of goods sold on credit. Then a credit note is prepared to show that the customers/debtors account is credited to the extent of the value of the goods returned by them to us. Goods are received from the customers and a credit note is sent to them. A credit note is a document prepared by the seller to inform the buyer that his account has been credited with the amount mentioned for the reasons stated therein. Credit notes are issued to the customers while debit notes are issued by the customers.

Format of Sales Returns Books

Date

Credit Note No. Name of the Supplier L.F

Details

Amount

The total of the book is transferred to an account called sales returns account in the ledger and this account shows debit balance. The respective personal accounts of the customers are credited with the value of the goods returned by them.
Self Assessment Questions:

13. Sales return are also called returns inwards. (State True/False) 14. Credit note is a document to indicate that the goods are received as returned by customers. (State True/False) 15. Credit noted is sent by _____________ to __________. 4.7 Bills Receivable Book

When a business person sells goods on credit, the proceeds is received at a later date. Suppose the business person requires cash immediately, he may opt to draw a bill of exchange against the customer. The duly filled bill of exchange is sent to the customer for acceptance. The customer accepts by the way of putting his signature on the bill of exchange and returns it back to the business person. On the receipt of the signed bill, the business person discounts it with his banker for a commission. The business person receives 90 percent of the bill amount on discounting. Bill of exchange is a document in writing, promising to pay a certain sum of money or moneys worth to the drawer at a certain date for value received. The businessman who draws the bill of exchange is called the drawer and the customer on whom it is drawn is called a drawee or acceptor. The businessman maintains a journal/ subsidiary book containing the details of the bills receivable.

Bills Receivable Book of Sham Sundar & Co.,

Date No. From Term Date of of Where Due of the Whom Acceptor of the Receipt the payable Date bill received bill bill 1 04-7-04 04-7Mr.X 04 01-8Mr. Y 04 09-9Mr. A 04 10-9Mr. B 04 Mr. X Delhi 3 mths 7-1004 4-1204 12-1204 13-105

LF

Amount Remarks Rs.

7,000

1-8-04

Mr. Y

Noida 4 mths

9,000

9-9-04

Mr. A

Agra

3 mths

12,000

10-9-04

Mr. B

Delhi

4 mnth

10,000 38,000

For every bill the due date is calculated after adding three days of grace. The total of the bill receivable is transferred to bills receivable account in the ledger. The bills receivable account shows debit balance and the amount receivable against them is an asset.
Self Assessment Questions:

16. A bill is an instrument in writing similar to that of a promissory note. (State true or false)

17. Who is a drawer of a bill of exchange in a business? 18. Who is the acceptor of a bill of exchange in a business? 19. Bills Receivable account shows _____________balance. 20. Can bills receivable be discounted? 4.8 Bills Payable Book What is bills receivable for a drawer, is bills payable to the drawee. When a business person purchases goods on credit he need not pay for it immediately. Instead he may accept the bills of exchange drawn by the supplier. All such bills accepted by the business person are recorded in a separate book called bills payable book. The sum of the value of bills payable for a period ending will be transferred to the ledger. Usually bills payable account shows credit balance and hence is a liability. The form of bills payable book is given here under.

Self Assessment Questions: 21. Bills of exchange drawn by the supplier of goods and accepted by the proprietor of the business are called _________. 22. Every bill has ______days of grace . 23. Bill payable account shows _________ balance. 24. Bill payable represents a ________. 25. When bills payable account is credited ________ account is debited. 4.9 Cash Book Cash book is an important subsidiary book and a book of original entry. It is a record of cash receipts and cash payments made during a particular period. On the right hand side, receipts are recorded and on the left hand side, payments are recorded. A simple cash book has two sides, receipts side and payment side. The receipts are on debit side and the payments are on credit side. Just as a ledger account, the words To and By are used. Cash book may also contain

cash column and bank column. Cash column represents cash in the business and bank column represents cash kept in the bank. Bank column of cash book is a reflection of bank pass book. In this connection, it is important to note that in a few transactions, affecting both cash and bank accounts, contra entries are drawn. Cash book containing cash and bank columns is known as two column cash book. In the case of three column cash book, on the receipt side, cash, bank and discount allowed columns are stated. On the credit side, cash, bank and discount received columns are mentioned.

Note the following points from the above illustration: a) Discount column on the debit side represents discount allowed and on the credit side, it represents discount received. Balancing is not done for these columns for a simple reason to find out separately the discount allowed and received. b) There are two contra entries each on 15th and 30th. On 15th the transaction is cash withdrawn from bank Rs. 3,000. It is a payment from bank and it is receipt to business cash. Similarly on

30th Cash is deposited to bank Rs.1000. It is a receipt to the bank account and payment from cash account. c) To indicate contra entry, C is mentioned against the entry. d) Drawings represent the amount withdrawn from bank for business purposes. e) Dividend from X Co is received by cheque and the company should have remitted the dividend directly to the bank account of the businessman. f) The balance c/d is the closing balance for the month of January 2002 and this becomes opening balance for February, 2002.
Self Assessment Questions:

State True or False 26. Cash book and cash account are one and the same. (State true/false) 27. Trade discounts allowed to customers or received from suppliers are not recorded in cash book. 28. Cash discount allowed to customers appears on the ____ side of cash book. Cash discount received appears on the ____ side of cash book. 29. Discount columns are independently totaled and not balanced. (State true/false) 30. Contra entry is an entry where both cash account and bank account are affected. (T/F) 4.10 Petty Cash Book In large organizations, petty expenses like stationery, postage, stamps, refreshments, carriage, cartage, daily wages etc are incurred day in and day out. All these expenses are more in number and very insignificant in value. To look after payment of such expenses, a separate petty cashier is appointed who obtains a definite sum of money at the beginning of a month and gives a statement of account at the end of the period to the chief cashier. To record such payments, a separate book, known as petty cash book is maintained. There is a distinct method, namely Imprest system which is adopted in maintaining such petty cash book. Under this system, at the beginning of a month, a definite sum of money is given by chief cashier to petty cashier for petty expenses. At the commencement of the next period, the petty cashier is reimbursed equal to what he had spent during the earlier period. Example: 1st Jan 2004: Amount received by the Petty cashier Rs. 10000

Expenses incurred during the month of Jan 2004 Rs. 9000 31st Jan 2004: Cash balance Rs. 1000 Amount reimbursed by the Chief cashier Rs. 9000 1st Feb 2004: Opening petty cash balance Rs. 10000 Self Assessment Questions: 31. Petty cash book is maintained in case of petty organization. 32. Imprest system of cash book is a system where the expenses incurred are reimbursed. 33. The closing balance in imprest system of petty cash book always remains the same. 34. Imprest system of cash book is also called analytical cash book. Illustration 3: Enter the following transactions in an analytical petty cash book. Also open relevant ledger accounts. 2005 Nov 1st. Received a cheque for petty cash Rs.1000 2nd. Paid bus fare to messengers Rs50 4th. Paid auto fare Rs.70 10th. Postal stamps purchased Rs.80 12th. Paid for stationery Rs90 15th. Paid for carriage Rs.60 16th. Purchased envelopes Rs.50 20th. Wages paid Rs 100 . 25th. Tips given to driver Rs.50 30th. Telephone calls paid Rs. 20

Note: 1. CBF stands for cash book folio 2. V.No. stands for Voucher No. 3. Tra stands for Travelling expenses 4. Carr indicates Carriage expenses 5. P & S stands for printing and stationery

4.11 Posting to Ledger The next important stage of accounting is preparation of ledger accounts in a book called ledger. The book contains the summary of transactions concerning to various heads of accounts for a given period. Posting is made to ledger accounts from journal entries and various subsidiary books and at the end of the accounting period, each ledger account is balanced. Procedure for balancing a Ledger account: Total both the debit column and credit column If the debit total exceeds credit total, put such difference (called debit balance) on the credit side as By balance c/d. If the credit total exceeds debit total put such difference ( called credit balance) on the debit side as To balance c/d Total both the debit side and the credit side of the account Enter the date of the next accounting period and bring down the balance of the previous accounting period. If it is debit balance write To balance b/d on the debit side of the account and if it is credit balance write By balance b/d on the credit side of the account.

Observe the following from the above account: 1. The balance brought down is the closing balance of the last month, December, 2004 2. The amount received from Joseph is Rs.10900 and his account is prepared in the in the page number 35 of the ledger. 3. The credit side contains payment of cash towards salary, furniture, purchase of goods and rent respectively on different dates.

4. The balance carried down is the closing balance on the last day of January, 2005 and it is brought down as opening balance on Feb,1 5. On the debit side, To and credit side By are the prefix used for every entry as a matter of convention. There is a standard form of drawing a ledger account. It is similar to that of a pass book issued by a bank. The above illustration is shown in the standard form.

Self Assessment Questions: 35. Rules of debit and credit are different for different types of accounts. True or False? 36. The words To Balance b/d or By Balance b/d denote an opening entry. State true or false. 37. _________ And __________ are balanced while ______ are closed by transfer to trading and profit and loss account. 38. In the ledger account during the beginning of the accounting period To Balance b/d denotes debit balance and By balance b/d denotes credit balance. State true or false. Activity 1 Given in the table are various types of accounts. Mention whether it carries debit or credit balance.
Debit / credit bal anc e

Name of the account

Capital Personal Drawings Creditors

Bills Payable Bank overdraft Loans from others Outstanding expenses Pre received incomes Reserves for future expenses or losses All items of incomes Cash in hand or at bank Assets such as furniture, buildings, plant, machinery, tools, stock of goods, etc Debtors, Bills receivable Loans given to others Investments made All expenses such as wages, carriage, insurance, salaries, printing and stationery, advertising, commission paid, interest paid, etc Prepaid insurance, rent or any prepaid expenses Outstanding incomes Losses like depreciation, loss in the revaluation of assets or sale of assets, Any other asset 4.12 Summary

Secondary books are also known as books of original entry. They include purchase book, sales book, purchase return book, sales return book, bills receivable book, bills payable book, cash book and journal proper. Trade discount is a reduction granted by a supplier from the list price of goods or services on business consideration. Cash discount is the reduction granted by the supplier from the invoice price in consideration of immediate payment or payment within a stipulated period. Bill of exchange is a document in writing promising to pay a certain sum of money or moneys worth to the drawer at a certain date for value received. Cash book is an important subsidiary book and a book of original entry. It records cash receipts and cash payments made during a particular period.
4.13 Terminal Question

1. Purchases book records___________________ purchases. 2. Cash purchases are recorded on_____________ side of cash book. 3. Credit sales are entered in __________book. 4. Record a journal entry for drawings made for personal purposes of the business person. 5. If drawings are made from bank for office purpose, what is the entry? 6. During the year, if the total owners equity of Beta Co. increased from Rs. 50,000 to Rs. 60000. This is due to earnings made during the year. Is this statement necessarily true? 7. Enter the following transactions in the single column cash book of Gopichand. March, 2003 1st .Commenced business with cash 20000 2nd Bought goods for cash 5000 3rd. Sold goods for cash 4000 4th. Goods purchased from Ravi Kumar 10000 10th .Paid to Ravi Kumar 7000 14th. Cash sales 8000 18th. Purchased furniture for office 4000

22nd. Paid wages 500 25th. Paid rent 600 30th. Received Commission 4000 30th. Withdrew for personal purpose 1000 31st. Paid salary 900 8. Record the following transactions in two column cash book (Cash and Bank)in the books of Soft Silk Co., for the month of July, 2004.Find out the closing balances for the month of July 2004. July, 2004 Rs. 01st. Opening balance b/d(Cash) 14,500 Opening balance b/d (Bank) 7,000 04th. Cash purchases 6,700 05th. Rent for June month paid by cheque 2,500 09th. Cash sales 15,200 12th. Dividend paid by cheque 4,350 15th. Cash deposited into bank 5,000 18th. Cash paid to Rahim Bros to settle his account 10,000 20th. Repairs paid 1,000 22nd. Commission paid by cheque 2,000 23rd. Customer, Deepak remitted to our bank account 20,000 25th. Cash withdrawn from bank for office use 5,000 27th. Drawings made from business cash for personal purposes 2,000 28th. Purchased stationery by cash 3,000 30th. Cash withdrawn for personal use from bank 1,400

9. Enter the following transactions in the cash book with discount, cash and bank columns. Prepare three columnar cash book for the month of May.

May 1st

Balance of cash in hand Rs. 14000; bank overdraft at bank Rs.5000 Invested further capital Rs. 10000 out of which Rs.6000 was deposited in the bank. Sold goods for cash Rs. 30000 Collected from debtors of last year Rs. 80000; Discount allowed to them Rs. 2000. Purchased goods for cash Rs. 55,000 Paid Ram Vilas, our creditor Rs. 25,000; discount allowed by him Rs.650 Commission paid to our agent Rs. 5,300 Office furniture purchased for cash Rs. 2,000 Rent paid Rs. 400; electricity charges paid Rs. 1,000 Drew cheque for personal use Rs. 7,000 Cash sales Rs. 25,000 Collection from Atal Bihari Rs.40,000, deposited in the bank on 19th April Drew from the bank for office use Rs.5,000 Drew cheque for petty expenses Rs.1,500 Dividend received by cheque Rs.500, deposited in the bank on the same day Commission received by cheque Rs.2,300, deposited in the bank on 28th April

4th

6th 6th

10th 11th

13th 14th 14th 14th 17th 18th

19th 22nd 24th

25th

29th 30th

Drew from the bank for salary of the office staff Rs.15,000 Deposited cash in the bank Rs.10,000.

10. Prepare petty cash book on imprest system from the following particulars. i. Jan 1st Received for petty cash payment Rs. 500/ii. Jan 2nd Paid for postage Rs. 40/iii. Jan 5th Paid for stationery Rs. 25/iv. Jan 8th Paid for advertisement Rs. 150/v. Jan 12th Paid for wages Rs. 50/vi. Jan 16th Paid for carriage Rs. 25/vii. Jan 20th Paid for conveyance Rs. 22/viii. Jan 25th Paid for traveling expenses Rs. 80/ix. Jan 27th Paid for postage Rs. 50/x. Jan 28th Paid wages to cleaner Rs. 10/xi. Jan 30th paid for telegram Rs. 20/xii. Jan 30th Sent registered notice Rs. 10/4.14 Answer for Self Assessment Questions

Answer for Terminal Questions:

1. Credit 2. Credit 3. Sales Day 4. Drawings A/c Dr. To Cash a/c 5. Cash account Dr. To Bank account. 6. The statement is true if additional capital is not brought in during the year. Owners equity increases if profits are added or additional capital is brought in. 7. Cash Book of Gopichand

1st March To Capital 3rd 14th 30th To Sales To Sales

20,000 4,000 8,000

2nd 10th 18th 22nd 25th 30th 31st 31st

By Goods By Ravi Kumar

5000 7000

By office furniture4000 By wages By rent By drawings By salary By bal c/d 500 600 1000 900 17,000 36,000

To Commission 4,000

36,000 Hint: Goods Purchased from Ravi Kumar is a credit purchase. 8. CASH BOOK

July04 1st

Cash Bank July04 To Op bal b/d 14,500 7000 4th By Purchases By Rent By dividend paid

Cash Bank 6700

9th 15th

To Sales To Cash (c)

15,200 5000

5th 12th

2500 4350

23rd 25th

To Deepak To Bank ( c ) 5000

20,000 15th 18th

By bank (c) 5000 By Rahim & 10,000 Bros By repairs By comm. paid By cash ( c ) By drawings 2000 By stationery By drawings By balance 7000 c/d 3000 1000 2000

20th 22nd

25th 27th 28th

5000

30th 30th

1400 16750

34700 32,000 To Bal b/d 7,000 16,750 9. Cash Book Disc Rs Cash Rs Bank Rs

34700 32,000

Disc Cash Rs Rs

Bank Rs

To bal b/d To Capital To Sales

14,000 4000 30,000 6000

By balance b/d By purchases By Ram vilas By commission By office furniture By rent 55,000 650 25,000 5,300 2000 400

5000

To Debtors 2000 80,000 To Sales To Atal Bihari To Cash (c) To Bank (c) To Dividend To Comm To Cash (c) To Cash (c) 2300 2300 5,000 500 25,000 40,000

40,000 By electricity By drawings By banks (c) By cash (c) By petty expenses

1000 7000 40,000 5,000 1,500 2,300 15,000 10,000 59,300 25,300

10,000 By bank (c) By salary By bank ( c) By balance c/d

Total

2000 2,00,300 58800 Total

650 2,00,300 58,800

10. Petty cash book

Cash Date Postage Total Carri- Printing Travel recd 1996 Particulars LF & Advt Wages Sundry payment age & stry exp. Rs Jan telegram

500 1st 2nd 5th

To cash By postage By stationery By advt 40 25 40 25

8th

150 50 25 22 25

150 50

12th By wages 16th By carriage 20th By conveyance 25th By traveling 27th By postage 28th By wages 30th By telegram 30th By register

22

Cr 80

80

50 10 20

50 10 20

10 482

10 120 25 25 150 102 60

30th By balance b/d 500 18 July To balance 1st b/d

18

500

482 July To cash 1st

Activity 1 Solution
Debit / credit balan ce

Name of the account

Capital Personal Drawings Creditors Bills Payable Bank overdraft Loans from others Outstanding expenses Pre received incomes Reserves for future expenses or losses All items of incomes Cash in hand or at bank Assets such as furniture, buildings, plant, machinery, tools, stock of goods, etc Debtors, Bills receivable Loans given to others Investments made

Credit Debit Credit Credit Credit Credit Credit Credit Credit Credit Debit Debit

Debit Debit Debit

All expenses such as wages, carriage, insurance, salaries, Debit printing and stationery, advertising, commission paid, interest paid, etc

Prepaid insurance, rent or any prepaid expenses Outstanding incomes

Debit Debit

Losses like depreciation, loss in the revaluation of assets Debit or sale of assets, Any other asset Copyright 2009 SMU Powered by Sikkim Manipal University
.

Debit

MB0041-Unit-05-Trial Balance
Unit-05-Trial Balance Structure: 5.1 Introduction Objectives 5.2 Meaning 5.3 Objectives of preparing a trial balance 5.4 Methods of preparing a trial balance 5.5 Preparation of Trial balance 5.6 Adjusting Entries 5.7 Errors and their rectification 5.8 Errors disclosed by Trial Balance 5.9 Errors not disclosed by Trial Balance 5.10 Steps to locate the errors

5.11 Summary 5.12 Terminal Questions 5.13 Answers 5.1 Introduction In the previous unit we learnt how about various secondary books that include purchase book, sales book, purchase return book, sales return book, bills receivable book, bills payable book, cash book and journal proper. Cash book is an important subsidiary book that records all cash receipts and payments during a particular period. Posting of entries in ledger and the procedure for balancing in ledger account was briefly dealt. In this unit we have dealt with the meaning, the need of trial balance, and the methods of preparing trial balance. There are different types of errors. Students should be acquainted with types of errors and how they are rectified. There are certain errors that are disclosed in the trial balance while certain errors are not disclosed in the trial balance. Trial Balance is a statement of debit balances and credit balances that are extracted from ledger accounts on a particular date. It stands as a bridge between primary and secondary books on one hand and final statements of accounts on the other hand.

Objectives: After going through this unit, you should be able to: 1. Explain the meaning and recall the format of trial balance. 2. Explain the objectives of preparing a trial balance 3. List the guidelines to prepare a trial balance. 4. Identify and rectify the errors that are disclosed by trial balance 5. Identify and rectify the errors that are not be disclosed by trial balance 6. Know the steps to locate the errors. 7. Prepare trial balance after incorporating adjustments. 5.2 Meaning Trial Balance is a statement containing the various ledger balances on a particular date. It is prepared to check the arithmetical accuracy of the posting of transactions to the ledger. It is a list

of debit and credit totals or a list of debit and credit balances of all the ledger accounts prepared on any particular date to verify whether the entries in books of accounts are authentically correct. As the primary and secondary books are maintained on the double entry concept, the balances in the trial balance must tally. A trial balance is not a part of books of account. It is drawn as a separate statement, and this becomes the source document for preparing external financial statement such as profit and loss account, cash flow statement and Balance Sheet. 5.3 Objectives of Preparing a Trial Balance: There are three objectives of preparing a trial balance. a) To check the arithmetic accuracy of entries made. In double entry, every debit has an equivalent credit. Even in General Journal, we have seen that the total of debits equals the total of credits. Similarly, if the debits and credits tally in a trial balance, it indicates that the books of account are arithmetically accurate. b) Basis for financial statements. As stated earlier, trial balance is a bridge between ledger and final statements. Trial balance facilitates in the preparation trading account, profit and loss account and balance sheet. c) It is a summarized ledger. The position of a ledger account is judged simply by looking at the trial balance. It is because, all ledger accounts, after being balanced, are grouped as those showing debit and those showing credit balances. The total of all debit balance is equal to total of all credit balance. If in any case the trial balance does not tally, the difference is temporarily transferred to suspense account till such difference is rectified. 5.4 Methods of preparing a Trial Balance Totals method and Balance method are the two techniques of preparing trial balance. In the first method, the totals of debits and credits of every account are shown in the trial balance. For instance, a cash account has a debit total of Rs.45000 and credit total of Rs.35000. Creditors has a debit total of Rs.70000 and credit total of Rs.80000. Both these totals are carried to trial balance. Performa of Trial Balance under Totals Method

The same logic is applied for all other accounts. Balance Method:

In the Balance method, instead of transferring the totals of both debit and credit, the net balance of every account for example in case of cash account of Rs.10000 (45000 35000) is shown on the debit side of trial balance.

If the debit side is greater than the credit side the difference is termed as debit balance. Creditors have Rs.70000 as debit total and Rs.80000 as credit total. The closing balance is shown on the credit side.

If the credit side is greater than the debit side the difference is termed as credit balance. Proforma of Trial Balance under Balance Method TRIAL BALANCE as on _________ Debit balance Particulars Rs. Cash Creditors Total 10,000 10,000 10,000 10,000 Rs. Credit Balance

In the second method only the difference (debit balance/credit balance) is alone considered. The same principle is adopted for all the other accounts. In the first method, more details are revealed but it is cumbersome. The second method gives the gist of the account and hence the second method is popular. 5.5 Preparation of Trial Balance The following steps should be followed to prepare a Trial Balance.

a) Prepare the ledger accounts b) Balance them at the end of accounting period c) Group all accounts showing debit balance and show them of left hand side of trial balance d) Group all those accounts showing credit balance and show them on the right hand side of trial balance. e) Total the debits and credits and they must be equal, what ever be the method of preparing the trial balance.

Illustration 1: The following are the ledger accounts of Mr. X as on 31st December, 1998. Prepare a trial balance.

Solution

5.6 Adjusting Entries Financial Accounting uses accrual basis of accounting which is supported by GAAP. Under accrual basis of accounting revenues are recognized when earned without regard to the timings of cash receipts. Expenses are recognized either in the period in which related revenues are recognized or when incurred without regard to the timings of cash disbursement.

When such system is used adjusting entries are required to be made at the end of the period to record any previously unrecognized changes in assets, liabilities, revenues or expenses. Adjusting entries are purely internal transactions. There are various types of adjusting entries. Opening Journal entries: In the case of running business, all the assets and liabilities of the previous year should be brought down to the current year and therefore an entry is drawn debiting all assets account individually and crediting liabilities account individually and the difference being credited to capital account. Illustration 2: On 31st Dec, 2004, the following were the assets and liabilities of a firm: (1) Cash at bank Rs. 50000; (2) Furniture Rs. 48000; (3) Plant and machinery Rs.200000; (4) Debtors Rs.100000; (5) Stock in trade Rs. 20000; (6) Creditors Rs. 50000; (7) Bank loan Rs. 45000. On 1st of January, 2005 the assets and liabilities have to be brought in. The following entry is recorded in Journal Proper.

Similarly, a newly set up business may commence its activities with some assets and liabilities. Then the assets are debited and liabilities are credited and the difference is transferred to capital account. Self Assessment Questions: 1. Opening journal entries are drawn at the commencement of accounting period. (State whether it is True / False).

2. When all assets are debited and all liabilities are credited, the difference is transferred to ___________ account. 3. If opening liabilities including capital are more than assets, to what account the difference is transferred? 4. Pass the opening entry in the journal of Bharat as on 1st April 20X1. Cash in hand Rs.1,000; Cash at bank Rs.5,000; Stock Rs.20,000; Land and Building Rs.1,00,000; Plant and Machinery Rs.50,000; Furniture and Fixtures Rs.25,000; Owing from X Ltd Rs.12,500; Prepaid Insurance Rs.500; Interest received in advance Rs.250; Loan from Y Ltd Rs.10,000; Owing to Z Ltd Rs.3,750. Closing entries Closing entries are drawn at the end of accounting period and the purpose is to close down several account balances for the current period. The accounts of assets and liabilities will not be closed because they continue to exist further. All expenses and income accounts are closed by transferring them to the respective revenue accounts such as Trading account and Profit and Loss account. Example:

Self Assessment Questions: 5. All revenue accounts are closed at the end accounting period. (State whether it is True / False). 6. All trade expenses are closed by debiting _________account 7. All expenses other than _________ are closed by transferring them to P & L account. 8. Are assets and liabilities accounts closed at the end of the accounting year? (State whether it is Yes / No). After the closure of accounting year, there might be a few more transactions left over and which are not incorporated into journal or ledger, owing to omission and practical difficulties. Example:

Similarly, with regard to rent paid in advance, expenses outstanding, incomes received in advance etc adjusting entries are made in Journal proper. If they are not considered, the profit or loss reflected by the final accounts will not give the correct picture for the accounting period.

Self Assessment Questions: 9. Transaction which are out of trial balance have to be adjusted for proper calculation of profit / loss (state whether it is True / False). 10. What is the adjusting entry in the following cases? a. Depreciation of Building b. Closing stock c. Pre-paid Insurance d. Outstanding salaries e. Stock used for personal purposes Transferring entries When the balance of one account is transferred to another account, transferring entry is made. For instance, drawings made by proprietor should be reduced from his capital account. To facilitate this, drawings account, which shows debit balance, is credited and capital account is debited (because capital is reduced as a result of drawings). This is a transferring entry and it is recorded in Journal proper. Compound Entry: A compound journal entry is passed when more than two accounts are involved in a transaction and the transaction is recorded by means of a single entry instead of passing several journal entries. Example 1: Paid Rs 980 to Bharat in full settlement of his account of Rs.1000 Journal Entry: Bharat a/c Dr.1000 To Discount Received a/c 20 To Cash a/c 980 (Being cash paid to Bharat in full settlement of his account) Example 2: Received Rs.490 from Rajan in full settlement of his account of Rs.500.

Journal Entry: Cash a/c Dr. 490 Discount allowed a/c Dr. 10 To Rajan a/c 500 (Being cash received from Rajan in full settlement of his account) 5.7 Errors and their Rectification An error is an unintentionally committed mistake. When the Trial Balance does not tally it is a clear indication that there are some errors in the preparation of accounts. The errors may be committed at various stages Journalizing, Posting, Casting (totaling), Balancing, Transferring to trial balance and so on. Mere tallying of the trial balance does not ensure an error free statement. There are certain errors such as errors of omission, error of principle and compensating errors are not disclosed by trial balance while errors of casting, posting to wrong side of an account or posting a wrong amount can be detected by trial balance. Errors whether disclosed or not disclosed by trial balance, have to be corrected or rectified in order to obtain the correct picture of profit or loss. It should be remembered that errors will have their impact not only on profit but also on the asset and liability position of the business organization. Self Assessment Questions: 11. Errors can be committed at all stages, beginning from journal entries, posting of entries in ledger accounts, while balancing the closing balances etc. (True / False). 12. Errors of omission, error of principle and compensating errors are not disclosed by trial balance (True / False). 13. Errors of casting, posting to wrong side of an account or posting a wrong amount etc can be detected by trial balance (True / False).

5.8 Errors disclosed by Trial Balance Those errors that can be disclosed by trial balance can easily be located. As soon as the trial balance does not tally, the accountant can proceed to find out the spots where the errors might have been committed. The total amount of difference in the trial balance is temporarily transferred to a Suspense Account so that it can be mitigated as and when the errors get rectified. Therefore the suspense account gets debited or credited as the case may be on rectification of these types of errors. The following are the errors which are disclosed by trial balance: a) Posting a wrong amount: This mistake may occur while posting an entry from subsidiary book to ledger.

b) Posting to the wrong side of an account: This error is committed while posting entries from subsidiary books to ledger.

c) Wrong Totaling: Both under casting and over casting are detected by trial balance. If any account is wrongly totaled, it gets reflected in the trial balance.

d) Omitting to post an entry from subsidiary book to ledger: If an entry made in the subsidiary book does not get posted to ledger, the trial balance does not tally.

e) Omission of an account altogether from being shown in trial balance:

f) Posting an amount to a correct account more than once: This result in imbalance in the trial balance.

g) Posting an item to the same side of two different ledger accounts: If two accounts are debited /credited for the same transaction, this type of error occurs.

Self Assessment Questions: 14. Suspense account is the difference between debit total and credit total of a trial balance. (True / false). 15. Suspense account is created temporarily and later, it is removed as and when errors are detected and suitable rectified (True / False). 16. If amount paid to Rama Rs 500 is credited to Ramanan accounts, what type of error has occurred and give the rectification entry. 17. Instead of putting Rs. 1500 to debit of wages account, Rs 15000 is recorded. Identify the type of error and tell what impact it has on profit? 18. Refer Q.No.17. How do you rectify the above error?

5.9 Errors not disclosed by Trial Balance There are four errors regarded as those which do not affect trial balance and it is difficult to locate them. A brief description of the four errors is offered in the following paragraphs: a) Error of omission: Error of omission occurs when a transaction is completely omitted from the books of accounts.

Since both aspects debit and credit of the transaction are missing, the trial balance is not affected at all. To rectify such errors, the transaction should be recorded when it is traced. b) Error of commission: If the error of wrong posting, wrong casting, wrong calculation etc., committed in the books of original entry or ledger, it is said to be error commission.

c) Error of principle: While drawing journal entries, often error of principle is committed and this goes unnoticed because it does not affect the total of trial balance.

Similarly, treating incomes as liabilities, providing insufficient provision for bad and doubtful debts, inadequate depreciation against assets etc., come under errors of principle. They must be rectified by applying the correct principles of accounting.

d) Compensating errors: It is also called off-setting error. Compensating error is one which is counter balanced by another error.

Self Assessment Questions: 19. If error of wrong posting, wrong casting, wrong calculation are committed in the books of original entry or secondary books, such errors are called ________. 20. Error of commission affects trial balance (True / False). 21. Furniture purchased for cash Rs 5000/- is not recorded in journal. Mention the type of error? 22. Error of omission can be detected only after a careful review of ledger balances of previous years (True / False). 23. Error of principle affects the value of revenue and capital items (True / False). 24. It is very difficult to find out the compensating errors. (True / False). 5.10 Steps to locate the Errors The following steps help to locate the errors. In spite of the efforts, if the difference in the trial balance persists, a suspense account may be created and subsequently the suspense account can be eliminated as and when the errors are located and rectification is made.

When errors are located, they should be rectified. It is not a good practice nor do we have the legal right to erase the mistakes and re write the correct ones. Rectification entries are recorded in General journal or journal proper. Self Assessment Questions: 25. Summary of all ledger balances is called _____________________ . 26. Trial balance is necessary to prepare _________________________ . 27. The broad two categories of errors are a) ________________ b) ____________. 28. Is casting error, an error of principle or error of commission? 29. Purchase of machinery is included in the purchases book. What type of error is it? 30. What is error of omission? Illustrate. 31. What are the errors that cannot be disclosed by trial balance? 32. The sum of errors in accounting is transferred temporarily to _________ account. 33. In which journal do you make rectification entries? 34. State any four steps to locate errors. 35. If sales account is under cast by Rs. 45, what is the rectification entry? 36. Returns inwards book is over cast by Rs. 9, write the rectification entry. 37. Salary paid to Gopal is debited to his personal account. What is the rectification entry to correct the error? 38. Discount received Rs. 50 is transferred to the debit side of discount account. Write the rectification entry. 39. An invoice of purchase for Rs. 760 is entered as Rs. 670. What type of error is this? How to rectify this error? Illustration 1 An accountant finds that the trial balance of his client did not tally and it showed an excess credit of Rs. 69.74. He transferred it to a suspense account and later discovered the following errors. a) Rs. 44.37 paid to Anand has been credited to his account as Rs. 34.37.

b) A purchase of Rs. 145.50 has been posted as Rs. 154.50 to the purchases account c) An expenditure of Rs. 158 on repairs has been debited to the Buildings account d) Rs. 80 was allowed by B as discount which has not been entered in the books. e) A sum of Rs.125.05 realized on the sale of old furniture has been posted to the sales account. Give journal entries to rectify the errors and show the suspense account as it would appear after adjustments.

Solution

Date 1

Particulars Anands account Dr To suspense account (Being wrong amount, wrongly credited to Anands a/c rectified) Suspense account Dr To Purchases account (Being over debit of purchase a/c rectified) Repairs account Dr To Buildings account (Being wrong debit given to building account rectified) Bs account Dr To Discount received a/c (Being discount received

LF

Debit (Rs.) Credit (Rs.) 78.74 78.74

2 3 4

9.00 158.00 80.00

9.00 158.00 80.00

from B, omitted earlier, brought to account) Sales account Dr 5. To old furniture account (Being sale of old furniture wrongly transferred to sales account rectified) 125.05 125.05

Note: 1. The entry should have been:

When amount is paid to Anand, his account should have been debited. On the other hand, his account was credited for a wrong amount of Rs.34.37. Hence there has been excess credit to the extent of Rs.78.74 (44.37 + 34.37). To rectify this double error we need to debit Anands account to the extent of Rs.78.74 and credit suspense account. 2. Purchases account was over debited by Rs.9 (Rs.154.50 Rs.145.50). To rectify this error we need to credit purchase account to the extent of Rs.9 and debit suspense account. 3. Repairs spent on building are, by mistake debited to building account. This is error of principle. Repairs account is debited and buildings account is credited to rectify the error. 4. Discount received from B has not been taken to records. This is an error of omission. Therefore, it is now brought to accounts. This has not affected the trial balance. 5. When old furniture is sold, the furniture account should have been credited. On the other hand, sales account was credited against to the principle of accounting. To rectify the error, sales account is debited and old furniture account is credited.

Illustration 2 The trial balance of Evergreen Co Ltd., taken out as on 31st December, 2002 did not tally and the difference was carried to suspense account. The following errors were detected subsequently. a) Sales book total for November was under cast by Rs1200. b) Purchase of new equipment costing Rs.9475 has been posted to Purchases A/c. c) Discount received Rs.1250 and discount allowed Rs.850 in September 2002 have been posted to wrong sides of discount account d) A cheque received from Mr Longford for Rs.1500 for goods sold to him on credit earlier, though entered correctly in the cash book has been posted in his account as Rs.1050 e) Stocks worth Rs.255 taken for use of Mr Dayananda, the Managing Director, has been entered in sales day book. f) While carrying forward, the total in Returns Inwards Book has been taken as Rs.674 instead of Rs.647. g) An amount paid to cashier, Mr. Ramachandra, Rs.775 as salary for November month has been debited to his personal account as Rs757. h) Pass journal entries and draw up the suspense account. Solution

Note: In Q. No.(c): Discount received Rs.1250 is posted on the wrong side of discount account. Discount received (income) should be credited and discount allowed (expenses) should be debited. Instead of crediting the discount received account, it has been wrong debited. To rectify

this error, we need to credit discount received account to the extent of Rs.2500 (Rs.1250+ Rs.1250). In the same context, discount allowed which is an expense should be debited instead, it is credited. To rectify the error, we need to debit discount allowed to the extent of Rs. 1700 (850 +850). The difference between discount received and discount allowed account is transferred to suspense account.

5.11 Summary Trial Balance is a list of debit and credit totals or a list of debit and credit balances of all the ledger accounts prepared on any particular date to verify whether the entries in books of accounts are authentically correct. As the primary and secondary books are maintained on the double entry concept, the balances in the trial balance must tally. Totals method and Balance method are the two techniques of preparing trial balance. In the first method, the totals of debits and credits of every account are shown in the trial balance. Financial Accounting uses accrual basis of accounting which is supported by GAAP. Under accrual basis of accounting revenues are recognized when earned without regard to the timings of cash receipts. Expenses are recognized either in the period in which related revenues are recognized or when incurred without regard to the timings of cash disbursement

5.12 Terminal Questions 1. Prepare a trial balance from the following:

Particulars Drawings Stock on 1-1-92

Amount Rs. 10,000 46,000

Particulars

Amount Rs.

Interest on investments 200 Sundry Debtors 36000

Purchases Purchases return Cash in hand Bank Balance Freehold expenses Trade expenses

1,50,200 600 3400 22660 38600 840

Sundry Creditors Wages Salaries Capital Income tax Discount allowed Discount received

29,000 25,000 14,000 1,14,000 1600 6300 4600

Printing Stationery & 1640 advertisement Professional charges 280 Commission received 3300 Investments as on 1st 4000 Jan @ 10% Office furniture Rent rates and insurance Total 3050 4000

Sales Sales return Bills receivable

2,08,950 550 3200

Bills payable

10000

Provision for bad debts 670

371320

Total

371320

2. The following Trial balance was extracted from the books Chetan, a small businessman. Do you think that it is correct? If not, rewrite it in the correct form.

Debits Stock Purchases Returns outwards

Rs. 8250 12750 700

Credits Capital Sales Returns inwards

Rs. 10000 15900 1590

Discount received Wages and salaries Rent and rates Sundry debtors Bank Overdraft

800 2500 1850 7600 2450

Discount allowed Scooty Carriage charges Sundry Creditors Bills payable

800 1750 700 7250 690

1. Rectify the following errors: a. Purchases from Padma Rs.191 posted to her account as Rs.119 b. Purchases from Lata credited to her account as Rs.117 c. Salaries Rs.400 posted to salaries account as Rs.300 d. A cash sales of Rs.430 to Rita posted as Rs.43 5.13 Answers to SAQs and TQs
SAQ:

1. True 2. Capital 3. Goodwill 4. Opening entry:

5. True 6. Trading

25. Trial balance. 26. final accounts

7. Trade expenses 8. No 9. True 10. a. Depreciation a/c Dr To Building account b. Closing stock A/c Dr To Trading A/c c. Pre-paid Insurance a/c Dr To insurance A/c d. Salaries A/c To outstanding salaries account. e. Drawings A/c Dr To stock account 11. True 12. True 13. True 14. True 15. True

27. Error that are disclosed by trial balance and those which cannot be disclosed by trial balance. 28. Error of commission. 29. Error of principle. 30. Omitting completely a transaction from books of original entry. Sales made to Raghu of Rs 12000 completely ignored. 31. Error of omission, commission, principle, compensating error. 32. Suspense account. 33. Journal proper 34. Four steps to locate the errors Check the total of both sides of trial balance, Total debtors & creditors accounts, Verify whether balancing is done correctly, Check the totals of ledger balances etc. 35.Suspense a/c Dr. 45 To Sales a/c 45

Being sales account which was under 16. It is a wrong posting and hence it is cast rectified errors of commission 36. Suspense a/c Dr.9 Ramas a/c Dr 500 To Return Inward a/c 9 Ramanans a/c Dr 500 Being returned inward book which

To Suspense a/c 1000

was over cast rectified.

(Being amount paid to Rama wrongly 37. Salary a/c Dr. credited to Ramanans account rectified) To Gopal a/c 17. Posting of wrong amount Trial balance is affected. Profit (gross) is reduced by Rs 13500. 18. Suspense a/c Dr. 13,500 To Discount received a/c Rs.100 To Wages a/c 13,500 Being wrong posting in discount (Being excess debit to wages account received a/c rectified. rectified) 39. This is an error of Commission. 19. Error of commission By checking the original invoice document, it can be rectified. 20. False Purchase a/c Dr Rs.90 21. Error of omission To Creditors a/c Rs.90 22. True Being wrong posting in purchase 23.True book rectified. 24. True Terminal Question Answers: 1. Trial balance total 371320 2. Trial balance total 37790 Being wrong debit given to Gopal rectified 38. Suspense a/c Dr. Rs.100

Activity 1 Solution

Copyright 2009 SMU Powered by Sikkim Manipal University


.

MB0041-Unit-06- Final Accounts


Unit-06- Final Accounts Structure: 6.1 Introduction 6.2 Adjustments before preparing final accounts 6.3 Depreciation 6.4 Bad Debts and accounting treatment of bad debts 6.5 Provision for doubtful debts 6.6 Reserves for Discount on Debtors 6.7 Reserve for Discount on Creditors

6.8 Closing Stock 6.9 Trading Account 6.10 Profit and Loss Account 6.11 Balance Sheet 6.12 Summary 6.13 Terminal Questions 6.14 Answers 6.1 Introduction The previous chapter has provided a birds eye view of the methods of preparing trial balance, different types of errors and the rectification process. Trial Balance is a statement of debit balances and credit balances that are extracted from ledger accounts. This unit deals with matters relating to adjustments before the preparation of final accounts, depreciation methods, preparation and presentation of Final accounts covering both trading and corporate enterprise. Financial statement encompasses two vital statements the position statement reflecting the assets, liabilities and capital of a business entity on a particular date called the Balance Sheet, and the other called the Profit and Loss account showing the results of business operations during the given period. Corporate financial statements should conform to the Companies Act 1956 requirements. The Balance sheet of a company should be presented in accordance with Part 1 of Schedule VI of the Companies Act. Objectives: After studying this unit, you should be able to: 1. Incorporate such transactions left out and various adjustments with regard to transactions taking place after the trial balance but relating to the current period. 2. Incorporate adjustments such as Reserve for bad debts, Reserve for discounts on Debtors, Reserve for discount on Creditors, bad debts out side the trial balance. 3. Incorporate adjustments relating to prepaid expenses, outstanding expenses, pre-received incomes, outstanding incomes etc. 5. Prepare Trading, Profit and Loss account and Balance Sheet of both trading companies and corporate entities. 6.2 Adjustments before preparing final accounts

The Generally Accepted Accounting Principles (GAAP) supports the accrual basis of accounting according to which revenue is recognized when it is earned and expenses are recognized when they are incurred irrespective of their actual receipt or actual payment. In the accrual basis of accounting adjustment entries are prepared at the end of the period to record any changes in assets, liabilities, revenue incomes and revenue expenses. Adjusting entries are regarded as internal transactions. A. Outstanding Expenses Expenses yet to be paid or outstanding expenses for the current period should be charged against the current periods income. The extent to which the amount belongs to the current year but payable in the next year is called outstanding expenses. Salaries outstanding (March 20XX month salary paid in April 20XX) Rent outstanding

Prepaid Expenses Expenses paid in advance or prepaid expenses should be not be charged against the revenues relating to the current period but taken to the coming period. Prepaid expenses form an asset and therefore prepaid expenses account is debited. Salaries paid in advance Insurance paid in advance Rent paid in advance Example: insurance premium is paid from April, 2004 to March, 2005 and the amount is Rs.3600. The accounting year of the firm ends on 31st December, 2004. Therefore the premium relating to Jan, Feb and Mar of 2005 amounting to Rs.900 is said to have been paid in advance. To record this internal adjustment, the entry is

B. Income received in advance Income received in advance that does not belong to the current period should not be considered.

Rent received in advance

Example: Rent received for one year from 1-4-2005 to 31-3-2006 Rs.48000. Accounts are finalized on 31-12-2005. Therefore rent received for January, February and March of 2006 is said to have been received in advance Rs.12000. The entry is

C. Accrued Income Accrued income is also called outstanding income. Income yet to be received for the current period should be considered as income for the current period whether actually received in cash or not. Outstanding income account is a personal account and it represents an asset.

Interest accrued but not received

Example: Interest accrued on Fixed Deposit of Rs 200000 at 12% simple interest on 31-122006, not yet received. The entry is

Self Assessment Questions: 1. Expenses due but not yet paid are known as ______ 2. Prepaid expenses appear on the asset side of balance sheet. (State True / False). 3. Income earned but not received is called ____________. 4. Any income received in advance is a liability (state True / False) 5. Advertisement expenses outstanding for the year ending March 2005 is Rs.5000. Give the journal entry 6.3 Depreciation Depreciation is reduction in the value of an asset due to constant use of the same, which is called wear and tear. Fixed assets like, buildings, plant, machinery, furniture etc., are subject to depreciation. Whenever, an asset is depreciated, its value goes down and therefore it is a loss to the organization.

Depreciation account is debited and the concerned asset account is credited. The item of depreciation may appear in the trial balance, which means that already the concerned asset is reduced by the amount of depreciation. If depreciation is given as an additional adjustment, then the depreciation amount should be charged against profit and loss account on one hand and the concerned asset account is reduced on the other hand in the balance sheet. There are two popular methods of depreciation:

Fixed installment method (Straight line method) Reducing balance method (Written down value method).

In fixed installment method, depreciation is calculated on cost of the asset. The depreciation charged remains same throughout the life of the asset. In case of reducing balance method (Diminishing balance method), the depreciation is charged on the reducing balance of the book value of the asset. The depreciation amount gets reduced year after year during the life of the asset. Reducing balance method is more popular and well recognized. Example: The book value of the building is Rs.400000. It is depreciated at 10% on fixed installment method. Show the journal entry and how does it appear in the balance sheet for the first and second year under both the methods.

1. Depreciation charged for the first year under straight line and reducing balance method. 2.

During the first year the depreciation charged is similar under both the methods. 3. Depreciation for the second year under:

Self Assessment Questions: 6. Depreciation is for __________ of an asset.

7. What entry is drawn if furniture costing Rs.50000 is depreciated at 5% under straight line method? 8. Calculate the depreciation amount charged for the second year under written down or reducing balance method for Plant and Machinery costing Rs.7,50,000. The depreciation rate is 10%. 6.4 Bad Debts Bad debts are those debts which are not recovered. Bad debts form loss to the business and reduce the amount of debtors.

Since bad debts are losses, they are debited and the debtors account is credited. If bad debts are recovered, cash account is debited and bad debts recovered account is credited. Accounting treatment of Bad debts: A. If bad debts are identified and shown in trial balance:

B. If bad debts are shown outside the trial balance: This denotes bad debts that they were identified after the preparation of Trial Balance. Two adjustments should be incorporated. Example: The sundry debtors for the year 2005 are Rs.50000. The bad debts amounted to Rs.4000 as on 31-12-2005 already shown in the trail balance. Write off further bad debts Rs.5000. Show how the above internal adjustments appear in the Profit and loss account and Balance Sheet. Solution Bad debts shown in the trial balance is Rs.4000 and not shown in the trial balance is Rs.5000. To incorporate those bad debts not yet shown in the trial balance, the adjusting entry is

Self Assessment Questions: 9. Unrecovered debts are called ______. 10. Bad debts are not expenses but they incur loss to the firm. (state True / false) 11. If bad debts are recovered, what entry can be drawn? 6.5 Provision for Doubtful Debts Debts that cannot be recovered are called bad debts. Debts that are doubtful in recovery are called doubtful debts. From the past experience of the business proprietor, what percentage of good debts may become bad in future can be estimated and in the current year an equal amount of profit is set aside. This provision is also known as Reserve for Bad Debts or Provision for Doubtful Debts or Reserve for Doubtful Debts. Since the provision for bad debts is a charge against current year profit the adjusting entry is

Provision for bad debts is a liability to be incurred in future and so it should appear on the liability side of balance sheet. However, the convention is RBD (Reserve for Bad and Doubtful Debts) is deducted from the amount of good debtors. The important note here is that RBD is computed as a percentage of good debts, which means total debtors minus bad debts unadjusted. Provision for bad and doubtful debts is a running account and every year the amount keeps on changing because from the provision made in the current year, bad debts occurring in the following year have to be adjusted and additional amount of provision to be made is calculated.

Illustration 1: On 1st January 2006, the RBD account stood at Rs.9000 in the books of a merchant. The bad debts written off during the year ended 31st December, 2006 amounted to Rs.4800 and Sundry Debtors stood at Rs.480000. It was desired to maintain the reserve for bad debts at 5% on Debtors. During the year 2007 bad debts written off amounted to Rs.12000 and sundry debtors

on 31st December 2007 amounted to Rs.380000. As usual 5% reserve was required. Show the journal entries for recording the above transactions and write up the bad debts reserve account.

Solution

Additional reserve required to be provided in P&L a./c in 2006 is Rs19800

Additional reserve required to be provided in P&L a./c in 2006 is Rs. 7000

Self Assessment Questions: 12. What is the difference between Bad debts and doubtful debts? 13. Provision for Doubtful debts is a charge against the profits of the firm (state True / False ) 14. Bad debts incurred in the subsequent period are written off against reserve for bad debts (state True / False). 15. Give the journal entry for writing off of bad debts against RBD? 6.6 Reserves for Discount on Debtors There are two types of discounts allowed to customers in a business. One is trade discount and the other is cash discount. After anticipating the amount of cash discount allowable, a provision is made in the current year itself. In the subsequent years, the actual discount allowed is set off against the provision for discount on debtors. Every year, the amount of provision for discount on debtors is deducted from the profits. The entry for making the provision is

The following guide lines may be kept in mind while dealing with the reserve for discount on debtors 1. If a reserve for discount on debtors do not exist and cash discount is allowed then transfer the discount to P&L account. 2. Any fresh reserve for discount on debtors is to be made, debit the P&L Account with the amount of reserve. 3. If provision for discount on debtors exists at the time of providing discount, then write off the discount from the provision already made for the purpose. 4. New provision should then be calculated and only as much as required to bring the existing provision to the new figure should be debited to P&L Account. 5. If the new provision required is lower than the provision already existing (old), then the difference shows profit and transfer the same to P&L Account. Illustration 2: The following items are found in the trial balance of M/s Sharada Enterprise on 31st December 2000.

Sundry Debtors Rs. 160000 Bad Debts written off 9000 Discount allowed to Debtors 1800 Reserve for Bad and doubtful Debts 31-12-1999 16500 Reserve for discount on Debtors 31-12-1999 3200 You are required to provide for the bad and doubtful debts at 5% and for discount on debtors at 2%. Give necessary journal entries and show bad debts account, bad debts reserve account, discount account and provision for discount on debtors account. Solution

Date 31.12.2000

Particulars RBD a/c Dr To Bad debt a/c Being bad debts written off against existing reserve P&L a/c Dr

L.F Debit 9000

Credit 9000

-doTo RBD a/c Being addition to RBD to make the new RBD equal to 5% of 160000 Reserve for discount on debtors a/c Dr To Discount on Debtors a/c Being discount on debtors written off against reserve for discount on debtors P&L a/c Dr -doTo Reserve for discount on debtors a/c Being additional reserve made to make the new reserve at 2% on

500

500

-do-

1800

1800

1640

1640

152000 NOTE: 1. The amount debited to P&L Account towards RBD is computed as follows Old RBD = Rs. 16500 (-) Bad debts = 9000 Balance = 7500 New RBD @5% on160000 = 8000 RBD to be provided = 500 (8000-7500) 2. The amount debited to P&L Account towards Reserve for Discount on Debtors is computed as follows: Good Debtors = 160000 8000 (New RBD)=152000 Old Res for Dis On Drs = Rs. 3200 Less Discount on Drs = 1800 Balance Reserve = 1400 New Res for Disc at 2% On good drs 152000 = 3040 Res for Discount to be Provided now = 1640 (3040 -1400)

In the balance sheet, the Sundry debtors are reduced by bad debts shown out side the trial balance, the new RBD, discount on debtors shown out side the trial balance and the new Reserve for discount on debtors. 6.7 Reserves for Discount on Creditors Just as reserve is for discount on debtors is created, reserve for discount on creditors is also created. Businessman expects that he would receive discounts from suppliers (creditors), when the businessman remits cash to them. Anticipating some percentage of creditors being received as discount in the coming year, the business proprietor makes a provision for the expected income in the current year itself. Discount on creditors is an income and therefore reserve for discount on creditors is debited and profit and loss account is credited to show it as anticipated profit. In the subsequent year, when discount on creditors is actually received, it is first set of against provision for discount on creditors and the difference between the new provision for discount on creditors and the balance of old provision left over is carried to P&L Account. Discount on creditors is income and to that extent the creditors due is reduced. So the journal entry to record them is

If the discount received is adjusted against reserve for discount on creditors, the entry will be

When provision for discount on creditors is made in P&L Account, the entry is

The amount of provision for discount on creditors is calculated at a percentage on creditors. In the balance sheet, creditors are shown after deducting reserve for discount on creditors. 6.8 Closing Stock Stock of goods raw materials, semi finished goods, finished goods at the end of the accounting year should be considered for preparing trading account and balance sheet. It is an internal adjustment. Closing stock is normally valued at cost or market price which ever is lower, even though there are several other methods to value stock. Closing stock does not appear in the trial balance because the value of it is ascertained only after the preparation of trial balance. To bring to the records, a journal entry is passed in journal proper by debiting closing stock account and crediting trading account. In the balance sheet, closing stock appears as an asset.
Self Assessment Questions:

16. Provision for Discount on Debtors is a charge against P & L a/c. (State True or False) 17. Provision for discount on debtors appears as a liability in the Balance Sheet (State true/false) 18. Discount on creditors is an item of income (State True / false ). 19. Provision for discount on creditors is shown as an anticipated income (State True/False). 20. What is the entry for adjusting the closing stock? 21. Closing stock always appears as an asset in balance sheet (State true/false) 6.9 Trading Account Trading account shows gross profit or gross loss arising out of trading activities. Trade means buying and selling. The account mainly focuses on finding the result of goods bought and goods sold. For every expense outstanding and prepaid aspects must be considered. The excess of credit over debit is gross profit and excess of debit over credit is termed as gross loss. The gross profit or gross loss is transferred to Profit and Loss Account. The format of a Trading Account is given below: To prepare Trading Account, the following steps may be followed: a) Identify the items of expenses relating to trading and show them on the debit side of Trading Account.

b) Effect the adjustments such as outstanding or prepaid to the relevant items of expenses c) Show the sales less returns and closing stock on the credit side of trading account d) The difference is gross profit if credit total is more than debit and gross loss if debit total is more than credit. e) Transfer the gross profit or gross los to Profit and Loss Account as the case may be.

Illustration 3: From the following balances extracted from Trial balance, prepare Trading Account. The closing stock at the end of the period is Rs. 56000 Particulars Stock on 1-1-2004 Returns inwards Returns outwards Purchases Debtors Creditors Carriage inwards Carriage outwards Amount in Rs. 70700 3000 3000 102000 56000 45000 5000 4000

Import duty on materials received from abroad Clearing charges Rent of business shop Royalty paid to extract materials Fire insurance on stock Wages paid to workers Office salaries Cash discount Gas, electricity and water Sales

6000 7000 12000 10000 2000 8000 10000 1000 4000 250000

6.10 Profit and Loss Account Profit and loss account is an important final account that reveals the net result of the business in the form of net profit or net loss. All revenue receipts are those which are received regularly arising out of day to day activities of the business and all revenue payments, which are known as expenses are incurred regularly are recorded in profit and loss account. Any capital receipts or capital payments are not considered while preparing profit and loss account. The following steps may help to prepare Profit and Loss Account 1. Identify the expenses and bring them to debit side of P&L Account 2. Identify the revenue incomes and put them on the credit side of P&L Account

3. Check whether all adjustments like outstanding, prepaid, pre received expenses and incomes as the case may be are brought to the account 4. Check the transfer of reserves to the relevant sides of the account 5. Transfer the net profit / net loss to the capital account

Illustration 4: The following Trial Balance is extracted from the books of a merchant on 31-122004. Furniture and fittings Motor Vehicles Buildings Capital Account Bad Debts Provision for Bad debts Sundry Debtors Sundry Creditors Stock on 1-1-2004 640 6250 7500 12500 125 200 3800 2500 3460 Bank Over Draft Sales Returns Purchase Returns Advertising 2850 200 125 450

Interest on Bank Over Draft118 Commission Cash Taxes and Insurance General Expenses 375 650 1250 782

Purchases Sales

5475 15450

Salaries

3300

The following adjustments are to be made: 1. Stock in hand on 31-12-2004 was Rs.3250 2. Depreciate Buildings at the rate of 5%, Furniture and fittings @ 10% and Motor Vehicles @ 20%. 3. Rs.85 is due for interest on bank overdraft. 4. Salaries of Rs300 and taxes Rs.120 are outstanding. 5. Insurance amounting to Rs.100 is prepaid 6. One-third of the commission received is in respect of work to be done next year 7. Write off a further sum of Rs.100 as bad debts and provision for bad and doubtful debts to be made equal to 10% on sundry debtors. 8. Prepare Trading Account and Profit and Loss Account.

6.11 Balance Sheet Corporate financial statement is financial statements / annual accounts of corporate enterprises that are prepared in conformity with the Companies Act 1956. Section 209 of the companies Act requires that every company should keep at its registered office proper books of accounts.

Section 210 of the Companies Act stipulates that at every annual general meeting of a company, the Board of Directors should lay (a) a balance sheet as at the end of the period and (b) a profit and loss account for the period (financial year). Section 211 requires that every balance sheet of a company should provide a true and fair view of the state of affairs of a company as at the end of the financial year and it should be set out in the form prescribed in Part 1 of Schedule VI of the Companies Act 1956. Section 215 of the Act requires that every balance sheet and P&L account duly approved by the Board of Directors should be signed on behalf of the Board of Directors of a company by its managers or secretary, if any, and at least two directors one of whom should be a managing director, if any. Section 216 of the Act requires that the profit and loss account of a company is annexed to the balance sheet and the auditors report including separate, special or supplementary report if any.

Share Capital Share capital refers to the capital raised by a company by the issue of share in a company is one of the units into which the total capital of the company is divided. Preference Share Capital Preference share capital is that part of share capital of a company which carries preferential right in respect of both dividend payment and repayment of capital. Equity Share Capital All share capitals which are not preference share capital are equity share capital. In other words, holders of equity share capital can get dividend only after it is paid to preference shareholders. Authorized Share Capital

It is the maximum amount of the equity share capital and preference share capital the company can rise in its life time. Issued Capital It is that part of the equity and preference share capital which has been actually issued by the company for cash and for considerations other than cash Subscribed Capital It refers to that part of the equity and preference share capital which has been actually allotted by the company. Called-up Capital It refers to that part of the allotted capital which has been called-up by the company and the part which has not been called-up is the uncalled capital. Paid up Capital It refers to the amount released on the called-up capital and it is equal to the called-up capital minus unpaid calls.

Illustration 5: From the following trial balance of Anjana Machineries Limited and additional information, prepare final accounts of the company as per schedule VI of the Companies Act.

Additional information: 1. Stock as at March 31,2008

Raw materials and stores Rs. 1,45,000; Work-in-process, Rs. 22,000; Finished goods, Rs. 1,98,000. 2. Provide depreciation on written down value basis on plant and machinery @ 20 per cent per annum and on furniture @ 15 per cent annum and on freehold premises @ 5 per cent per annum 3. In the middle of the year machine costing Rs. 3,00,000 was purchased and duly recorded 4. Sundry debtors include Rs. 18,000 due for more than six months. Provide for bad and doubtful debts @ 5 per cent on debtors. 5. Market value of investments is Rs. 3,19,000 6. Make a provision for income-tax @ 35 per cent 7. Corporate dividend tax is 14.025 per cent including surcharge of 10 per cent and education cess of 2 per cent. 8. The Board of Directors has recommended a final dividend @ 15 per cent on equity shares. 9. Transfer Rs. 1,00,000 to debenture redemption reserve. 10. Transfer minimum amount to statutory reserve as required by company law. 11. Provision for depreciation on freehold premises as in 31/03/2007 was Rs. 12,70,000. 12. Write of 1/5th of preliminary expenses 13. Interest on debentures becomes due on October 31 and March 31.

Solution

6.12 Summary The Generally Accepted Accounting Principles (GAAP) supports the accrual basis of accounting according to which revenue is recognized when it is earned and expenses are recognized when they are incurred irrespective of their actual receipt or actual payment. Outstanding expenses yet

to be paid or outstanding expenses for the current period should be charged against the current periods income. Prepaid expenses paid in advance or prepaid expenses should be not be charged against the revenues relating to the current period but taken to the coming period. Depreciation is reduction in the value of an asset due to constant use of the same, which is called wear and tear. Fixed assets like, buildings, plant, machinery, furniture etc., are subject to depreciation. Whenever, an asset is depreciated, its value goes down and therefore it is a loss to the organization. Corporate financial statement is financial statements / annual accounts of corporate enterprises that are prepared in conformity with the Companies Act 1956. Section 209 of the companies Act requires that every company should keep at its registered office proper books of accounts. 6.13 Terminal Questions 1. Explain the meaning of Depreciation. Mention the different types of depreciation. 2. What is Bad debts? Mention the accounting treatment of bad debts 3. What is Provision for Doubtful Debts? 4. What are the guidelines that deal with reserve for discount on debtors? 5. Explain reserve for discount on creditors with suitable examples. 6.14 Answer to SAQs and TQs 1. Outstanding expenses 2. True 3. Accrued or Outstanding income 4. True 12. Bad debts are totally not recoverable, doubtful debts may be recovered. 13. True 14. True

15. Reserve for Bad debts a/c Dr 5. Advertisement expenses a/c Dr To Outstanding Advertisement expenses a/c To Bad debts a/c 6. Wear and Tear 16. True 7. Depreciation a/c Dr.2500 17. True To Furniture a/c 2500 18. True Being furniture depreciated

8. (7,50,000-75,000) = 6,75,000 x 10% 19. True = 67,500 20. Closing stock a/c Dr 9. Bad debts To Trading a/c 10. True 21. True 11. Cash a/c To Bad debts recovered a/c Answer for Terminal Question: 1. Refer Unit 6.3 2. Refer Unit 6.4 3. Refer Unit 6.5 4. Refer Unit 6.6 5. Refer Unit 6.7 Copyright 2009 SMU Powered by Sikkim Manipal University

MB0041-Unit-07-Introduction to Management Accounting


Unit-07-Introduction to Management Accounting Structure: 7.1 Introduction 7.2 Meaning of Management accounting 7.3 The Role of Management Accounting 7.4 Management Accounting Framework 7.5 Functions of Management Accounting 7.6 Tools of Management Accounting

7.7 The Balanced Scorecard 7.8 Cost Management System 7.9 Value Added Concept 7.10 Merits of Management Accounting 7.11 Demerits of Management Accounting 7.12 Distinction between Management Accounting and Financial Accounting 7.13 Summary 7.14 Terminal Questions 7.15 Answers to SAQs and TQs 7.1 Introduction The origins of modern management accounting can be traced to 19th century enterprises like textile mills, family owned business etc. These enterprises were formed as a single large organization wherein different processes were performed taking advantage of the economies of scale. Slowly a need was felt to measure the efficiency of the internal production processes. Typically for textile mill, the internal measure could be cost per yard, and for a fertilizer unit it may be cost per tonne of fertilizer Management accounting system plays a vital note in helping the managers to plan & control their operation. A good management accounting system may not guarantee competitive success if the firm does not have good products, efficient operating process, effective marketing & sales activities.

Objectives: After studying this unit, you should be able to: 1. Explain the meaning of Management Accounting 2. Explain the strategic role of management accounting in the present day setup. 3. Describe the functions of management accounting 4. State the tools of management accounting

5. Know the balance score card, Cost management system and value added concept 6. Identify the differences between management accounting and financial accounting. 7.2 Meaning Managerial accounting is the process of identifying, measuring, analyzing, interpreting and communicating information in pursuit of an organizations goal. Managerial accounting is an integral part of the management process. Management accountants have taken leadership roles; act as trusted advisor and transformed into strategic business partner for the valuable contribution they provide. The American Accounting association has defined Management Accounting as the application of appropriate techniques and concepts in processing historical and projected economic data of an entity to assist management in establishing plans for reasonable economic objectives in the making of rational decisions with a view towards achieving these objectives. Management Accounting is the presentation of accounting information in such a way as to assist management in the creation of policy and in the day-to-day operations of an undertaking. 7.3 The Role of Management Accounting Management accounting information serves several major roles. One of the most important types of management accounting information is cost information. Firms use cost information to make important product features and product mix decision. Also organizations use appropriate cost information to improve efforts on activities that are major contributors to cost. The accounting people are expected to do things that are much more strategic and forward looking. Nowadays managerial accountants serve as internal business consultants involving in all areas of business. They take on leadership roles on their teams and are sought out for the valuable information they provide. The role of the accountants in leading edge companies has been transformed from number cruncher and financial historian to being business partner and trusted advisor. Another important transformation is the role it plays in strategic management highlighted in Balanced Score Card and Value Chain. Management accountants add value to an organization by pursuing five major objectives: 1. Providing information for decision making and planning and proactively participating as part of the management team in decision making and planning processes. This includes budget preparation on projected revenues and costs. 2. Assisting managers in directing and controlling operational activities. Managerial accounting information often assists management through its attention-directing function. However managerial accounting information often directs managers attention to an issue that requires their skills.

3. Motivating managers and other employees towards the organizations goals. Organizations have goals and organization comprise people who have goals of their own. The goals of individuals are diverse and they do not always match those of the organization. The main purpose of managerial accounting is to motivate managers and other employees to direct their efforts with the help of budgeting. 4. Measuring the performance of activities, subunits, managers and other employees within the organization. Such measurements can be used as the basis for rewarding performance through positive feedback, promotions and pay raises. 5. Assessing the organizations competitive position due to ever changing business environment. These changes are reflected in global competition, fast changing technology, improved communication system etc. Organizations competitive position is determined by how well it is doing from customers perspective. Secondly how the organization is performing its internal operations and business processes. Thirdly how well is it in research and development? Does it provide a platform for learning, continuous improvements and adaptation to new technology. Finally it assesses the financial health in terms of liquidity, ability to raise finance, viability and future growth. Self Assessment Questions: 1. Management accountants have transformed from mere cost information providers to _______________________ 2. Management accountants facilitate in budget preparation on projected ______ 3. Management accountants assess organizations _______________that arise due to changing business environment. 4. Competitive position of a firm is determined by how well it provides a platform for __________, ___________________ and adaptation to new technology. 7.4 Management Accounting Framework For offering accounting and financial advice as well as for capitalizing the available opportunities for future development, it is necessary that an effective frame must be designed. The management accountant must organize the whole accounting division in such a way that there is prompt and immediate recording of the entire information flow into the department from functional and service department. The frame must concentrate on. Getting rid of routine work Reporting actual and planned performance Fixing organizational responsibilities

Application of new modern and modified practices of analyzing and interpreting results. Designing of sound and efficient organization taking into account the native and size of the business unit.

7.5 Functions of Management Accounting Management accounting functions may be said to include collecting, processing, interpreting and presenting information to management. Management Accounting serves as a vital source of data for management planning. The data thus collected are properly complied and classified. The accounting data is then analyzed meaningfully for effective planning and decision making through comparative statements. It provides a means for communicating management plans upward, downward and at lateral levels. It helps in translating given objectives and strategies into specified goals for attainment by a specified time and secures effective accomplishment of these goals in an effective manner. More specifically, it provides invaluable service to the management at every stage of functioning. Major functions are: Forecasting and Planning: Short and long term forecasts are very essential. Planning the future operations of a business is crucial. This includes profit planning, programmes of capital investment and financing, sales forecast, expense budget and cost standards. Controlling Performance: The management accounting is very helpful in controlling the financial performance of the organization through comparing actual performance with operating plans and standards and to report and interpret the results of operations to all levels of management. Coordinating: Management accountant consults various departments and is responsible for policy decisions. Coordination increases the efficiency of an organization. Other functions: Management accounting serves in a number of other ways. It supplies useful information to different functional authorities. It provides accounting information and advice for price determination and pricing decisions. It also helps in making certain strategic decisions, decisions regarding seasonal or temporary suspension of production, make or buy decisions, replacement decisions etc. Self Assessment Questions: 5. Management Accounting serves as a vital source of data for management _________. 6. It provides a means for communicating management plans ______, __________and at _________levels. 7. The management accounting is very helpful in controlling the financial performance of the organization through comparing actual performance with __________________.

7.6 Tools of Management Accounting Management Accounting uses the following tools or techniques to fulfill its responsibilities and duties towards management. Financial Statement Analysis Funds Flow Analysis Cash Flow Analysis Costing Techniques that includes marginal costing, differential costing, standard costing, and responsibility costing Budgetary control Management Reporting. Financial Statements are indicators of two significant factors that include profitability and financial soundness. Analysis and interpretation of financial statements enables full diagnosis of the profitability and financial soundness of the firm. Analysis means methodical classification of the data given in the financial statements. Methodical classification enables comparison of the various inter-connected figures with each other. Interpretation explains the meaning and significance of the data. Funds Flow Analysis is an important tool for management accountant. It reveals the changes in working capital position, the sources from which the working capital was obtained and the purpose for which it was used. It also reveals the changes that have taken place behind the Balance Sheet. Cash Flow Statement identifies the sources and application of cash. It is prepared on the basis of actual or estimated data. It depicts the changes in the cash position from one period to another. A projected cash flow or a cash budget will help the management in ascertaining how much cash will be available to meet obligations to trade creditors, to pay bank loans and to pay dividends to the shareholders. Standard Costing is the preparation and use of standard costs, their comparison with actual costs and the analysis of variance. It discloses the cost of deviations from standards. It aims at assessing the cost of a product, process or operation under standard operating conditions. Budgetary Control has become an essential tool of management for controlling costs and to maximize profit. It helps to compare the current performance with pre-planned performance thereby correcting the deviations if any.

Management Reporting System is an organized method of providing each manager with all the data and only those data which he needs for his decisions, when he needs them and in a form which aids his understanding and stimulates his action. Self Assessment Questions: 8. ______________ is the preparation and use of standard costs, their comparison with actual costs and the analysis of variance. 9. ___________ will help the management in ascertaining how much cash will be available to meet obligations to trade creditors, to pay bank loans and to pay dividends to the shareholders. 10. ______________________reveals the changes in working capital position, the sources from which the working capital was obtained and the purpose for which it was used. 11. ____________ means methodical classification of the data given in the financial statements. 12. __________________helps to compare the current performance with pre-planned performance thereby correcting the deviations if any 7.7 The Balanced scorecard The Balanced Score Card is a framework for integrating measures derived from strategy. While retaining financial measures of past performance, the Balanced Score Card introduces the drivers of future financial performance. (Figure 1) The drivers (customer, internal business process, learning & growth perspectives) are derived from the organizations strategy translated into objectives and measures. The Balanced Score Card is more than a measurement system it can be used as an organizing framework for their management processes. The real power of the Balanced Score Card is when it is transformed from a measurement system to a management system. It fills the void that exists in most management systems the lack of a systematic process to implement and obtain feedback about strategy

In India few noted organizations have adopted Balanced Score Card successfully. The commercial vehicles business unit (CVBU) of Tata Motors was among the first Asian organizations to be inducted into the prestigious Balanced Scorecard Hall of Fame, in recognition of its exemplary success with the model. The company is one of the worlds top 10 truck manufacturers and the CVBU began deployment of Balanced Scorecard in 2000, in an attempt to remedy years of poor financial performance. The focus was on achieving a turnaround, and then progressing to sustainable growth. Within 2 years of implementation, the company began to show tangible improvement in performance including a 40% growth in revenue. The implementation of the Balanced Scorecard has enabled them to focus on different elements of operational performance. Defining, cascading and communicating strategies across the organization have brought about transparency and alignment. The scorecard incorporates SQDCM (Safety, Quality, Delivery, Cost and Morale) and VMCDR (Volume, Market Share, Customer Satisfaction, Dealer Satisfaction and Receivables). 7.8 Cost Management System The explosion in technology we are experiencing, coupled with increasing worldwide competition is forcing managers to produce high quality goods and services, provide outstanding customer service and do so at the lowest possible cost. Many companies have moved away from a historical cost accounting perspective and toward a proactive cost management perspective. A cost management system is a management planning and control system with the following objectives. 1. To measure the cost of the resources consumed in performing the organizations significant activities 2. To identify and eliminate non-value added costs. These are the costs of activities that can be eliminated with no deterioration of product quality, performance or perceived value. 3. To determine the efficiency and effectiveness of all major activities performed in the enterprises. 4. To identify and evaluate new activities that can improve the future performance of the organization. 7.9 Value Added Concept Value added concept is a performance measure and it reports the wealth generated by a business undertaking over a period of time. According to ICAI, the term value added refers to the increase in value of a product or service resulting from an alteration in the form, location or availability excluding the cost of bought out materials and services It is calculated by deducting the value of goods and services purchased from sales revenue. It is a versatile measure for evaluating the performance and efficiency of a firm because value added measure caters all concerned parties viz., owners, workers, government, lenders etc.

Value added Statement is based on enterprise theory rather than entity theory. Enterprise theory encompasses everyone unlike entity theory which focuses on owners or shareholders of the firm. Self Assessment Questions: 13. _____________is a performance measure and it reports the wealth generated by a business undertaking over a period of time 14. ____________________identify and eliminate non-value added costs 15. The drivers of Balanced Score Card are ___________, _________________,___________________ and _____________________ 7.10 Merits of Management Accounting Merits 1. Efficient planning and organization which are the end product of the system of management accounting bring systematic regularity in the business activities. 2. Maximum return on capital employed is ensured by the use of management accounting because it helps in the functions of planning, coordination and control 3. Better and improved services by management to customers are assured by this system. 4. Management accounting removes unacceptable standards or sub-standards. 5. Industrial relations may be improved by adoption of management accounting principles. 6. Eliminations of various types of wastages, production defectives and other related work deficiencies are removed with the help of management accounting. 7. Economic uplift of community and development of nations economy can be achieved by the use of management accounting.
7.11 Demerits of Management Accounting

1. Most of the information used in Management accounting is derived from financial accounting records or cost accounting records other records. As such fairness and accuracy of decisions deduced depends to a greater extent upon fairness and accuracy of these original records. 2. Decisions or conclusions derived are insignificant unless properly executed at all levels of business operations.

3. Management accounting is a mere tool for management. It cannot substitute for financial accounting. 4. The evolution has been on account of inter-alia development of new theories in other sciences. Hence there is a need to have a comprehensive knowledge and understanding of all these related disciplines to derive the full advantage. 5. Management accounting is still in its evolutionary stage. Hence, there is an uncertainty in its use. 6. The installation of management accounting is a costly affair and as such it has very limited scope for its use. 7.12 Distinction between Management Accounting and Financial Accounting Financial accounting is the preparation and communication of financial information to outsiders such as creditors, bankers, government, customers and so on. Another objective of financial accounting is to give complete picture of the enterprise to shareholders. Management accounting on the other hand aims at preparing and reporting the financial data to the management on regular basis. Management is entrusted with the responsibility of taking appropriate decisions, planning, performance evaluation, control, management of costs, cost determination etc., For both financial accounting and management accounting the financial data is the same and the reports prepared in financial accounting are also used in management accounting But the following are major differences between Financial accounting and Management accounting.

Financial accounting The primary users of financial accounting information are shareholders, creditors, government authorities, employees etc., Accounting information is always expressed in terms of money

Management accounting Top, middle and lower level managers use the information for planning and decision making

Management accounting may adopt any measurement unit like labour hours, machine hours or product units for the purpose of analysis Reports are prepared on continuous basis, monthly or weekly or even daily

Financial data is presented for a definite period, say one year or a quarter Financial accounting focuses on historical data

Management accounting is oriented towards future

Financial accounting is a discipline Management accounting makes use by itself and has its own principles, of other disciplines like economics, policies and conventions management, information system, operation research etc., Self Assessment Questions: 16. Management accounting aims at preparing and reporting financial data to _____________. 17. Management accounting is oriented towards ______ 18. In ____________financial data is presented for a definite period, say one year or a quarter 19. __________accounting may adopt any measurement unit like labour hours, machine hours or product units for the purpose of analysis 7.13 Summary Management Accounting is the presentation of accounting information in such a way as to assist management in the creation of policy and in the day-to-day operations of an undertaking. The accounting people are expected to do things that are much more strategic and forward looking. Nowadays managerial accountants serve as internal business consultants involving in all areas of business. They take on leadership roles on their teams and are sought out for the valuable information they provide. The role of the accountants in leading edge companies has been transformed from number cruncher and financial historian to being business partner and trusted advisor. The tools of Management Accounting are financial statement analysis, funds flow analysis, cash flow analysis, budgetary control, Management Reporting system. The Balanced Score Card is a framework for integrating measures derived from strategy. A cost management system is a management planning and control system Value added concept is a performance measure and it reports the wealth generated by a business undertaking over a period of time. 7.14 Terminal Questions 1. Briefly explain the role of Management Accounting. 2. Describe the functions of Management Accounting. 3. Explain the tools of Management Accounting.

4. What is Balanced Score Card? 5. What is Cash Management System? 6. Distinguish between Management Accounting and Financial Accounting 7. What is Valued added concept?

7.15 Answer for SAQs and TQs SAQ 1: 1. Strategic Business Partner 2. Revenues and Costs 3. Competitive Position 16. Management 4. Learning, continuous improvement 17. Future 5. Planning 18. Financial Accounting 6. Upward, downward, lateral 19. Management 7. Operating plans & standards 8. Standard costing 9. Cash budget 10. Funds Flow statement 11. Analysis 12. Budgetary Control
Answer for Terminal Questions:

13. Value added concept 14. Cost Management System 15. Customer, internal business process, learning, growth perspective

1. Refer to unit 7.3 2. Refer to unit 7.5 3. Refer to unit 7.6

4. Refer to unit 7.7 5. Refer to unit 7.8 6. Refer to unit 7.12 7. Refer to unit 7.9. Copyright 2009 SMU Powered by Sikkim Manipal University
.

MB0041-Unit-08-Financial Statement Analysis


Unit-08-Financial Statement Analysis Structure: 8.1 Introduction Objectives 8.2 Meaning of Ratio 8.3 Steps in Ratio Analysis 8.4 Classification of Ratios 8.5 Du Pont Chart

8.6 Solved Problems 8.7 Advantages of Ratio Analysis 8.8 Limitation of Ratio analysis 8.9 Summary 8.10 Terminal Question 8.11 Answers to SAQs and TQs 8.1 Introduction Every fact that is learned becomes a key to other facts E.Y. Youmans. Based on this, this unit deals with analysis of financial statements, the functions of which is to identify and highlight the firms strengths and weaknesses. The objective of ratio analysis is to provide with the financial information necessary to make financial decisions. Objectives: After completing this unit, you should be able to: 1. Explain the meaning, various forms of ratio analysis and its role in comparative analysis 2. Describe the steps involved in ratio analysis 3. Explain the classification of ratios 4. Calculate various ratios from Balance Sheet and Income Statement 5. Derive balance sheet items using various ratios 8.2 Meaning of Ratio Absolute numbers tell very little. Assume that two companies A and B, operating within the same industry submit the information:

Company A NET PROFIT 10000

Company B 100000

One can easily say that Company B makes the most profit. But which company is most profitable? The answer for this will naturally call for further additional information relating to

profit such as size of the company, the total sales it generates or to how much capital is invested in it. Hence, an assessment or a judgment is made based on making some sort of comparison. Extending the example

Company A NET PROFIT SALES 10000 200000

Company B 100000 500000 200000

NET WORTH (CAPITAL RESERVE) 100000

If net profit is compared with Sales, an assessment can be made on which company generates the most net profit per Re.1 received from customers. Return on Capital Employed:

Company A Net Profit / Sales * 100 Net Profit / Net Worth * 100 5% 10 %

Company B 20 % 25%

Ratio can be expressed in the following three forms: 1. As proportion 2. As percentage 3. As turnover rate Simple or pure ratio is merely a quotient arrived by simple division of one number by another. When the current assets of a business firm are Rs. 60,000 and current liabilities is Rs. 15,000 The ratio is derived by dividing Rs. 60,000 by Rs. 15,000. It will be expressed as 4:1 Ratios are expressed as percentage relations when the simple or pure ratios are multiplied by 100. (4 x 100 = 400 %) Ratios are expressed as rates which refer to ratios over a period of time. Example: Stock has turned over 6 times a year

Ratio Analysis is separation or breaking up of anything into its elements or component parts. Ratio analysis is therefore a technique of analysis and interpreting various ratios for helping in making certain decisions. It involves the methods of calculating and interpreting financial ratios to assess the firms performance and status. The ratio analysis is one of the most powerful tools of financial analysis. The analysis is not restricted to any one aspect but takes into account all aspects such as earning capacity of the firm, financial obligation, liquidity and solvency aspects, liquidity and profitability concepts. 8.3 Steps in Ratio Analysis Ratio analysis can provide you with this information in three steps: 1. Calculate the firms ratios for the current or recent period. Ratios are calculated from the firms income statement or balance sheet. It is helpful and sometimes necessary to have the financial statement independently audited. 2. Compare these ratios to those calculated in past records. The purpose of this comparison is to identify tendencies in the firms ratios. This is known as trend analysis. 3. Compare the ratios to industry averages to show how the company compares to firms of the same size in its industry. This process is known as Cross-sectional analysis. Self Assessment Questions: 1. Analysis of performance between two companies can be made using ratios. State true or false. 2. Ratios can be expressed in three forms _____________,____________ and _____________. 3. Stock has turned over 3 times a year the ratio is expressed as ________________ 8.4 Classification of Ratios

A. Liquidity Ratios: It is the ability of a firm to satisfy its short term obligations as they become due for payment. The liquidity is a prerequisite for the very survival of a firm. It reflects the short term financial strength or solvency of the firm. The ratios which indicate the liquidity of the firm are: 1. Net Working Capital 2. Current Ratios

3. Acid test/Quick ratio 4. Super quick ratio 5. Cash flow from operations ratio 1. Net Working Capital: It represents the excess of current assets over current liabilities. Net working capital = Current Assets Current Liabilities Although NWC is really not a ratio, it is frequently employed as a measure of a companys liquidity position. The greater is the amount of NWC, the greater is the liquidity of the firm. Inadequate working capital is the first sign of financial problems for a firm. 2. Current Ratio: It is the ratio of total current assets to total current liabilities.

The Current assets of a firm include cash and bank balances, marketable securities, inventory of raw materials, semi-finished and finished goods, debtors net of provision for bad and doubtful debts, bills receivable and prepaid expenses. The Current liabilities include trade creditors, bills payable, bank credit, provision for taxation dividends payable and outstanding expenses. As a measure of short term financial liquidity, it indicates the rupees of current assets available for each rupee of current liability payable. Higher ratio, i.e., more than 2:1 indicates sound solvency position but at the same time it may be indicative of slack management policies and practices as it might signal excessive inventories or poor credit management. Lower ratio i.e., less than 2:1 indicates inadequate working capital. In capital rich countries, where long-term funds from capital market are available in abundance firms dependence on current liabilities may be less. For public utility companies such as BSNL, MTNL current ratio is usually very low as they required fewer current assets. 3. Quick Ratio: Quick ratio is also known as liquid ratio or acid test ratio. One defect of the current ratio is that it fails to convey any information on the composition of the current assets of the firm. A rupee of cash is considered equivalent to a rupee of inventory or receivable which may not be so. The acid test ratio is a measure of liquidity designed to overcome this defect by measuring those current assets that can be quickly converted into cash to meet the short term obligations of current liabilities. In a way it excludes inventory that are not easily and readily converted into cash.

Acid test ratio of 1:1 is considered satisfactory. This ratio is a more rigorous and penetrating test of the liquidity position of a firm. Higher ratio i.e., more than 1:1 indicates sound financial position. Lower ratio, i.e., less than 1:1 indicates financial difficulty. 4. Super quick / Cash ratio: This ratio is calculated by dividing the super quick assets by the current liabilities of a firm. The super quick current assets are cash and marketable securities. This ratio is the most rigorous and conservative test of a firms liquidity position.

5. Cash flow from Operations Ratio: This ratio measures liquidity of a firm by comparing actual cash flows from operations (in lieu of current and potential cash inflows from current assets) with current liability.

Illustration 1: Given: Current Ratio is 2.5 and working capital is Rs.1, 80,000. Calculate the Current Assets and current liabilities. Solution: Current Ratio = CA / CL 2.5 = CA /1 CA = 2.5 (In the absence of any value, the current liability is always taken as 1 unit) Working Capital = CA CL = 2.5 1 Working Capital = 1.5 For 1.5 WCR = Rs.1,80,000 ( Working Capital value) For 2.5 CAR, = Rs.1,80,000 x 2.5 / 1.5 1. Current Assets = Rs.3,00,000

For 1.0 CLR = Rs. 1,80,000 x 2.5 / 1 2. Current Liabilities = Rs.1,20,000 Illustration 2: Given Current ratio 1.5 :1; Quick ratio 1 : 1 and Current liabilities Rs.50,000. Calculate current assets, quick assets and inventory.

Solution: 1. Current Ratio = 1. 5 : 1 [ CA / CL] Current liabilities = Rs.50,000 Current Ratio (1.5) = CA / 50,000 Current Assets = Rs.75,000 2. Quick Assets ( QR) = QA / 1 [ QA/CL] 1 = QA / 50,000 Quick Assets = Rs.50,000 3. Inventory = CA QA = Rs.75,000 Rs.50,000 Inventory = Rs.25,000 B. Solvency / Capital structure Ratios The long-term lenders/creditors would judge the soundness of a firm on the basis of the longterm financial strength measured in terms of its ability to pay the interest regularly as well as repay the installment of the principal on due dates or in one lump sum at the time of maturity. There are two aspects of the long term solvency of a firm: (i) the ability to repay the principal when due and (ii) regular payment of the interest. Accordingly there are two different but mutually dependent and interrelated types of leverage ratios.

Balance sheet ratios Debt Equity ratio

Capital Structure ratios Interest coverage ratios

Debt- asset ratio

Dividend coverage ratios

Equity- asset/Proprietors fund Total fixed charges coverage ratios ratio Cash flow coverage ratios Debt services coverage ratios 1. Debt Equity Ratio: The relationship between borrowed funds and owners capital is known as debt-equity ratio. This ratio reflects the relative claims of creditors and shareholders against the assets of the firm. Debt-equity ratio is calculated a

The D/E ratio is an important tool to appraise the financial structure of a firm. The ratio reflects the relative contribution of creditors and owners of business in its financing. If D/E ratio is 1:2 it implies that for every rupee of outside liability (debt) the firm has two rupees of owners capital or the stake of the creditors is one-half of the owners. Therefore a safety margin of 66.67 per cent is available to the creditors of the firm. A higher debt-equity ratio say 2:1 implies low safety margin to the creditors. It would lead to inflexibility in the firms operation. Treatment of Preference Share Capital in D/E ratio: The inclusion or exclusion of preference share capital depends upon the purpose for which the D/E ratio is computed. If the objective is to examine the financial solvency of a firm in terms of its ability to avoid financial risk, preference capital should be clubbed with equity capital. On the other hand, if D/E ratio is calculated to show the effect of the use of fixed-interest/dividend sources of funds on the earnings available to the ordinary shareholders, preference capital should be clubbed with debt. Trading on Equity: A high debt-equity ratio denotes the use of larger proportion of debt capital in the financial structure of the firm. The debt capital is cheaper to equity capital because interest on debt is a tax deductible expense. The equity shareholders stands to gain for two reasons (i) Higher returns (ii) Limited stake would be enable them to retain control. Trading on equity or leverage is the use of borrowed funds in expectation of higher returns to equity shareholders. Debt Asset Ratio: It measures the share of total assets financed by outside funds.

A low ratio of debt to total assets is desirable from the point of creditors/lenders as there is sufficient margin of safety available to them.

A high ratio would expose the creditors to high risk. The implications of the ratio of equity capital to total capital are exactly opposite to that of the debt to total assets. A firm should have neither a very high ratio nor a very low ratio. 2. Proprietary Ratio: This ratio indicates the proportion of total assets financed by the owners.

Higher ratio, say more than 75% shows lesser dependence on external sources. Lower ratio, say less than 60% shows more dependence on external sources. 3. Capital Gearing Ratio: It shows the mix of finance employed in the firm.

Important Concepts Equity Capital = Loan Capital = Even Gear Equity Capital > Loan Capital = Low Gear = Over Capitalization Equity Capital < Loan Capital = Higher Gear = Under Capitalization 4. Coverage Ratios: These ratios measure the firms ability to pay certain fixed charges. In the ordinary course of business, the obligations of the creditors are met out of the earnings or operating profits. These claims consist of (i) interest on loans, (ii) preference dividend and (iii) amortization of principal or repayment of the installment of loans or redemption of preference capital on maturity. The important coverage ratios are (i) interest coverage (ii) dividend coverage (iii) total coverage (iv) total cash flow coverage (v) debt service coverage ratio. 5. Interest Coverage: This ratio measures the firms ability to make contractual interest payments.

An interest coverage of five times indicates that a fall in EBIT level to one-fifth of the present level, the operating profits available for servicing the interest on loan would still be equivalent to the claims of the lenders. From the lenders point of view higher the coverage, better is the position of long-term creditors. It also highlights the ability of the firm to raise additional funds in future.

6. Dividend Coverage: It measures the ability of a firm to pay dividend on preference shares which carry a stated rate of return. Higher the coverage better is the position.

7. Debt Service Coverage Ratio: It is considered a more comprehensive and apt measure to compute debt service capacity of the firm. It is the ability of a firm to make the contractual payments required on a scheduled basis over the life of the debt.

The higher the ratio, the better it is. A ratio of less than one may be taken as a sign of long term solvency problem as it indicates that the firm does not generate enough cash internally to service debt. Financial Institutions consider 2:1 as satisfactory ratio. Illustration 3: The Balance Sheet of Dravid Ltd is as follows :

Calculate the Debt Ratio and Debt-equity ratio. Solution: 1. Debt Ratio = Total Liabilities to outsiders / Total assets = (Debentures + Trade creditors )/(Fixed + current assets) = (2,00,000 + 1,00,000) / (10,00,000 + 5,00,000) = 3,00,000 / 15,00,000 = 1:5

2. Debt Equity Ratio = Outsiders funds / shareholders equity or

= = 3,00,000 / 12,00,000 =1:4 C. Profitability Ratio: The management of the firm is interested in the financial soundness of a firm. They are designed to provide answers to questions such as (i) is the profit earned by the firm adequate? (ii) What rate of return does it represent? (iii) What is the rate of profit for various divisions and segments of the firm? (iv)What was the amount paid in dividends? (v) What was the amount paid in dividends? (vi) What is the rate of return to equity-holders? 1. Gross Profit Margin: It measures the percentage of each sales rupee remaining after the firm has paid for its goods. The gross profit margin or gross margin measures the relationship between profit and sales. There are two types of margins- gross profit margin and net profit margin

A high ratio of gross profit to sales is a sign of good management as it implies that the cost of production is relatively low. A relatively low gross margin is definitely a danger signal, a need for careful and detailed analysis of the factors responsible for it. 2. Net Profit Margin: This measures the relationship between net profits and sales of a firm. It measures the percentage of each sales rupee remaining after all costs and expenses including interest and taxes have been deducted.

The net profit margin is indicative of managements ability to operate the business with sufficient success not only to recover all the cost but also to leave a margin of reasonable compensation to the owners. Higher the ratio of net operating profit to sales better is the operational efficiency of the concern.

3. Expenses Ratio: These ratios indicate the relationship of various expenses to net sales. It is computed by dividing expenses by sales. Operating expenses include cost of goods sold, administrative expenses, selling, distribution expense and financial expenses but excludes taxes, dividends and extraordinary losses.

Cost of Goods Sold = Opening Stock + Purchase Closing Stock Operating Expenses = Administrative Expenses + Financial Expenses + Selling Expenses The expenses ratio should be compared over a period of time with the industry average. A low ratio is preferable to high one is unfavorable. For manufacturing concern an operating ratio between 75% and 80% is expected. 4. Return on Capital Employed: It refers to long term funds supplied by the lenders and owners of the firm. The capital employed provides a test of profitability related to the source of long-term funds. A comparison of this ratio with similar firms, with the industry average and over time would provide sufficient insight into how efficiently the long term funds of owners and lenders are being used.

The higher the ratio, the more efficient use of the capital employed and better is the financial position. 5. Return on Shareholders Equity: It measures the return on the total equity funds of ordinary shareholders. This ratio judges whether the firm has earned a satisfactory return for its equity holders or not.

Illustration 4: Ranjandas Ltd provides the following information. Cash Sales Rs.8,00,000; Credit Sales Rs.10,00,000; COGS Rs.15,80,000 and Return Inwards Rs.20,000.

Calculate Gross Profit Ratio and ratio of COGS. Solution Gross Sales = Cash Sales + Credit Sales = 8,00,000 + 10,00,000 = 18,00,000 Net Sales = Gross Sales Return Inwards = 18,00,000 20,000 = 17,80,000 Gross Profit = Net Sales COGS = 17,80,000 15,80,000 = 2,00,000
1. Gross Profit Ratio = (Gross Profit / Net Sales) x 100

= [2,00,000 / 17,80,000] x 100 = 11.2 %


1. Ratio of COGS = 100 GP ratio

= 100 -11.2 = 88.8% D. Activity Ratios or Efficiency Ratios: They are concerned with measuring the efficiency in asset management. The efficiency with which the assets are used would be reflected in the speed and rapidity with which assets are converted into sales. 1. Turnover Ratio: This ratio examines how quickly inventory is converted into cash. This ratio helps the financial manager to evaluate in inventory policy. The ratio reveals the number of times finished stock is turned over during a given accounting period. The three relevant turnover ratios are (i) Inventory turnover ratio (ii) Debtors turnover ratio (iii) Creditors turnover ratio.

To judge whether the ratio of a firm is satisfactory or not, it should be compared over a time on the basis of trend analysis.

2. Debtors Turnover Ratio: It is determined by dividing the net credit sales by average debtors outstanding during the year.

Net Credit sales consist of gross credit sales minus returns from customers. It also includes bills receivables. A high ratio is indicative of shorter time lag between credit sales and cash collection. A low ratio indicates that debts are not being collected rapidly. Debt collection period is calculated by any of the following ratios:

The higher the turnover Ratio and the shorter the average collection period, indicates better trade credit management and the better the liquidity of debtors. 3. Creditors Turnover Ratio: It is the ratio between net credit purchase and the average amount of creditors outstanding during the year.

A higher ratio shows that the creditors are not paid in time. A lower ratio shows that the business is not taking the full advantage of credit period allowed by the creditors. 4. Assets Turnover Ratio: It indicates the efficiency with which firm uses all its assets to generate sales. It is based on the relationship between cost of goods sold and assets of a firm.

The total assets and fixed assets are net of depreciation and the assets are exclusive of fictitious assets. Higher the ratio, greater is the intensive utilization of fixed assets. Lower ratio means under utilization of total and fixed assets. 5. Capital Turnover Ratio:

Lower ratio shows lower profit and higher ratio shows higher profit. Illustration 5: Birla Cements Ltd provides the following Stock: Opening Rs.75,000; Closing Rs.1,00,000. Credit Sales Rs.2,00,000. Cash Sales Rs. 50,000. Gross Profit 25 %. Calculate the Inventory Turnover Ratio Solution: Net Sales = Cash Sales + Credit Sales = 2,00,000 + 50,000 = 2,50,000 Gross Profit = 25% of 2,50,000 ( Net Sales) = 62,500 COGS = Net Sales Gross Profit = 2,50,000 62,500 = 1,87,500 Average Inventory = (Opening + Closing stock) / 2 = (75,000 + 1,00,000)/2

= 87,500 Inventory Turnover Ratio = COGS / Average Inventory = 1,87,500 / 87,500 = 2.14 times

Illustration 6: Total sales of a firm Rs.5,00,000 of which the credit sales are Rs.3,65,000. Sundry Debtors and Bills receivable are Rs.50,000 and Rs.2,000 respectively. Calculate the Debtors Velocity. Solution: Debtors Turnover Ratio = Net Credit Sales / (Debtors + Bills Receivables) = 3,65,000/ (50000 + 2000) = 7.02 Debtors Velocity = No. of days in a year / Debtors turnover ratio (Debtors collection period) = 365/7.02 = 52 days Note: No. of days in a year is taken as 365 days. Illustration 7: Total purchases Rs.1,00,000. Cash purchases Rs.20,000. Discount Provision on creditors Rs.1,000. Purchase returns Rs.2,000. Creditors at close Rs.30,000. Bills payable at close Rs.25,000. Calculate Creditors Velocity. Solution: Credit purchases = Total purchase cash purchase purchase return = 1,00,000 20,000 2,000 = Rs.78,000

Creditors Turnover Ratio = = 78000 / (30000 + 25000) = 1.42

Creditors Velocity = (Crs collection period) = 365 /1.42 = 257 days Note: The Reserve for discount on creditors should not be considered for calculating the net credit sales. E. Integrated Ratios: We have dealt with various ratios and their effect on firms liquidity, solvency, efficiency and profitability independently. However there exist interrelationships among these ratios. The overall profitability of a firm can be accessed on the basis of a combination of earning power and return on assets ratio. The earning power of a firm is the overall profitability of a firm and it is computed by multiplying the net profit margin and the asset turnover. The earning power of a firm is portrayed in Du Pont Chart. Self Assessment Questions: 4. For capital rich countries, the current ratio is usually ________ 5. In quick ratio _________ and ___________ are excluded because they cannot be readily converted into cash. 6. ___________ ratio is the most rigorous and conservative test of all liquidity ratios 7. ___________ ratio reflects the relative contribution of creditors and owners of the business in its financing 8. In debt equity ratio if the objective is to examine the financing solvency of the firm preference share capital is _________. 9. ___________ is the use of borrowed funds is to enhance higher returns to equity shareholders. 10. ___________ is the ability of a firm to make the contractual payments required on a scheduled basis over the life of the debt.

11. A high debtors turnover ratio indicates ____ time lag between credit sales and cash collection. 12. ______________is indicative of managements ability to operate the business with sufficient success not only to recover all the cost but also to leave a margin of reasonable compensation to the owners. 13. _______________ is based on the relationship between cost of goods sold and assets of a firm. 8.5 Du Pont Chart

Return on Investments represents the earning power of the company. It depends on Net profit ratio and capital turnover ratio. A change in any of these ratios will change the firms earning capacity. This chart shows how the return on capital employed is affected by various factors such as cost of goods sold, change in working capital, change in selling and administrative expenses etc. This chart helps the management in detecting the core issues that confront the management and it helps in effective use of capital. 8.6 Solved Problems Problem 1: The income statement of Vignesh Ltd is as follows:

Calculate the Gross Profit Ratio, Net Profit Ratio, Operating Ratio, Operating Profit Ratio and Expense Ratio.

Solution

1. Gross Profit Ratio = Gross Profit / Net Sales x 100 = 2,00,000 / 12,00,000 x 100 = 16.67 % 2. Net Profit ratio = Net Profit after tax / Net Sales x 100 = 80,000 / 12,00,000 x 100 = 6.67% COGS = Sales Gross Profit = 12,00,000 2,00,000 = 10,00,000 3. Operating Ratio = COGS + operating expenses / Net Sales x 100 = 10,00,000+(1,00,000+80,000) / 12,00,000 x 100 = 98.33 % 4. Operating Profit Ratio = 100 98.33% = 1.67 % 5. Expenses Ratio = Operating Expenses/Net sales x 100 = 1,80,000/ 12,00,000 x 100 = 15.00 % Illustration 2: The capital structure of M/s NDW and M/s GDF Ltd are as follows:

Solution: Capital Gearing Ratio = Fixed Income bearing Securities / Total Equity

NDW = 3,00,000/ 12,00,000 = 0.25 GDF = 6,00,000/ 8,00,000 = 0.75 The capital of NDW is low geared when compared to GDF. Illustration 3: The capital structure of Arvind Ltd is as follows: Equity Share Capital 10,00,000 Redeemable Preference Capital 5,00,000 6 % Debentures 3,00,000 Long term liabilities 2,00,000 Reserves and surplus 2,00,000 Calculate the Capital Gearing Ratio and Ratio of Total Investment to Long-term liabilities Solution: Capital Gearing Ratio = Fixed Cost bearing securities / Total Capital = 10,00,000 / 12,00,000 = 0.83 : 1 Total Investment to LTL = Total Liabilities / Long term liabilities = 22,00,000 / 10,00,000 = 2.2 :1 Illustration 4: From the following information provided Sarawath Ltd draw up the Balance Sheet. a. Current Ratio : 2.50 b. Liquidity Ratio : 1.50 c. Net Working Capital : Rs.300000

d. Stock Turnover Ratio : 6 times e. Ratio of Gross Profit to Sales : 20% f. Fixed Asset Turnover Ratio : 2 times g. Average Debt collection period : 2 Months h. Fixed Assets to Net Worth : 0.80 i. Reserve and Surplus to Capital : 0.50

Working Notes

8.7 Advantages of Ratio Analysis The various advantages of ratio analysis are as follows:

a) Financial Forecasting and Planning: Ratio analysis helps in the financial forecasting and planning activities. Ratios based on the past sales are useful in planning the financial position. Based on these future trends are set. b) Decision Making: Ratio analysis throws light on the degree of efficiency. It is also concerned with the management and utilization of the assets. Thus, it enables for making strategic decisions. c) Comparison: With the help of ratio analysis, ideal ratios can be composed. These can be used for comparison in respect of the firms progress and performance, inter-firm comparison with industry average. d) Financial Solvency: It indicates the trends in the financial solvency of the firm. Long term solvency refers to the financial liability of a firm. It can also evaluate the short term liquidity position of the firm. . e) Communication The financial strength and weaknesses of a firm are communicated in a more easy and understandable manner by the use of ratios. The information contained in the financial statements is conveyed in a meaningful manner. It thus helps in the communication and enhances the value of the financial statements. f) Efficiency Evaluation It evaluates the overall efficiency of the business entity. Ratio analysis is an effective instrument which, when properly used, is useful to assess important characteristics of business liquidity, solvency, profitability. A critical study of these aspects may enable conclusions relating to capabilities of business. g) Control It helps in making effective control of the business. Actual results can be compared with the established standard and to take corrective action at the right time. h) Other uses Financial ratios are very helpful in the early and proper diagnosis and financial health of the firm. 8.8 Limitations of Ratio Analysis Undoubtedly, ratios are precious tools in the hands of the analyst. But its significance comes from proper use of these ratios. Misuse or mishandling of these ratios and using them without proper context may lead the analyst or management to a wrong direction. The limiting factors are:

1. The user should possess the practical knowledge about the concerns and the industry in general. 2. Ratios are not an end. They are only means to an end. 3. A single ratio in itself is not important. The trend is more significant in the analysis. Comparison of ratios should be made. 4. For comparative purposes, there should be a standard ratio. There are no such standards prescribed for the ratios. 5. The accuracy and correctness of ratios are totally dependent upon the reliability of the data contained in the financial statement on the basis of which ratios are calculated. 6. To use ratios, first of all there should be uniformity in the accounting plan used by both the firms. In addition. There must be consistency in the preparation of financial statement and recording the transactions from year to year within that concern. 7. Ratios become meaningless if detached from the details from which they are derived. The should be used as supplementary and not substitution of the original absolute figures. 8. Time lag in calculation and communicating the same should not be unnecessarily too much. 9. The method of presentation should be precise and without any ambiguity. 10. Price level changes make the ratio analysis meaningless. 11. Inter-firm comparison should never be undertaken in the case of concerns which are not associated or comparable. 12. All techniques concerning the ratio analysis should be taken into account. 8.9 Summary Ratio Analysis is separation or breaking up of anything into its elements or component parts. Ratio analysis is therefore a technique of analysis and interpreting various ratios for helping in making certain decisions. It involves the methods of calculating and interpreting financial ratios to assess the firms performance and status Ratios are calculated from the firms income statement or balance sheet. It is helpful and sometimes necessary to have the financial statement independently audited. Ratios are compared with past records. The purpose of this comparison is to identify tendencies in the firms ratios. This is known as trend analysis. Ratios are classified into liquidity, solvency, profitability, activity and integrated ratios. 8.10 Terminal questions Problem 1:

Calculate Current ratio, acid test ratio from the following information: Cash in hand Rs.3,000. Cash at Bank Rs.65,000. Bills receivable Rs.10,000. Stock Rs.1,20,000, Debtors Rs.80,000. Prepaid expenses Rs.2,000. Creditors Rs.1,20,000. Bills payable Rs.20,000.
Problem 2:

Calculate Debt equity Ratio and Proprietary ratio from the following information:

Problem 3: The current assets and current liabilities were Rs.16,00,000 and Rs.8,00,000 respectively. What is the effect of each of the following transactions individually and totally on the current ratio: 1. Purchase of new machinery for Rs.5,00,000 2. Purchase of new machinery for Rs.10,00,000 on a medium term loan from a bank with 20 % margin. 3. Payment of a dividend of Rs.2,00,000 of which Rs.0.47 lakh was tax deducted at source. 4. Materials purchased costing Rs.5,00,000 in respect of which bank financed Rs.3,00,000. Problem 4: The current ratio is 2:1. Which of the following suggestions would improve the ratio, which would reduce it and which would not change it? a) to pay a current liability b) to sell a motor car for cash at a slight loss c) to borrow money for short time on an interest bearing promissory note d) to purchase stock for cash e) to give an interest bearing promissory note to a creditor to whom money was to be paid. 8.11 Answer for SAQs and TQs

Activity 1: Solution Debtors Turnover Ratio = Net Credit Sales / (Debtors + Bills Receivables) = 3,65,000/ (50000 + 2000) = 7.02 Debtors Velocity = No. of days in a year / Debtors turnover ratio (Debtors collection period) = 365/7.02 = 52 days Note: No. of days in a year is taken as 365 days. Activity 2: Solution: Credit purchases = Total purchase cash purchase purchase return = 1,00,000 20,000 2,000 = Rs.78,000 Creditors Turnover Ratio = Net credit purchases / (Creditors + Bills Payable) = 78000 / (30000 + 25000) = 1.42 Creditors Velocity = No. of days in a year / Creditors turnover ratio (Creditors collection period) = 365 /1.42 = 257 days

Note: The Reserve for discount on creditors should not be considered for calculating the net credit sales. Copyright 2009 SMU Powered by Sikkim Manipal University

MB0041-Unit-09- Funds Flow Analysis


Unit-09- Funds Flow Analysis Structure: 9.1 Introduction Objective 9.2 Meaning of Funds Flow Statement

9.3 Ascertainment of flow of funds 9.4 Technique of preparing funds flow statement 9.5 Schedule of Changes in Working Capital 9.6 Adjusted Profit and Loss account 9.7 Funds Flow Statement 9.8 Summary 9.9 Terminal questions 9.10 Answers to SAQs and TQs 9.1 Introduction Financial statements as an aid to evaluate past and / or present performance of a business concern is unquestionable and beyond any dispute. The Income Statement reports the revenues earned and expenses incurred or outstanding. The Balance Sheet conveys about the deployment of funds in various assets and equities. International Accounting Standards 7 reads as: A statement of changes in financial position should be included as an integral part of financial statements. The statement of changes in financial position should be presented for each period for which the income statement is prepared. The inclusion of such a statement, therefore, is very helpful to improve the understanding of the operations and activities of an enterprise for the reporting period. Objectives: After studying this unit you should be able to: 1. Explain the meaning and the concepts of funds flow statement. 2. Ascertain transactions that involve flow of funds and those that does not involve flow of funds. 3. Know the techniques of preparing fund flow statement. 9.2 Meaning of Fund Flow Statement Funds Flow Statement is essentially derived from the analysis of changes which have occurred in assets and equities between two accounting periods. Funds Flow Statement being a toll of Management accountant can be prepared at point in time. This statement does not form a part of financial statement but acts as a valuable tool for the management in decision making.

Funds flow statement is very useful in working capital management, deciding the capital structure of the firm, financial planning and forecasting. According to the International Accounting Standard 7, the term Fund refers to cash, to cash and cash equivalent, or to working capital. The term flow refers to change and therefore the term Funds flow refers to change in funds or change in working capital. In other words, any increase or decrease in working capital means flow of funds. Fund Flow Statement enables us to identify and recognize the changes in assets and asset sources which are not readily evident in the income statement or financial statement. It reveals how funds were obtained to pay off its long term debts, how the firm managed to pay regular dividends during volatile period, how the funds from equity issue were utilized etc. However Funds flow statement is not a substitute to comprehensive income statement (Profit and Loss account) and statement of Financial Position (Balance Sheet). It facilitates additional information regarding movement of funds during a particular period. There are two concepts of working capital gross concept and net concept. Gross working capital refers to the firms investment in current assets. Net working capital means excess of current assets over current liabilities.

Current assets are those which are held or receivable within a year or within the operating cycle of the business. They are intended to be converted into cash within a short period of time. Current liability is that obligation which has to be satisfied within a year. Examples of current assets and current liabilities are:

CURRENT ASSETS Cash and bank balances Inventory Sundry Debtors Temporary investments Pre-paid expenses Outstanding incomes

CURRENT LIABILITIES Accounts payable Sundry creditors Bank overdraft Unclaimed dividends Provision for taxation Proposed dividends Short term loans

Accounts receivables Bills receivables Non-Current Assets refers to those assets other than current assets that are realizable in cash or sold or consumable after one year or after a considerable period of time. Fictitious assets are those expenses which could not be written off during the period of their incidence. For example, promotional expenses of a company which could not be treated as expenditure in the year of incidence are shown as fictitious asset. Non-current Liabilities refers to all those obligations other than current liabilities that are likely to mature after one year period. Examples of non-current asset and non-current liabilities are:

NON CURRENT LIABILITIES Share capital Long term loans Debentures

NON CURRENT ASSETS Fixed assets Fictitious assets like goodwill, patents, copyrights, trademarks. Long term investments

Share premium a/c Profit & loss a/c (debit bal) Forfeited shares a/c Profit & loss a/c (credit bal) Appropriation of profits Provision for taxation Provision for depreciation Capital reserve Statement of Sources and Uses of Funds or Funds Flow Statement is a statement which depicts the sources from which funds are obtained and how they have been utilized. When a transaction results in increase of funds it is termed as Source of Fund and when it results in decrease of fund it is termed as Application of Fund. However there are certain transactions that do not result in either increase or decrease of fund. Such transactions are termed as Non fund Transaction. Eg: If the funds are Rs.10000 and a fixed Discount on issue of shares & debentures Deferred expenditures like preliminary expenses, advertising expenses.

asset of Rs.5000 is purchased by issuing shares of Rs.5000 the funds position will not change and therefore this transaction will be taken as a non-fund transaction. There are certain transactions which are not apparent and are hidden. Such transactions have to be located in order to know their effect on the funds. In such circumstances the relevant ledger accounts should be prepared for all non-current assets and liabilities to find out the hidden information. Self Assessment Questions: 5. Flow of Funds refers to change in funds or ________________ 6. ____ Working Capital refers to the firms investment in current assets 7. __________ are those expenses which could not be written off during the period of their incidence 8. When a transaction results in decrease of funds it is termed as ___________________. 9.3 Ascertainment of Flow of Funds The flow of fund can be ascertained from Balance Sheet, Profit and Loss account and from the hidden information. A few independent transactions are given below and the effect of each of the transaction on flow of funds is determined.

Computation of Working capital:

1. The company realizes Rs.20000 from its debtors. Debtors will reduce from Rs.80000 to Rs.60000 Cash balance will increase from Rs. 20000 to Rs.40000

2. The company pays to its creditors a sum of Rs.10000 out of the cash balance. Cash balance gets reduced from Rs.20,000 to Rs.10,000 Sundry creditors will stand reduced from Rs.50,000 to Rs.40,000 3. The company purchases furniture of Rs.10000 by raising long-term loans of Rs.10000.

4. The company redeems preference shares of Rs.100000 by issuing 12% debentures of Rs.100000.

5. The company raises Rs.50,000 in cash by issue of new shares. This transaction will increase the cash balance of the company from Rs.20,000 to Rs.70,000. The working capital position will be:

6. The company sells its building having a book value of Rs.50,000 for a sum of Rs.60,000.

This transaction will increase the cash balance with the company from Rs.20,000 to Rs.80,000.

Note: Each transaction is independent and not linked to the previous example. From the above, the following general rules can be formed: 1. There will be flow of funds if a transaction involves: Current assets and fixed assets, e.g. purchase of building for cash Current assets and capital, e.g., issue of shares for cash Current assets and fixed liabilities, e.g. redemption of debentures in cash Current liabilities and fixed liabilities, e.g. Creditors paid off in debentures Current liabilities and capital, e.g., creditors paid off in shares. Current liabilities and fixed assets, e.g. building transferred to creditors in satisfaction of their claims 2. There will be no flow of funds if a transaction involves. Current assets and current liabilities, e.g., payment made to creditors through cash Fixed assets and fixed liabilities, e.g., building purchased and payments made in debentures. Fixed assets and capital, e.g. building purchased and payment made in shares

Self Assessment Questions: 5. When cash is collected from debtors there is flow of funds. State true or false. 6. When there is sale of fixed assets and cash is obtained there is flow of funds since it involves non-current asset and current asset. State true or false. 7. X Ltd transfers Rs.10 lakhs of its profits to Redemption Reserve account. Does it involve flow of funds? State yes or no 8. Y Ltd writes off goodwill during the current accounting period. This transaction involves flow of funds. State true or false. 9. A firm accepts bills payable drawn by its creditors. Will transaction have effect on flow of funds? Why? 10. Give one transaction which involves one current liability and noncurrent liability. 11. Give one transaction which involves one current liability and noncurrent asset.

9.4 Techniques of Preparing a Funds Flow Statement Like other accounting statements, the structure of Fund Flow Statement is based on the equality of financial assets and liabilities including capital. The basic understanding is that the funds are obtained through profit, external borrowings or by issue of shares. If funds are not available readily from these sources, the other alternative available is to sell the fixed assets and investments. Steps in Preparation of Funds Flow Statement There are three steps involved in the preparation of a Fund Flow Statement (FFS). They are as follows: a) Preparation of Statement of changes in working capital or Schedule of changes in working capital. b) Preparation of Adjusted Profit and Loss Account (APL) c) Statement of changes in Financial position as per AS 7

9.5 Schedule of Changes in Working Capital It is also known as Comparative change in Working Capital Statement or Working Capital Variation Statement. The net change in working capital is projected here in the place of individual changes in all the current assets and current liabilities in the Funds Flow Statement. The statement indicates the amount of working capital at the end of two years. It shows the increase or decrease in the individual items of current assets and current liabilities. The effect of the changes in the individual items of the current assets and current liabilities on working capital is also presented clearly and precisely. The difference in the amount of working capital at the end of two years will depict either the increase or decrease in working capital. While ascertaining the increase or decrease in individual items of current assets and current liabilities and its impact on working capital, the following Rules should be taken into account. Rules for preparing the Schedule of Changes in Working Capital:

Format of Schedule of Changes in Working Capital

*Provision for Taxation: It can be treated in two ways: 1. Treated as current liability: when there is no income tax paid or additional provision made it is treated as current liability. It can be taken to schedule of changes in working capital. No further treatment is required. 2. Treated as non-current liability: A ledger account (Provision for taxation a/c) is prepared. Sometimes we may have to arrive at income tax paid during the year from the given information. These are hidden transactions which are not apparent and are hidden.

*Proposed Dividend: It can be treated in two ways: 1. Treated as current liability: Proposed dividend may be taken as Current liability since declaration of dividends by share holders is simply a formality. It is taken to schedule of changes in working capital with no further treatment. 2. Treated as non-current liability: Proposed dividend can be taken as an appropriation of profit. In such a case, proposed dividend for the current year will be added back to current years profit in order to find out funds from operations if such amount of dividend has already been charged to profit. Payment of dividend will be shown as an application of fund. 9.6 Adjusted Profit and Loss account Revenue transactions such as depreciation, amortization, Profit/Loss on sale of assets etc appearing in Profit and Loss account does not belong to either current or non-current category. All such non-operating incomes and non-operating expenses appear in Adjusted Profit and Loss account to ascertain the Funds from Operations. Funds from Operations: Profit earned by the concern during the current year is deemed to be the source of funds. It is very important source of funds inflow. Net profit is arrived at by deducting cost of goods sold and other expenses from total sales revenue. However, the profit so calculated is seldom equal to the funds from operations because there are many items which are debited or credited in the Profit and Loss Account which do not affect working capital. Therefore, in calculating the funds from operations, the following adjustments must be kept in mind: Items to be added back to Net Profit: a. Non-fund revenue deductions: These are items which are debited to Profit and Loss account. These do not cause outflow of funds such as depreciation and depletion on non-current assets, amortization of fictitious and intangible assets, preliminary expenses, redemption of preference shares or debentures, deferred charges, advertising suspense account written off. If non fund expenditures do not affect the current assets such as unexpired insurance, do not add back. So also, all allowances for income tax payable in future years are excluded. b. Non-trading charges or losses: These items which were debited to Profit and Loss account reduce the profits but they do not cause any outflow of funds. Hence, profit should be corrected by adding back all such charges and losses. These include appropriation of retained earnings such as general reserve, dividend equalization fund, and reserve for contingencies, sinking fund. In addition the dividend on shares must be added back since it is an appropriation and not trading charge. The losses arising out of sale of land, buildings, machinery, long term investments which were written off to the profit and loss account must be added back. Do not add the loss arising out of sale of a current asset such short term investments. It is a trading loss and hence it will not require any adjustment. The amount set aside as provision for current taxation will also be added back. This will be considered only when the provision for taxation is treated as a charge on profits.

Items that are to be deducted from Net Profit: The non fund and non trading revenue receipts or incomes must be deducted from net profit in order to compute funds from operations. The items are: (a) Dividend received or receivable: Although this transaction increases the current assets such as cash and debtors, it is not a trading income. Hence, it should be deducted from the net profits to determine the funds from operations. (b) Retransfer of excess provisions: Where the provisions made for taxation, depreciation, doubtful debts exceed the genuine requirements, the excess amount is transferred back to the Profit and loss account. It does not create any inflow of funds since it is an accounting entry. Hence, deduct it. (c) Profit on sale of non current assets: It is a non trading income. Hence it must be eliminated from the amount of profit. (d) Appreciation in fixed assets: The amount of appreciation on revaluation of fixed assets is normally credited to the profit and loss account. If it is so, deduct it from the profit to compute the funds from operations. Adjusted Profit and Loss a/c

To Depreciation written off Preliminary exp written off Goodwill written off Discount on issue of shares Loss on sale of fixed assets

By Balance b/d (Opening bal ) Profit on sale of investments Profit on sale of Fixed assets Dividend and interest received Funds from Operations (bal fig)

Loss on sale of trade investments Transfer to General Reserve Provision for Tax

Provision for Proposed Dividend Balance c/d (Net Profit) Closing balance Total NOTE: If debit total of Adjusted Profit and Loss a/c is more than the credit total, the difference is Funds generated from Operation If credit total of Adjusted Profit and Loss a/c is more than the debit total, the difference is funds lost in operations. 9.7 Funds Flow Statement This forms the final step in funds flow analysis. It consist of two components the source of funds and the application of funds. This statement reveals the overall creditworthiness of the enterprise. A Funds Flow Statement differs from an Income Statement in the following aspects: 1. Funds flow statement reveals how the funds were obtained and how they were utilized whereas the income statement discloses the results of the business activity. 2. A funds flow statement matches the funds raised with funds utilized 3. Income statement which discloses the results of operations cannot accurately furnish funds from operations because non-fund items such as depreciation, writing off fictitious assets etc are included in it. FORMAT OF FUNDS FLOW STATEMENT Total

SOURCES OF FUNDS Funds from Operations Non-trading incomes Issue of Shares Issue of Debentures

Rs.

Rs.

Borrowing of loans Acceptance of deposits Sale of fixed assets Sale of Investments Decrease in Working Capital APPLICATION OF FUNDS Funds lost in Operations Non-operating expenses Redemption of Preference shares Redemption of debentures Repayment of loans Repayment of deposits Purchase of fixed assets Purchase of long term instruments Increase in Working Capital Illustration1: XYZ Ltd provides the following information

January 1 Sundry Debtors Cash in hand Cash at Bank 65,000 13,000 15,000

December 31 1,05,000 20,000 20,000

Bills Receivable Inventory Bills Payables Outstanding expenses Sundry Creditors Bank Overdraft Short term Loans

16,000 90,000 12,000 6,000 30,000 30,000 32,000

30,000 84,000 8,000 5,000 58,000 42,000 36,000

Prepare a schedule of changes in working capital


Solution

Schedule of changes in Working Capital

llustration 2: The following are the summarized Balance Sheets of Anderson Ltd. BALANCE SHEET AS ON.

Liabilities Share Capital

2006 5,00,000

2007 6,00,000

Asset

2006

2007 11,20,000

Fixed Assets 10,00,000

Reserves

1,50,000

1,80,000 65,000 2,50,000 1,60,000 80,000

Less : Dep Stock Book Debts Cash

(3,70,000) 2,40,000 2,50,000 1,00,000

(4,60,000) 3,70,000 2,30,000 75,000

P & L Account 40,000 Debentures Creditors Prov. for IT 3,00,000 1,70,000 60,000

12,20,000 13,35,000 Prepare a Funds Flow statement Solution:

12,20,000

13,35,000

Notes: 1) The increase in General Reserve is due to transfer a part of profit of the current year and hence the difference is transferred to Adjusted Profit and Loss account since its a non-cash item 2) The difference in depreciation is charged to Adjusted P&L, since its a non-cash item. 3) Increase in Equity Share capital is assumed to be the fresh issue which is a cash item. It is recorded in Funds Flow Statement as source.

4) The difference is debenture is the redemption. It is taken to Funds Flow Statement as application of funds. 5) Purchase of fixed asset is difference between the opening and closing balance of fixed assets. It is application of funds and taken to Funds Flow Statement.

Illustration 3: Following is the Balance Sheet of M/s Srinivas Ltd. You are required to prepare a Fund Flow Statement

Particulars Equity Share capital Profit & Loss

2006 50,000

2007 65,000

Particulars Cash balances

2006 10,000

2007 13,000

14,750

17,000 31,000 15,000 16,500

Debtors Investment Fixed Assets Less: Depreciation Goodwill Stock

25,000 5,000 50,000

27,000 nil 80,000

Trade Creditors 29,000 Mortgage 10,000

Short term loans 15,000

(5,250) (7000)

Accrued expenses8,000

7,500

5,000 37,000 1, 26,750

nil 39,000 1, 52,000

Total

1, 26,750 1, 52,000

Total

Additional Information: 1. Depreciation provided is Rs.1750. 2. Write off goodwill. 3. Dividend paid Rs.3500. Solution:

Illustration 4: Following is the balance sheet of M/s Mahaveer Enterprise for the year 1996 and 1997.

Additional Information: During the year ended 31st December,1998 1. Dividend of Rs.23,000 was paid 2. Assets of another company were purchased for a consideration of Rs.50,000 payable in shares. The assets include Stock Rs.20000, Machinery Rs.25,000

3. Machinery was further purchased for Rs.8000 4. Depreciation written off on machinery Rs.12,000 5. Income tax provided during the year Rs.33,000 6. Machinery worth Rs.2000 was sold for Rs.1800. Loss on sale of machinery Rs.200 was transferred to general reserve. You are required to prepare Schedule of changes in Working Capital and Funds Flow Statement

* Issue of shares which involves current assets alone has to be taken here. Illustration 5: From the following Balance Sheet of M/s. Rao Bros Ltd prepare Statement of Changes in Working Capital and Funds Flow Statement

Additional Information: 1. Preference Shares were redeemed at a premium of 5% during the year 1991.

2. Dividend at 15% on Equity Share for the year 1190 and Preference dividend for 1990 were paid. 3. The Provision for Depreciation stood at Rs.150000 and Rs.190000 for the year 1990 and 1991 respectively. 4. A machine costing Rs.70000, depreciation written off Rs.30000 was disposed off for Rs.25000. Solution:

* Fixed asset should be shown as gross amount. Hence Provision for depreciation is added to Fixed Asset (Net).

Self Assessment Questions: 12. The difference in General Reserve between two accounting period shown in the Balance Sheet is transferred to Adjusted Profit and Loss account since it is a _________ 13. Any increase in Equity Share capital shown in the balance sheet is recorded as _________ in Funds Flow Statement 14. Purchase of Fixed Asset is considered as application of fund. How do you ascertain the amount if provision for depreciation is shown separately? 15. While preparing the schedule of changes in working capital: a. Increase in current asset and decrease in current liability results in ______ in working capital b. Decrease in current asset and increase in current liability results in _________ in working capital. 9.8 Summary Funds flow indicates the inflows and outflows of funds during a particular accounting period generally a year. As such, the term flow in the context of funds indicates the transfer of cash or cash equivalent from asset to equity or one equity to equity or from one asset to another asset.

Statement of Sources and Uses of Funds or Funds Flow Statement is a statement which depicts the sources from which funds are obtained and how they have been utilized. When a transaction results in increase of funds it is termed as source of fund and when it results in decrease of fund it is termed as application of fund. Funds flow statement reveals how the funds were obtained and how they were utilized whereas the income statement discloses the results of the business activity.
9.9 Terminal Question

1. What type of transactions affect flow of funds in funds flow statement? 2. What types of transactions are unaffected in the funds flow? 3. Distinguish between funds flow and income statement 4. Briefly explain the steps involved in preparation of funds flow statement 5. Prepare a statement of changes in working capital from the following information.

Prepare Statement of Changes in Working Capital 9.10 Answer Self Assessment Questions

1. Change in Working Capital 2. Gross 3. Fictitious Assets 4. Application of Funds 5. False

12. Non cash items 13. Source 14. If provision for depreciation is shown separately the difference between closing fixed asset (gross) and opening fixed asset (gross) is taken as additional purchase made during the year.

6. True 7. No 8. False 9. This transaction will not have any effect on flow of funds because it involves only current liability. 10. Creditors paid off by issue of debentures 11. Building transferred to creditors in satisfaction of their claims. Answer for Terminal Questions 1. Refer Unit 9.3 2. Refer Unit 9.3 3. Refer Unit 9.7 4. Refer Unit 9.4

15. (a) increase (b) decrease

5. Statement of changes in working capital during the year 6.

Note: If the investments are in the form of Government Securities, it is treated as current assets.

Copyright 2009 SMU Powered by Sikkim Manipal University


.

MB0041-Unit-10-Cash Flow Analysis


Unit-10-Cash Flow Analysis Structure: 10.1 Introduction Objective 10.2 Meaning of Cash Flow Statement 10.3 Purpose of Cash Flow Statement 10.4 Preparation of Cash Flow Statement

10.5 Format of Cash Flow Statement (AS3: Revised Method) 10.6 Cash Flow from Operating Activities 10.7 Cash Flow Statement under Direct Method 10.8 Different between Cash Flow Analysis and Fund Flow Analysis 10.9 Uses of Cash Flow Statement 10.10 Summary 10.11 Terminal Questions 10.12 Answers to SAQs and TQs 10.1 Introduction The funds flow analysis deal with the flow of funds within and outside the organization. However the main focus of funds flow statement is to explain the changes which have taken place in net working capital during the period under consideration. It fails to explain the changes in cash balance. The movement of cash is of vital importance to the management. The organization may become directionless if the cash inflows are not sufficient to meet the cash outflows. Many a time, a management is posed with the paradox of huge profits and yet impossible to pay dividends or even taxes. This is due to the ground realities that cash is either not received or the cash received is drained out in other items. Hence, it has become a necessity to have a cash flow analysis on periodic intervals say every quarter. The statement shows the items resulting in cash inflows and cash outflows. Objectives: After studying this unit, you should be able to: 1. Explain the meaning of cash flow statement. 2. Describe the broad classification of cash flow statement 3. Recall the revised AS 3 format of cash flow statement 4. Acquaint with steps in preparation of CFS. 5. Compute the Cash Flow Statement 6. Distinguish between Cash Flow Statement and Funds Flow Statement 10.2 Meaning of Cash Flow Statement

Cash flow statement, also known as Statement Accounting for variations in cash, Where Got Where Gone Statement. It shows the movement of cash and their causes during the period under consideration. The statement is significant to the stakeholders of the company and is prepared to show the impact of financial policies and procedures on the cash position. It takes into account all the transactions that have a direct impact upon cash and cash equivalent. 10.3 Purpose of Cash Flow Statement According to Accounting Standard 3, it is mandatory to prepare and present Cash flow statement along with Statement of financial Position and Statement of Income position at the end of accounting period. Unlike Fund Flow statement, cash flow statement explains in depth the inflow and outflow of cash and cash equivalent in three categories viz Net cash flow from operating activities, Net cash flow investment activities and Net cash flow from financing activities. It answers some of the important questions on the company such as: How much cash has been spent on investment activities such as purchase of new plant and machinery, purchase of land? Have long-term source of cash both internally generated plus raised externally adequate to finance purchase of new long term fixed assets? What is the liquidity position of the company? Has it improved? How the company is handling large dividend payment? Is it managing with its reserves or is it borrowing? 10.4 Preparation of Cash Flow Statement According to Accounting Standard 3 (Revised) method cash flow statement is sub divided into three parts (i) cash flow from operating activities (ii) cash flow from investing activities (iii) cash flow from financing activities. 1. Cash flow from Operating Activities: Operating activities are the principal revenue producing activities of the enterprise. Therefore, they generally result from the transactions and other events that enter into the determination of net profit or loss. The amount of cash flows arising from operating activities is a key indicator of the extent to which the operations of the enterprise have generated sufficient cash flows to maintain the operating capability of the enterprise, pay dividends, repay loans and make new investments without recourse to external source of financing. Information about the specific components of future operating cash flows is useful in conjunction with other information in forecasting future operating cash flows. Computation of Operating Profit before Working capital changes:

The Net Profit shown in the Profit and Loss Account will have to be adjusted for non-cash items for find out operating profit before working capital changes. Some if these items are as follows: i. Depreciation: Depreciation does not result in outflow of cash and, therefore, net profit will have to be increased by the amount of depreciation or development rebate charged, in order to find out the real cash generated from operations. ii. Amortization of intangible assets: Goodwill, preliminary expenses, etc., when written off should therefore, be added back to profits to find out the cash from operations. iii. Loss on Sale of fixed assets: It does not result in outflow of cash and, therefore, should be added back of profits. iv. Gains from sale of fixed assets: Since a sale of fixed assets is taken as a separate source of cash, it should be deducted from net profits. v. Creation of reserves: If profit for the year has been arrived at after charging transfers to reserves, such transfers should be added back to profits. If cash operations show a net loss, such net loss after making adjustments for non-cash items will be shown as an application of cash. Thus cash from operations is computed on the pattern of computation of Funds from operations. Cash generated from Operations To find the cash from operations, adjustments will have to be made for changes in current assets and current liabilities arising on account of operations. Any decrease in current assets or any increase in current liabilities between two periods should be added back to Operating profit before working capital changes. Likewise any increase in current assets or any decrease in current liabilities should be deducted from Operating profit before working capital changes to arrive at cash generated from Operations.

Computation of Net Cash Flow from Operating Activities: From cash generated from operations Income tax paid, cash flow from extraordinary items (if any) should be adjusted (subtracted) to arrive at Net cash flow from operating activities. 2. Cash Flow from Investing Activities Transactions like purchase or sale of fixed assets, proceeds from sale of equipments, Interest on Investment received, Dividends received are recorded. 3. Cash Flow from Financing Activities Transactions such as proceeds from issue of shares, debentures, proceeds from long term loans, repayment of long term loans, Interest paid on debentures, dividend payment to equity, preference share holders are shown to arrive at net cash used in financing activities. Purchase of plant and machinery on lease or hire purchase should be shown separately as deferred credit. However the cost of machinery purchased will be shown as application of cash. Computation of Net Increase in Cash and Cash Equivalent The net cash flow from operating, investing and financing activities are added to arrive at net increase in cash and cash equivalent. To this cash and cash equivalent at the beginning of the period is added to get cash and cash equivalent at the end of the period. Self Assessment Questions 1: 7. Preparation and submission of Cash Flow Statement is mandatory according to ______________ 8. Cash Flow Statement has three sub division- _____________ ,________________________ and ______________________. 9. Since depreciation, a component of internal source does not result in outflow of cash; the depreciation amount is ______ to the Net Profit. 10. Give any three internal sources of cash that does not result in outflow of cash. 11. Decrease in liability denotes ____________ of cash 12. Purchase of Plant and Machinery on deferred payment basis is shown separately as source of cash or ________________

10.5 Format of CASH FLOW STATEMENT [AS 3: (Revised) Method]

CASH FLOW STATEMENT (Indirect Method) for the year ending on 1. Cash flow from Operating Activities Net Profit before taxation and extraordinary items: Adjustments for Depreciation Foreign Exchange loss Interest Income Dividend Income Interest expenses Operating Profit before Working Capital changes (+)Decrease/(-)Increase in Sundry Debtors (+)Decrease/(-)Increase in Inventories (-)Decrease /(+) Increase in Sundry Creditors Cash generated from operations Income Tax Paid Cash flow from extraordinary items Proceeds from earthquake disaster settlement Net Cash flow from Operating activities (i)

Cash Flow from Investing Activities Purchase of Fixed Assets

Proceeds from sale of equipment Interest received Dividends received Net cash flow from investing activities (ii)

Cash flow from Financing Activities Proceeds from issuance of share capital Proceeds from long term borrowings Repayment of long term loans Interest Paid Dividends Paid Net cash used in financing activities (iii) Net increase in cash & cash equivalent (i)+(ii)+ (iii) (+) Cash and cash equivalents at the beginning of the period = Cash and cash equivalents at the end of the period The closing balance of Cash and cash equivalent should tally with cash and bank balance of Balance Sheet. Self Assessment Questions: 7. Income Tax paid is ___________activity (operating/investing/financing). 8. Purchase of fixed assets is cash flow from ________ activity ( financing / investing).

9. Repayment of long term loans, dividend paid are _____________ activity (financing / investing). 10. Net Increase in cash and cash equivalent + _______________________ = Cash and cash equivalent at the end of the period 11. Decrease in Sundry Debtors should be _________ to Operating profit before working capital changes 12. Increase in Sundry Creditors should be _________ to Operating profit before working capital changes. Illustration 1: Compute the cash flow from operating activities Profit and Loss Account

To Cost of goods sold 4,00,000

By Sales including cash 5,00,000 sales 1,00,000 Profit on sale of land 30,000 Interest on investment 20,000

Office expenses Selling expenses Depreciation Loss on sale of plant Goodwill written off Income tax Net Profit

12,000 8,000 6,000 4,000 3,000 7,000 1,10,000 5,50,000

5,50,000

Solution

Illustration 2: Calculate cash flow from operating activities if the operating profit before working capital changes Rs.3,80,000

Solution

Illustration 3: The following is the Balance sheet for the period ending 31st March 2006 and 2007. If the Current year net loss is Rs.38,000. Calculate the cash flows

Solution

Illustration 4: Following is the extracts of Balance Sheet in respect of a company.

The current year net loss is Rs. 50,000. Calculate the cash flows. Solution:

Illustration 5: Following is the balance sheet of Amit and bros for the year 2006 and 2007. You are required to prepare Cash Flow Statement BALANCE SHEET as on March 31st 2006 and 2007

Liabilities Share Capital General Reserve Profit and Loss

2006 2,00,000 50,000 30,500

2007 2,50,000 60,000 30,600 1,35,200 35,000 5,10,800

Bank Loan (Long-term) 70,000 Sundry Creditors 1,50,000

Provision for Taxation 30,000 Total Assets Land and Building Machinery Stock 2,00,000 1,50,000 1,00,000 5,30,500

1,90,000 1,69,000 74,000

Sundry Debtors Cash Bank Goodwill Total

80,000 500 5,30,500

64,200 600 8,000 5,000 5,10,800

Additional Information: During the year ended 31st December,2007 1. Dividend of Rs.23,000 was paid 2. Assets of another company were purchased for a consideration of Rs.50,000 payable in shares. The assets include Stock Rs.20000, Machinery Rs.25,000 3. Machinery was further purchased for Rs.8000 4. Depreciation written off on machinery Rs.12,000 5. Income tax provided during the year Rs.33,000 6. Machinery worth Rs.2000 was sold for Rs.1800. Loss on sale of machinery Rs.200 was transferred to general reserve.

Ledger accounts of certain transactions are prepared to find the hidden information. By the virtue of practice certain information can be traced even without preparation of ledger accounts [eg. Bank loan a/c, land and building a/c]

Illustration 6 Following are the summarized balance sheets of Thomson as on 31st December 2005 and 2006.

Adjustments 1. Dividend of Rs. 11,500 was paid 2. Assets of another Company were purchased for a consideration of Rs. 30,000 payable in shares. The following assets were purchased (a) stock Rs. 10,000 (b) Machinery Rs.12,500 3. Machinery was further purchased for Rs. 4,000 4. Income tax paid Rs. 16,500 for the tear 5. Depreciation written-off in Machinery Rs. 6,000 6. Loss on sale machine Rs. 100 was written-off to General Reserve. Prepare Cash flow statement.

Solution:

Working Notes:

Self Assessment Questions:

13. Dividend received is ____________ activity because income is received from investment in shares of another company. 14. Dividend paid is _____________ activity. 10.7 Cash Flow Statement under Direct Method Under this method gross cash receipts and gross cash payments for the major items are disclosed, such as cash receipts from customers and cash paid to suppliers.

Illustration 6: Given below are the Balance Sheet as on March 31, previous year and current year, and a Statement of Income and Reconciliation of earnings for the current year of Electronics Ltd. The only item in the plant and machinery account sold during the year was a specialized machine that originally cost Rs.15 lacs. The accumulated depreciation on this machine at the time of sale was Rs.8 lacs. The machine was sold for Rs.6 lacs and full payment was received in cash. Electronics Ltd. purchased patents for Rs.16 lacs during the year. Besides cash purchases of plant and equipment, the assets of another company were also purchased for Rs.1 crore payable in fully paid-up shares, issued at par; the assets purchased being goodwill, Rs.30 lacs and plant, Rs.70 lacs.

From the foregoing information, prepare a Cash-flow statement both under Direct and Indirect Method for Electronics Ltd. Solution [A]: [Direct method]

N.B: * It may be recalled that cash from operating activities [shown in Section II] was Rs.14; the difference of Rs.14 [Rs.28 as per AS 3 and Rs.14 as per CFS] is due to exclusion of interest payment on debentures [Rs.14]; this interest payment is shown under financing activities.

Working Notes: 1

Solution [B]: [Indirect method]

The Statement highlights that the firm does not have enough funds from its operating activities [Rs.28 lacs] and financing activities [Rs.20 lacs] to cater to investment requirement of Rs.85 lacs, causing decline in cash [Rs.37 lacs]

Problem on Cash Flow And Funds Flow Statement: Illustration 7: The financial position of M/s C and D on Jan 1 and December 31st, 2004 was as follows:

Liabilities Current liability for goods (crs)

1st Jan 36,000

31st Dec 40,600

Asset Cash

1st Jan 4000

31st Dec 3,600

Mrs.As Loan Loan from Bank 30,000

20,000 25,000

Debtors Stock

35,000 25,000

38,000 22,000

Hire Purchase vendor Capital

20,000

Land

20,000

30,000

1,48,000 1,54,000 Building

50,000

55,000 86,000 25,000

Machinery 80,000 Delivery Van Total 2,14,000 2,59,600 Total -

2,14,000

2,59,600

The delivery van was purchased in December 2004 on hire purchase basis; a payment of Rs.5000 was made immediately and the balance of the amount is to be paid in 20 monthly installments of Rs.1000 each together with interest at 12% p.a. During the year the partners withdrew Rs.26,000 for domestic expenditure. The provision for depreciation against machinery on 31st December 2003 was Rs.27,000 and on 31st December 2004 Rs.36,000 You are required to prepare the Funds flow and Cash Flow statement. Observe the difference between funds from operations and cash from operations and give the inference. 1. Preparation of funds flow statement

Activity 1: Prepare Cash flow Statement using the format given below

Working Notes:

10.8 Difference between Cash Flow Analysis and Funds Flow Analysis Following are the points of difference between Cash Flow Analysis and a Funds Flow:

Cash Flow Analysis

Fund Flow Analysis

1. It is concerned only with the 1. Is concerned with change in working change in cash position capital position between; two balance sheet dates. 2. It merely a record of cash receipts and disbursements 2. In Fund Flow statement net effect of receipts and disbursements are recorded. 3. It is concerned with the total provision of funds.

3. It is more useful to the management as a tool of financial analysis in short period. 4. Cash is part of working capital and therefore, an improvement in cash position results in improvement in the funds position

4. An improvement in funds positions need not resulting improvement in cash position

5. An increase in a current 5. An increase in a current liability or liability of decrease in a current decrease in a current asset results asset will result in increase in decrease in working capital cash position 6. It is based on cash basis 6. It is based on accrual basis

7. It is not based on ledger mode7. It is based on ledger principles 10.9 Uses of Cash Flow Statement The cash flow statement, being one of the important financial documents a firm has to possess, reveals the effective uses. First of all, it explains in depth the reasons for the low cash balance available at a particular time. Based on this, it is possible to find the reasons for such a situation. It also shows the major sources and uses of cash. By effectively maintaining the cash and controlling the outflow of cash, it is possible to set in motion the smooth functioning of the organization. It helps the financial decisions more effectively with regard to short term liquidity position of an organization. Projections of cash inflows and outflows can be regulated based on

the records available in the past. Proper projections can be made once the reasons are analyzed. Based on this, it is possible to liquidate the short term obligations without much fun-fare. Short term obligations need to be serviced so that the credit worthiness of an organization can be carried on unabated. 10.10 Summary Cash flow statement, also known as Statement Accounting for variations in cash, Where Got Where Gone Statement. It shows the movement of cash and their causes during the period under consideration. According to Accounting Standard 3, it is mandatory to prepare and present Cash flow statement along with Statement of financial Position and Statement of Income position at the end of accounting period. Operating activities are the principal revenue producing activities of the enterprise. Therefore, they generally result from the transactions and other events that enter into the determination of net profit or loss. 10.11 Terminal Questions 1. What is cash flow statement and how is the cash flow statement subdivided? 2. Bring out the draft format of cash flow statement as per AS3 (revised) method? 3. What is cash flow from operating activities? 4. Bring out the difference between cash flow analysis and funds flow analysis 5. From the following balance sheets of Joy Ltd. Prepare cash flow statement under indirect method.

Additional Information

a) Depreciation of Rs. 10,000 and Rs. 20,000 have been changed on plant and building during the current year. b) An interim dividend of Rs. 20,000 has been paid during the current year. c) Rs. 35,000 was paid during the current year for income tax. 10.12 Answer Self Assessment Questions SAQ 1: 1. AS3 2. a. Net cash flow from operating activities b. Net cash flow from investment activities c. Net cash flow from financing activities 3. Added back 4. Depreciation, Amortization of intangible assets, Gains from sale of fixed assets 5. Outflow 6. Deferred credit 7. Operating 8. Investing 9. Financing 10. Cash and cash equivalent at the beginning of the year 11. Added 12. Added Answer for Terminal Questions 1. Refer to unit 10.4 2. Refer to unit 10.5 3. Refer to unit 10.6 4. Refer to unit 10.8 5. Cash flow from operating activities Rs.1,25,000; Cash flow from investing activities (Rs.1,20,000); Cash flow from financing activities (Rs.12,000).

13. Investment 14. Financing

Activity 1: Solution

Copyright 2009 SMU Powered by Sikkim Manipal University


.

MB0041-Unit-11-Understanding Cost
Unit-11-Understanding Cost Structure: 11.1 Introduction Objectives 11.2 Meaning of Cost 11.3 Objective of Costing 11.4 Methods of Costing

11.5 Technique of Costing 11.6 Classification of Cost 11.7 Elements of Cost 11.8 Statement of Cost Sheet 11.9 Solved Problems 11.10 Summary 11.11 Terminal Questions 11.12 Answers to SAQs and TQs 11.1 Introduction Management accounting is the application of accounting techniques for providing information necessary for decision making. It is designed to help all levels of management in planning and controlling the activities of a business enterprise. The objective of management accounting is to determine product cost, to facilitate planning and control of regular business activities and to supply information for short and long run decisions. Costing is the process of determining the cost of doing something, eg. cost of manufacturing an article, rendering a service or performing a function. Costing includes the techniques and processes of ascertaining cost. In this unit we are dealing with different methods of costing, different techniques of costing and finally classification of costs under various heads. Objectives: After studying this unit, you should be able to: 1. Explain the meaning of cost. 2. Differentiate between cost, expenses and loss 3. Explain the objective and methods of costing 4. Know the techniques of costing 5. Know the various classification of cost 6. Understand the element of cost. 7. Draft the specimen of cost sheet

8. Prepare the statement of cost. 11.2 Meaning of Cost Cost is the amount of resources given up in exchange of some goods and services. The resources are expressed in money or moneys equivalent. CIMA defines the term cost as the amount of expenditure (actual or notional) incurred on or attributable to a given thing. The given thing may be taken as a product, service or any other activity. While the actual expenditure refers to the amount spent, the notional expenditure does not involve in any cash outlay. It does not reflect itself in the accounting records. But, it is important for the purpose of comparison of cost and in decision making. Cost Vs Expenses and Loss An expense is defined as an expired cost resulting from a productive usage of an asset. Unconsumed or unexpired part of the cost is recorded as an asset in the balance sheet.

Loss is defined as reduction in firms equity other than from withdrawals of capital for which no compensating value has been received. 11.3 Objective of Costing It aims to serve the information needs of management for planning, control and decision making. It helps to determine product cost. They are important in inventory valuation, decision regarding pricing of the product. It facilitates planning and control of regular business activities. Different alternative plans are evaluated in terms of respective cost and associated benefits. In control process the data generated can be compared with budgets and estimates. It supplies information for short term and long term decisions. These decisions involve whether to develop new products, when to enter which market etc. 11.4 Methods of Costing Costing refers to the techniques and processes of determining costs of a product manufactured or a service rendered. The method of costing depends on the nature of product, production method and specific business conditions. For example, in cement or steel industry, raw materials passes through different stages (processes) and production is done on continuous bases while in case of construction of a house or contract to build a metro rail / flyover / underpass the job is for a

specific purpose and each job is different from the other. In service industry like hospitality industry, software development, back office processing work etc process costing technique is used.

A. Job Costing: It is used in those business concerns where production is carried out as per specific order and customers specification.

B. Process Costing: This method is used in those industries where manufacture is done continuously thereby it is difficult to trace costs to specific units. The total cost is averaged for the number of units manufactured.

Unit Costing This method is used when a Eg. Sugar industry, Cement, single item is produced and Fertilizer, Chemicals, the final product is Petroleum refining, LPG, composed of homogeneous Paper etc. units. The cost per unit is obtained by dividing the total cost by the total number of units manufactured. Operating Costing This method is used by Eg. Passenger mile, bed in a service industries. The unit hospital, per student in a cost differs among these college. services depending upon the nature of service being rendered. This product costing is used Eg. The professional basket when conversion activities balls are covered with are very similar across genuine leather whereas the

Operation Costing

products lines but the direct scholastic basketballs are materials differ covered with imitation significantly. leather. Cost Centre: Cost centre refers to a section of the business to which costs can be charged. It may be location (a department, sales area), an item of equipment (a machine, a delivery van), a person (a salesman, a machine operator or a group of these). Cost Unit: It is a unit of quantity of product, service or time (or a combination of these), in relation to which cost may be ascertained or expressed.

Self Assessment Questions: 1. Unexpired costs is termed as ___________ 2. Different types of job costing are ____________,___________ and __________ 3. Different types of process costing are _________.________ and ____________ 11.5 Techniques of Costing Techniques refer to principles and rules that are applied for ascertaining cost of the product manufactured or an activity rendered. The following are the costing techniques generally used: 1. Historical Costing: In this system costs are determined after they have been incurred. 2. Standard Costing: In this system, standard costs are ascertained and then compared with the actual cost and the variance if any is determined. Standard costs are predetermined costs in conformity with the most efficient operations or industry benchmark. 3. Absorption or Full Costing: In this method, all costs both fixed and variable are charged to the products, jobs or processes. 4. Variable or Marginal Costing: In this method, only variable costs are charged to the product or jobs. The fixed costs are written off against profits in the period in which they arise. 5. Uniform costing: It is an attempt by several undertakings to use similar costing principles or practices. 11.6 Classification of cost

The elements of costs are classified as materials, labor and expenses. These three elements of cost would be grouped in to direct and indirect categories. Following are the three broad elements of cost Materials Direct (chargeable) and Indirect Labor Direct and Indirect Expenses 1. Direct Material: It refers to the cost of materials which are conveniently and economically traceable to specific units of output. Eg. raw cotton in textiles, crude oil to make diesel, steel to make automobiles, components or parts, primary packing materials, import duties, dock charges, transport of raw materials. 2. Direct Labour: It is defined as the labour of those workers who are engaged in the production process. It is the labour expended directly upon the materials comprising the finished goods. 3. Direct Expenses: These include any expenditure other than direct material and direct labour directly incurred on a specific product or job. Eg. cost of hiring special machinery or plant, cost of special moulds, designs and patterns, experimental cost on models and pilot schemes, fee paid to architects, surveyors, inward carriage and freight on special materials, cost of patent and 4. Factory Overhead: It is defined as the cost of indirect materials, indirect labour and indirect expenses.

Indirect materials: It refers to materials that are needed for the completion of the product but whose consumption with regard to the product is either so small or so complex that it would not be appropriate to treat it as a direct material item. These materials cannot be conveniently assigned to specific physical units. Eg. Lubricants, cotton waste, hand tools, works stationery.

Indirect Labour: Indirect labour includes foremen, shop clerks, general helpers, cleaners, material handlers, plant guards, maintenance men. Indirect expenses: It covers all indirect expenditure incurred from the time production has started to its completion and its transfer to the finished goods store. eg. heat, light, maintenance, factory managers salary. 5. Selling, Distribution and Administration Overhead: Such expenses are generally incurred when the product is in saleable condition. It includes advertising, salesmen salaries, commission, packing, storage, transportation and sales administrative costs. 6. Fixed Cost: It is a cost which does not change in total for a given time period despite wide fluctuation in output or volume of activity. These costs are also known as standby costs, capacity costs or period costs. Eg. Rent, property taxes, supervising salaries, depreciation of office facilities, advertising and insurance.

Example: The production volume and Fixed costs

Total Fixed Cost

Production (in units)

Average Fixed Cost per unit Rs.1000 Rs.500 Rs.200 Rs.100

10,00,000 10,00,000 10,00,000 10,00,000

1000 2,000 5,000 10,000

Fixed costs will not change over a wide range of volume. They will fluctuate before and beyond that range.

Classification of Fixed Cost: Committed cost: Such costs are primarily incurred to maintain the companys facilities and physical existence and over which management has little or no discretion. Depreciation on P&M, taxes, insurance rent etc are the examples.

Managed cost: They relate to the current operations which must continue to be paid to ensure the continued operating existence of the company. eg Staff and management salaries. Discretionary cost: They are also known as programmed cost which is incurred due to special policy decision, mgt program, new research or new system development. Step Cost: This cost is constant for a given amount of output and then increases in a fixed amount at a higher output level. One supervisor is required at a salary of Rs.10000 pm for every 50 workers. The cost of supervisor salary increases to Rs.20000 on employing 51st worker. 7. Variable Cost: Variable costs are costs that vary directly and proportionately with the output. There is a constant ratio between the change in the cost and the change in the level of output. Eg. Direct material and Direct labour and Variable overheads (factory supplies, indirect materials, sales commission, office supplies)If the factory is shut down, variable costs are eliminated.

8. Mixed Cost: are costs made up of fixed and variable element. They are a combination of semi-variable cost and semi-fixed costs. Because of the variable component they fluctuate with volume; because of the fixed component they do not change in direct proportion to output. Semi-fixed cost is those costs which remain constant up to a certain level of output after which they become variable. Semi-variable cost is the cost which is basically variable but whose slope may change abruptly when a certain output level is reached. 9. Product Cost: Are those costs which are identified with the product and included in inventory values. In other words the costs that is included in the cost of manufacturing a product. It is composed of four elements- direct materials, direct labour, direct expenses and manufacturing overhead. 10. Period Cost: are the costs which are not identified with the product or job and are deducted as expenses during the period in which they are incurred. They are not carried forward as a part of value of inventory to the next accounting period.

11. Opportunity cost: it is cost of opportunity lost. It is the cost of selecting one course of action in terms of the opportunities which are given up to carry out that course of action. It is the benefit lost by rejecting the best competing alternative to the one chosen. 12. Sunk Cost: It is the cost that has already been incurred. It is generally unavoidable because these costs cannot be changed once incurred. If the plant has a book value of 10 lakhs and a scrap value of 50000, then the sunk cost is 9.5lakhs. 13. Relevant Cost: are those future costs which differ between alternatives. It may also be defined as the cost which are affected and changed by a decision. They are not historic (sunk) cost and are only incremental (additional) or avoidable cost. 14. Differential cost: It is the difference in total costs between any two alternatives. It is equal to the additional variable expenses incurred in respect of the additional output, plus the increase in the fixed costs if any. 15. Imputed cost: are costs not actually incurred in some transaction but which are relevant to the decision as they pertain to a particular situation. Eg. Interest on internally generated funds, rental value of company owned property and salaries of owners. 16. Out-of-Pocket cost: While imputed costs do not involve cash outlays, out-of-pocket costs signify the cash cost incurred on an activity. This cost concept is significant for management in deciding whether or not a particular project will at least return the cash expenditure associated with the project. 17. Shut Down cost: are those costs which have to be incurred under all situations in the case of stopping manufacture of a product or closing down a department or a division. They are fixed cost. It also refers to minimum fixed cost which is incurred in the event of closure. Self Assessment Questions: 4. Standard costs are predetermined costs which are compared with actual costs and the variance is determined. (state true or false) 5. _______ include costs both fixed and variable are charged to the products, jobs or process 6. Cost of hiring special machines, cost of special moulds are examples of _______ 7. _______ are costs are unavoidable because these costs cannot be changed once incurred. 11.7 Elements of Cost Every product that is manufactured whether it is a pin or a computer needs resources. The management must know the cost of using their resources for its planning and control function. Therefore, the elements of costs are classified as materials, labor and expenses. These three elements of cost would be grouped in to direct and indirect categories

Prime Cost: It is the total of direct materials cost, direct labor cost and chargeable expenses. Factory Cost: It consists of prime cost and factory overheads. Office cost or Cost of Production: It comprises of factory cost and office and administration overheads. Total Cost: By adding selling and distribution expenses to cost of production, one can get the total cost or cost of sales. 11.8 Statement of Cost Sheet Cost sheet is a statement prepared to show the different components of the total cost. It generally shows the total cost and sales as well as cost and selling price per unit. It is generally presented in a tabular form. Specimen of Cost Sheet Cost Sheet for the period __________ Production _____ units

Total Cost Cost per unit Opening Stock of Raw Materials Purchases Carriage Inward (-) Closing Stock of Raw Materials (-) Scrap Direct materials Consumed (I) Direct Wages (II) Direct Expenses (III) I PRIME COST = (I)+ (II)+(III)

(+)

Factory Overheads Indirect Materials Loose Tools Indirect Wages Rent and Rates (factory) Lighting & Heating (factory) Power & Fuel Lubricant Repairs and Maintenance Drawing Office Expenses Cost of research Depreciation of factory plant Works Stationery Welfare service expenses Insurance Fixed assets Insurance Stock and finished goods Works Managers salary

II (+) (-)

FACTORY OR WORKS COST Opening stock of Work-in-progress Closing stock of Work-in-progress

III (+)

COST OF GOODS MANUFACTURED Office and Administrative overhead Rent and Rates (office) Salaries (office) Lighting and Heating Insurance of office building Telephone and Postage Printing and Stationery Depreciation on furniture and fixtures Legal Expenses Audit fee Bank Charges

IV (+) (-) V (+)

COST OF PRODUCTION Opening Stock of Finished Goods Closing Stock of Finished Goods COST OF GOOD SOLD Selling and Distribution Overhead Show room rent and rates Salesmans salary Commission

Travelling expenses of salesman Advertisement Bad debts Carriage outward Samples and other free gifts VI COST OF SALES

VII NET PROFIT (LOSS) VIII SALES Items not included in Cost Sheet: a) Income tax b) Dividends to shareholders i) Commission to managing directors j) Underwriting commission

c) Premium on redemption of shares k) Writing of goodwill, preliminary and debentures expenses d) Capital losses i.e. loss out of sales l) Reserve for bad debts

e) Interest on loan or debentures or m) Transfer to all reserves or bank interest appropriation of profits f) g) Donations Capital expenditure n) Share premium o) Interest on capital p) Drawing of proprietors q) All personal expenses of owner 11.9 Solved Problems

h) Discounts on shares and debentures

Illustration 1: Prepare a cost sheet: Direct materials Rs.2,00,000; Factory expenses Rs.1,20,000; Office expenses Rs.90,000; Total sales Rs.6,50,000; Prime cost Rs.4,10,000; 10 % of the output is in stock. Solution Cost Sheet Direct materials Direct wages (Prime cost minus Direct materials Prime Cost Factory expenses Factory Cost Office expenses Cost of Production Less: closing stock of finished goods 10 % of 6.20,000 Cost of Sales Profit (balancing figure) SALES Illustration 2: The following information is obtained from Alice Ltd. Prepare Cost sheet. 2,00,000 2,10,000 4,10,000 1,20,000 5,30,000 90,000 6,20,000 -62,000 5,58,000 92,000 6,50,000

Stock on Jan 1, 2007 :

Work in progress : 1.1.2007 40,000 31:.12.:2007

64,000.

Raw materials

72,000

Finished goods Purchases of Raw materials Direct wages Works expenses Dividends paid Office expenses Depreciation Selling and Distribution expenses

30, 000 2,40,000

Goodwill written off Stock on 31.12.2007

40,000.

1,36,000. 70,400 40,000 24,000. 10,000 32,000

Raw materials 42,000 Finished goods

42,000 32,000

Sale of finished goods 5,50,000 Payment of sales tax 16,000

Solution

Activity 1 Prepare a cost sheet in the format given below. The following are the details: Raw materials consumed Rs.1,60,000. Direct wages Rs.80,000. Factory overheads Rs.16,000. Office overheads 10% of factory cost. Selling overheads Rs.12,000. Units produced 4,000.Selling price per unit Rs.100.The closing stock is 10% of units produced.

Solution:

Illustration 3: Calculate prime cost, factory cost, cost of production and cost of sales from the following particulars:

Rs Direct materials Direct wages Direct expense Oil and waste

Rs 40,000 10,000 2,000 100 Consumable stores Managers salary Directors fees Office printing & stationery Telephone charges Postage, telegrams Salesmens commission & salary Travelling expenses 700 Advertising Warehouse charges

Rs

Rs 1,000 2,000 500 200

Wages of foremen Storekeepers wages Electric power

1,000 500 200

50 100 500

Lighting Factory Office Rent Factory

500 200 2,000

200 500 200

Office Repairs and Renewals: Factory -plant - machinery - Office premises Depreciation Office Premises

1,000 3,000

Carriage outward

150

500 1,000 200 500 1,700

Plant and machinery 200

700

Solution: Cost Sheet for the period

Total Direct Materials Direct Wages Direct expenses I Prime Cost (+) Factory overhead Oil and waste Wage for foremen Storekeepers wages Electric power Lighting Factory 100 1,000 500 200 500 40,000 10.000 2.000

Cost / unit

52,000

Rent Factory Repairs and Renewals: Factory plant Machinery Depreciation plant and machinery Consumable stores II Factory or Works Cost (+) Office overhead Lighting Office Rent Office Repairs office premises Depreciation office premises Director fees Office printing and stationery Telephone charges Postage and telegram Managers salary III Cost of Production (+) Opening stock of finished goods (-) Closing stock of finished goods

2,000

500 1,000 200 1,000 7,000 59,000

200 1,000 200 500 500 200 50 100 2,000 4,750 63,750 Nil Nil

IV

Cost of Goods Sold Selling Overhead Salesmens commission and salary Travelling expenses Advertising Warehousing charges Carriage outward 500 200 500 200 150 1550 65,300

Cost of sales/ Total Cost

Illustration 4 A factory produces a standard product. The following information is given to you from which you are required to prepare a cost sheet of January 2000.

Rs Raw materials Direct wages Other direct expanse Factory overheads 80% of direct wages Office overheads 10% of works cost Selling and distribution expenses Rs. 2 per unit sold Units produced and sold during the month 10,000 Also find the selling price per unit on the basis that profit mark up is uniformly made to yield a profit of 20% of the selling price. There was no stock or work-in progress either 91,000 29,000 11,000

at the beginning or at the end of the period. Solution: Cost Sheet for the period

Total Direct Materials Direct Wages Direct expenses I Prime Cost (+) Factory overhead 80% of direct wages II Factory or Works Cost (+) Office overhead 10% of work cost office overhead III Cost of Production/Cost of goods sold Selling and distribution 2 units (10,000) IV Cost of Sales Profit 25% on selling price # V Sales 20,000 1,89,620 47405 2,37,025 15,420 1,69,620 23,200 1,54,200 91,000 29,000 11,000 1,31,000

# Working Notes: Profit is 20% of selling price.

If selling price is 100 profit is 20. Therefore cost price is 80. If cost price is 80, profit is 20 or 25% (100 x (20/80) If cost price is Rs.189620, profit is Rs.47405 (189620 x (25/100) Illustration 5 The following data are related to the manufacture of a standard product during the month of July 2009.

Raw Materials consumed Direct Wages Machine hours worked Machine hours rate Administrative overheads Selling overheads Units produced Units Sold

Rs.15,000 Rs. 9,000 900 hours Rs.5 20% of works cost Re.0.50 per unit 17,100 16,000 @ Rs.4 per unit

You are required to prepare a cost sheet from the above showing:
1. The cost per unit 2. The profit per unit sold and profit for the period.

Solution:

Illustration 6: Vijay industries manufactures a product X. On 1st Jan 2007, there were 5000 units of finished product in stock. Work-in-progress Rs. 57,400 Raw materials Rs. 1,16,200 The information available from cost records for the year ended 31st Dec 2007 was as follows: Direct material Direct Labour Freight on R M purchased Indirect labour Other factory overhead Stock on Raw Materials on 31st Dec 2007 Work-in-progress on 31st Dec 2007 Sales (1,50,000 units) Indirect materials 9,06,900 3 ,26,400 55,700 1,21,600 3,17,300 96,400 78,200 30,00,000 2,13,900

There are 15000 units of finished stock in hand on 31st Dec 2007. You are required to prepare:

A statement of cost and profit assuming that opening stock of finished goods is to be valued at the same cost per unit as the finished stock at the end of the period. Solution

Working Notes:

Illustration 7 Prepare the cost sheet to show the total cost of production and cost per unit of goods manufactured by a company for the month of Jan 2005. Also find out the cost of sales.

Rs. Stock of raw materials as on 3,000 1-1-2005 Raw materials purchased 28,000 Office rent

Rs. 500

General expenses (admn) Discount on shares & debentures

400

Stock of raw materials as on 4,500 31-1-2005

300

Manufacturing wages

7,000

Advertisement expenses 600 to be charged fully Income tax paid 2,000

Depreciation on plant Loss on sale of a part of plant Factory rent and rates

1,500 300

3,000

The number of units produced during Jan 2005 was 3,000. The opening and closing stock of finished goods was 200 and 400 units respectively. The value of opening stock of finished goods is Rs.2800. Solution:

Note: Loss on sale of a part of plant is a capital loss hence excluded in cost sheet. So also Discount on Shares & Debentures, and Income tax paid should not be excluded in cost sheet. 11.10 Summary Costing is the process of determining the cost of doing something, eg. cost of manufacturing an article, rendering a service or performing a function. Costing includes the techniques and processes of ascertaining cost. CIMA defines the term cost as the amount of expenditure (actual or notional) incurred on or attributable to a given thing. The given thing may be taken as a product, service or any other activity.

Batch costing is the method used to determine the cost of a group of identical products. The batch consist of similar products is a unit and not a single item within the batch. Contract costing is a method is based on the principle of job costing used by house builders and civil contractors. The contract becomes the cost unit for which relevant costs are determined The elements of costs are classified as materials, labor and expenses. These three elements of cost would be grouped in to direct and indirect categories. 11.11 Terminal Questions Problem 1: The following extract refers to a commodity for the half year ending 31st March 2008. Prepare a cost statement.

Purchase of raw materials

1, 20,000

Direct wages Opening stock

1, 00,000 20,000 16,000

Rent, rate, insurance 40,000 and Works expenses

Raw materials Finished goods (1000 units) Closing stock:

Work in progress: opening closing Carriage inwards Cost of factory

4, 800 16, 000

22, 240 raw material F. Goods (2,000 tons) 1, 440 8,000. Sale of finished goods 3, 00,000

Advertising, discounts allowed and selling costs Re.1 per ton sold. Production during the year is 16,000 tons. Prepare a cost sheet. Problem 2: Calculate the cost of raw materials purchased: Opening stock of raw materials Rs.10,000. Closing stock of raw materials Rs.15,000. Expenses on purchases Rs.5,000. Direct wages Rs.50, 000 Prime cost s Rs.1, 00,000. Problem 3:

The cost data is as follows: Raw materials consumed Rs.1,82,000. Direct wages Rs.40,000. Chargeable expenses Rs.20,000. Opening stock of finished goods (1,000 units) Rs.32,000. Closing stock 2,000 units. Factory overheads 100 % of direct labor. Office overheads 10% of works cost. Selling of Distribution expenses Rs.4 per unit sold. Units produced 10,000. Profit mark-up 20% on selling price. Prepare a cost sheet. 11.12 Answer to SAQs and TQs 1. Assets 2. Batch costing, Contract Costing and Composite costing 3. Unit costing, Operating costing and Operation costing 4. True 5. Absorption costing 6. Direct costing 7. Sunk Cost Answers to Terminal Questions Solution: 1

*To be valued only at number of units sold. Opening stock of finished goods + production minus closing stock = Number of units sold. ** Always to be valued at number of units sold. Number of units sold x Selling price per

Solution: 2
Computation of cost of raw materials purchased

Opening Stock of raw materials Add Purchases Expenses on purchases

10,000 X 5,000 15,000 + X

Less closing stock of raw materials Raw materials consumed Direct wages Prime Cost Prime cost given in the problem is Rs.1,00,000

-15,000 X 50,000 50,000 + X

Hence substituting, 1,00,000 = 50,000 + X; Therefore X = Rs.50,000 Cost of Raw Materials purchased is Rs.50,000 Solution: 3

Activity 1 : Solution

Copyright 2009 SMU Powered by Sikkim Manipal University


.

MB0041-Unit-12-Marginal Costing and Break Even Analysis


Unit-12-Marginal Costing and Break Even Analysis Structure: 12.1 Introduction Objectives 12.2 Concept of Marginal Costing 12.3 Characteristics of Marginal Costing 12.4 Difference between Absorption Costing and Marginal Costing 12.5 Marginal Cost 12.6 Contribution 12.7 Cost Volume Profit (CVP) Analysis 12.8 Break Even Chart

12.9 Break Even Point 12.10 Profit Volume ratio or MCSR 12.11 Target profit 12.12 Margin of Safety 12.13 Application of Marginal cost 12.14 Limitations of Marginal cost 12.15 Solved Problems 12.16 Summary 12.17 Terminal Questions 12.18 Answers to SAQs and TQs 12.1 Introduction In the previous unit we learnt about understanding cost. Costing is the process of determining the cost of doing something. Costing includes the techniques and processes of ascertaining cost. We dealt with different methods of costing, different techniques of costing and finally classification of costs under various heads. Marginal Cost determines the rate of change in costs if the volume of output is increased or decreased by one unit Marginal costing a technique of costing concerned with the changes in costs and profits resulting from changes in the volume of output. Marginal costing is very helpful in decision making and it most widely used profit planning techniques. The cost volume profit analysis shows the relationship among unit sale price, variable cost, sales volume, sales mix and fixed cost. Objectives: After studying this unit, you should be able to: 1. Explain the concept of Marginal costing. 2. Distinguish between Marginal Costing and Absorption Costing 3. Describe contribution and its advantages 4. Familiarize with cost volume profit analysis, break even chart, and break even point.

5. Understand the contribution marginal approach, target profit and margin of safety. 12.2. Concept of Marginal Costing According to Institute of Cost and Management Accounting, London Marginal costing is defined as the ascertainment of marginal cost and of the effect on profit of changes in volume or type of output by differentiating between fixed costs and variable cost. Marginal costing a technique of costing concerned with the changes in costs and profits resulting from changes in the volume of output. Marginal costing is also known as Variable Costing. 12.3 Characteristics of Marginal Costing The technique of marginal costing is based on the distinction between product costs and period costs. Only the variable costs are regarded as the cost of the product while the fixed cost is treated as period costs. The main characteristics of marginal costing are: 1. It is a technique of analysis and determination of costs to help management in decision making 2. All elements of costs are classified into variable and fixed components. Even semi variable costs are classified into variable and fixed components. 3. The variable costs are regarded as the cost of the products 4. Fixed costs are treated as period cost and are charged to profit and loss account for the period for which they are incurred 5. The stock of finished goods and work-in-progress are valued at marginal costs only Variable and Absorption costing are techniques of costing wherein variable costing considers only variable costs for the purpose of product costing, inventory valuation and for other important management decisions. In absorption costing, total costs are taken into consideration for these purposes. Direct material, direct labour and variable overheads constitute the only relevant costs in variable costing whereas the full/absorption costing technique recognizes fixed overheads also as a product costs in addition to material, labour and variable overheads. The two techniques are not mutually exclusive. Full costing is needed while preparing income statements for external reporting and for tax purpose while variable costing is extensively used for internal purpose in decision making. Self Assessment Questions 1. Elements of costs are classified into ______ and ______ 2. Fixed Cost is treated as __________ and charged to P& L account 3. _____________ costs are treated as product cost.

4. The _____________shows the relationship among unit sale price, variable cost, sales volume, sales mix and fixed cost Illustration 1 Hydro Electric Ltd furnishes the following information from its cost records for the first quarter of the current year

Normal production (units) Actual production (units) Actual overheads per quarter at normal production Other expenses per quarter Standard fixed overhead rate per unit Variable costs per unit Sales volume (selling price is Rs. 14)

1,000 1,100 4,000

300 4 6 NIL

Prepare the income statement under absorption and variable costing Solution: Income statement (Absorption Costing)

Particulars Sales revenue Less : Total cost of manufacturing: Variable costs (1,100 x Rs. 6) Fixed overheads (1,100 x Rs. 4)

Amount

Amount Nil

Rs. 6,600 4,400 11,000

Less : Cost of inventory at the end of the year (1,100 x 11,000 Rs.10) Cost of goods manufactured and sold Gross margin (unadjusted) Capacity variance (favorable) (over-absorbed 100 x Rs. 4) Gross margin (adjusted) Less : Other expenses Net income before taxes Income Statement (Variable Costing)

Nil

Nil Rs. 400

400 300 100

Particulars Sales revenue

Amount

Amount Nil

Less : Variable costs (production costs) (1,100 x Rs. Rs. 6,600 6) Less : Cost of inventory at the end to the year (1,100 6,600 x Rs 6) Cost of goods manufactured and sold Contribution Less: Fixed costs: Fixed overheads Other expenses Net income before taxes (loss) 4,000 300 Rs. (4,300) (4,300) Nil Nil

Inference: Under absorption costing, the net income before taxes is Rs.100 while in marginal costing net income before taxes is (Rs.4300) (loss).This significant difference can be attributed to the fact that under absorption costing the fixed manufacturing overheads are included in inventory, whereas in variable costing, inventory carries only variable costs Inventory valuation under absorption costing 11000 Inventory valuation under variable costing 6600 Difference 4400 (This difference is equal to difference between Net income before taxes under absorption costing and variable costing) 12.4 Difference between Absorption Costing and Marginal Costing

Absorption Costing

Marginal Costing

It is known as full costing. Both fixed Only variable costs are included. and variable are included to ascertain Fixed costs are recovered from the cost contribution. Different unit costs are obtained at Marginal cost per unit remain same at different levels of output because of different levels of output because fixed expenses remaining the same variable expenses vary in the same proportion in which output varies Difference between sales and total Difference between sales and cost (marginal cost and fixed cost) is marginal cost is contribution and profit difference between contribution and fixed cost is profit or loss A portion of fixed cost is carried Stock of work-in-progress and forward to the next period because finished goods are valued at marginal closing stock of work-in-progress and cost. Fixed cost of a particular period finished goods is valued at cost of is charged to that very period and is production which is inclusive of fixed not carried over to the next period. cost The apportionment of fixed expenses Only variable cost are charged to on an arbitrary basis gives rise to over products hence marginal costing does or under absorption of overheads not lead to over or under absorption of

fixed overheads. It affects managerial decisions in the It is very helpful in taking managerial areas such as whether to accept the decisions because it takes into export order or not, whether to buy or consideration the additional cost manufacture etc involved only assuming fixed expenses remaining constant. Costs are classified according to functional basis such as production cost, office and administrative cost and selling and distribution costs It fails to establish relationship of cost, volume and profit 12.5 Marginal Cost According to C.I.M.A. London, Marginal Cost means the amount at any given volume of output by which aggregate costs are changed if the volume of output is increased or decreased by one unit. Marginal cost per unit remains unchanged irrespective of the level of activity or output. It is also known as Variable Cost. Marginal cost is the sum total of direct material cost, direct labor cost, variable direct expenses and all variable overheads. The marginal cost of producing a unit declines as output increases. Cost Drivers The activities that cause costs to be incurred are called Cost Drivers. A fixed cost remains unchanged in total as the level of activity (cost drivers) varies. If activity increases or decreases say by 20 %, the total fixed costs remain the same e.g. depreciation, property tax, rent to landlord. But fixed costs per unit will change. A variable cost changes in total in direct proportion to a change in the level of activity or cost driver. If activity increases, say by 20%, total variable cost also increases by 20 %. The total variable cost increases proportionately with activity. Variable cost is fixed per unit but varies in total. 12.6 Contribution It is the difference between sales and variable cost. It may defined as the excess of selling price over variable cost per unit. It is also termed as Contribution Margin or Gross Margin Costs are classified according to the behavior of costs fixed costs and variable costs.

CVP relationship is an integral part of marginal costing.

Advantages of contribution: The concept of contribution is a valuable aid to management in making managerial decisions. It includes 1. It helps the management in the fixation of selling prices 2. It assists in determining the break even point 3. It helps management in the selection of a suitable product mix for profit maximization 4. It helps in choosing from among alternative methods of production. 5. It helps the management in deciding whether to purchase or manufacture, add a new product or not etc. Self Assessment Questions: 5. Fixed cost remains constant _________. 6. Those activities that results in cost are known as ___________. 7. Variable cost is fixed ______ but varies in________. 8. Contribution is also known as contribution margin or _____________. 9. Fixed cost Loss = ______________

12.7 Cost Volume Profit (CVP) Analysis This technique summarizes the effects of changes in an organizations volume of activity on its costs, revenue and profit. CVP analysis can be extended to cover the effects on profit of changes in selling prices, changes in sales volume, changes in product mix etc. It provides management with a comprehensive overview of the effects on revenue and costs of all types of short run financial changes. Since CVP analysis explores the fundamental relationship between costvolume-profit variables it becomes easier to recognize certain level of output or a certain volume of sales that equates cost with revenue. Such level is termed as breakeven point. Break even analysis is an integral part of CVP analysis. 12.8 Break Even Chart It is a graphic or visual presentation of the relationship between costs, volume and profit. It indicates the point of production at which there is neither profit nor loss. It also indicates the estimated profit or loss at different levels of production. While constructing the chart, the following assumption is normally considered.

a) Costs are classified into fixed and variable costs b) Fixed costs shall remain fixed during the relevant volume range of graph. c) Variable cost per unit will remain constant during the relevant volume range of graph d) Selling price per unit will remain constant e) Sales mix remains constant. f) Production and sales volume are equal g) There exists a linear relationship between costs and revenue. h) Linear relationship is indicated by way of straight line. 12.9 Break Even Point BEP is the volume of activity where the organizations revenues and expenses are equal. At a particular amount of sales, the organizations have no profit or loss: it normally breaks even. The general formula for computing the break even sales volume in units is:

Illustration 2: Find the contribution and profit earned if the selling price per unit is Rs.25, variable cost per unit Rs.20 and fixed cost Rs.3,05,000 for the output of 80,000 units. Solution:

Illustration 3: Calculate the profit earned for the data given: Fixed cost Rs.5,00,000; Variable cost Rs.10 per unit; Selling price Rs.15 per unit; Output 150,000 units Solution:

Illustration 4: Find the fixed costs if sales is Rs.2,00,000; Variable Cost Rs. 40,000; and Profit Rs. 30,000. Solution:

12.10 Profit / Volume Ratio or MCSR It expresses the relationship between contribution and sales. It is also termed as Marginal Contribution Sales Ratio (MCSR).

Illustration 5: Calculate MCSR or P / V Ratio if the Marginal cost is Rs.24,000 and Sales is Rs. 60,000 Solution:

Illustration 6: The sales turnover and profit during two periods are as under:

Period 1 Sales Profit Calculate the MCSR. Solution: 20,000 2,000

Period 2 30,000 4,000

Illustration 7 : Calculate MCSR from the following details

Total Sales Year ending 31st December 2006 Year ending 31st December 2007 Solution: 22,23,000 24,51,000

Total Costs 19,83,600 21,43,200

Illustration 8: Calculate the sales if marginal cost is Rs.2,400 & MCSR is 20 %. Solution:

Illustration 9 : Find, Contribution and MCSR. Variable cost per unit Rs.40. Selling price per unit Rs.80. Fixed expenses Rs.2,00,000. Output 10,000 units. Solution:

Illustration 10: Find the Break Even Point.

Selling Price per unit Fixed Cost

Rs.10 60,000

Variable cost per unit Relevant range (units) : Lower limit : Upper limit Break up of variable cost per unit: Direct material Rs.2.00 Direct Labour Rs.1.50 Direct Expenses Re.1.00 Selling expense Re.0.50 Actual sales 18000 units Plant capacity 20,000 units Tax rate 50 per cent

5 6000 20,000

1,80,000 2,00,000

Breakeven point lies at the point of intersection of sales line and total cost line. The vertical distance between the sales revenue and the total cost line measures the estimated net income (after BEP) and the estimated net loss (before BEP) at the related sales volume. The fixed cost line is parallel to the horizontal axis. The variable cost line is superimposed on the fixed cost line and moves upward uniformly with sales volume at the variable cost to volume ratio.

12.11 Target Profit Based on the experiences gained, an organization may intend to increase the production and sales. When an organization was to be on its optimum level, a direction will be provided to achieve the maximum level. In this connection, if one intends to increase the current year production to higher levels, no variable expenses would be incurred. A target net profit or income may be decided in advance. To achieve this profit, efforts will be made to effect sales. The problem of computing the volume of sales required to earn a particular target net profit is very similar to the problem of finding the break even point. After all, the break even point is the number of unit sales required to earn a target net profit of zero. The target net profit is known as desired profit. The formula is:

Illustration 11: Calculate sales in units and in rupees: Units produced 60,000. Fixed Expenses Rs.1,50,000 Selling price per unit Rs.15. Variable cost per unit Rs.10 Profits to be earned Rs.87,500. Solution:

12.12 Margin of Safety

The safety margin of an enterprise is the /difference between the budgeted sales revenue and the break even sales revenue. The safety margin gives management a feel for how close projected operations are to the organizations break even point. The formula is:

Illustration 12: Calculate BEP and MOS: Sales 50,000 units per annum. Selling price Rs.6.00 per unit, Prime cost Rs.3.00 per unit. Variable overheads Re.1.00 per unit. Fixed cost Rs.75,000 per annum. Solution:

12.13 Applications of Marginal Costs The marginal costing helps the management in taking many policy decisions. The vital areas where these concepts are applied directly are as follows: Level of activity planning: Normally, the managements will consider different levels of production or selling activities to decide optimum level of activity. Such periodic exercise shall put the organization in the right tract to achieve its goal. Since the optimum level of activity results in the maximum contribution per unit, the planning can become a perfect execution tool. Alternative methods of production: With the help of marginal costing techniques, its possible to undertake decision about the alternate methods of production. All the decisions should be focused at the greater contribution so that profit can be maintained at a balanced level. Make or buy decision: Depending upon the situational ambience, the management can have a blue print on a vital decision. Management can think of outsourcing the production activities or

to undertake it within its purview. Based on the comparative statement of cost of manufacture with the purchase price, decisions can be taken. Fixation of Selling Price: While pricing a product, the marginal costing techniques can come handy. While fixing a price for a product, it is prudent to take into account the recovery of marginal cost in addition to get a reasonable contribution to cover fixed overheads. Pricing will be at ease once the marginal cost and overall profitability of the concern are known. Selection of optimum sales mix: The product mix plays an important role when a firm produces more than one product. The main focus will on profit maximization. With the help of marginal costing techniques, it is possible to decide the best product mix which will result in maximum profits to the firm. New Product introduction: When a firm intends to diversify its activities or to expand its existing markets, with the help of marginal costing techniques. By fixing the time horizon to recover the fixed costs and profit, decisions can be taken for the introduction of new products. Balancing of profits: As the economic trends gets changed on account of government fiscal policies and regulations, competition at the regional, national, and international levels, marginal costing techniques can aid to bring out facts with regard to maintaining a desired level of profits. Final balancing decisions: If the sales of the product were not encouraging to cover the fixed costs, it is quite natural that the firm may decide about its continuance. This may lead to dovetailing or completely closing down the operations. Marginal costing helps the management to take a sound decision. 12.14 Limitations of Marginal Costing There are certain limitations which can be described as follows: Suitability: The techniques of marginal costing cannot be applied to all the concerns. When a concern needs to carry large stocks by way of work-in-progress, the technique becomes redundant In addition; the marginal costing techniques are not suitable to industries working on contract basis. Inventory valuation difficulties: Since the work in progress and the closing inventories are valued at marginal cost basis, it will not be a sound decision from the Balance Sheet point of view. The main focus on the true and fair value concept gets diluted and the very purpose of exhibiting the financial position will get defeated. Segregation of costs: Though the marginal costing principles call for the differentiation of costs into fixed and variable, in actual practice it becomes difficult to classify them precisely. Many overheads which are appear to be fixed and variable may not exactly align at various levels of production. There is no logical method to segregate semi-variable expenses into fixed and variable.

Time factor: The marginal costing ignores the time factor which is very important in all managerial decisions. Ignoring the time value factor would naturally relate to unreliable and incomplete basis for comparing two alternative jobs. Sales emphasis: Marginal costing principles are basically a sales-oriented concept. While the selling function gets the prominence, other functions are not given equal weight age. This would be a major setback. 12.15 Solved Problems Problem 1: Calculate Break even point. Fixed costs Rs.80,000; Variable cost per unit Rs.4. Sales Rs.2,00,000. The number of units involved coincides with expected volume of output. Each unit sells at Rs.20. Solution: BEP in units = Fixed expenses / contribution per unit Contribution = S V or Rs.20 Rs.4 or Rs.16 = Rs.80,000 / Rs.16 or 5,000 units. Problem 2: Calculate the Break Even point: Sales Rs.2,00,000; Fixed expenses Rs.50,000. Variable expenses. Rs.1,00,000. Solution: Since no information about the number of units produced and costs per unit is given, only Break even point in value can be ascertained. BEP in Rs. = Fixed costs x Total sales / Total Sales Variable costs = 50,000 x 2,00,000 / 2,00,000 1,00,000 = Rs.1,00,000 Problem 3: Calculate MCSR and Break Even Point: Sales Rs.5,00,000. Fixed Costs Rs.1,00,000. Profit Rs.1,50,000. Solution: MCSR = Contribution / Sales Contribution = Fixed Costs + Profit = Rs.1,00,000 + Rs.1,50,000 = Rs.2,50,000 MCSR = 2,50,000 / 5,00,000 = 50%

BEP in Rs. = Fixed Costs / MCSR = 1,00,000 / 50 % or Rs.2,00,000. Problem 4: Find BEP. Variable cost per unit Rs.12. Selling price per unit Rs.20. Fixed expenses Rs.60,000. What will be the selling price per unit if the BEP is brought down to 6000 units? Solution: BEP in units = FC / CPU CPU = S V 20 12 or Rs.8 = 60,000 / 8 or 7,500 units BEP in Rs = 7,500 units x Rs.20 = Rs.1,50,000. The Selling price = FC / CPU or FC / (SP VP) per unit (at BEP 6000 units) = 60,000 / (x 12) 6,000 = 60,000 / x 12 6000 (x 12) = 60,000 X = Rs.22 on simplification. Problem 5: Calculate the following (1) MCSR. (2) Profit when sales are Rs.20,000 (3) New BEP if selling price is reduced by 20 %. Given Fixed expenses Rs.4,000 and Break Even Point Rs.10,000. Solution: (1) BEP sales = FC / MCSR MCSR = FC / BEP Sales = 4,000 / 10,000 x 100

= 40 %. (2) To calculate profit we need to find out contribution. MCSR = Contribution /Sales 40% = Contribution / 20,000 40 % x 20,000 = Contribution Rs.8,000 = Contribution Contribution = Fixed cost + Profit 8,000 = 4,000 + Profit 4,000 = Profit (3) New BEP is Selling Price is reduced by 20 % Let the original Selling Price be Rs.x. Therefore, at 20 % reduction, the reduction would be [0.2x] Hence the revised SP would be x 0.2x or 0.8x Variable cost = 1- Contribution Margin = 1-0.4 = 0.6 New contribution = S V = 0.8x 0.6x = 0.2x. Sales Ratio = 0.2x / 0.8x or 0.25 BEP in volume = 4,000 / 0.25 or Rs.16,000 Problem 6: Given fixed cost is Rs.8,000. Profit earned Rs.2,000 and BEP sales Rs.40,000. Find the actual sales. Solution: MCSR is based on BEP sales BEP sales = FC / MCSR

MCSR = FC / BEP sales or 8,000 / 40,000 = 0.2 Actual sales = FC + desired profit / MCSR = 8,000 + 2,000 /0.2 = Rs. 50,000 12.16 Summary Marginal Cost determines the rate of change in costs if the volume of output is increased or decreased by one unit Marginal costing a technique of costing concerned with the changes in costs and profits resulting from changes in the volume of output. The technique of marginal costing is based on the distinction between product costs and period costs. Only the variable costs are regarded as the cost of the product while the fixed cost is treated as period costs. Break even chart is a graphic or visual presentation of the relationship between costs, volume and profit. It indicates the point of production at which there is neither profit nor loss. It also indicates the estimated profit or loss at different levels of production. CVP analysis provides management with a comprehensive overview of the effects on revenue and costs of all types of short run financial changes. 12.17 Terminal Questions 1. A factory is manufacturing sewing machines. The variable cost of each machine is Rs.200 and each machine is sold for Rs.250. Fixed costs are Rs.12,000. Calculate the BEP for output. 2. Calculate break even point and margin of safety. Fixed cost Rs.1,60,000. Variable cost per unit Rs.2 and Selling price per unit Rs.18. Also compute the margin of safety if the company is earning a profit of Rs.36,000. 3. Calculate the break-even point and turnover required to earn a profit of Rs.3,600. Fixed overheads Rs.1,80,000. Variable cost per unit Rs.2 Selling price Rs.20. If the company is earning a profit of Rs.36,000, express the margin of safety available to it. 4. Given variable cost Rs.6,00,000. Fixed cost Rs.3,00,000. Net profit Rs.1,00,000. Sales Rs.10,00,000. Find (a) MCSR (b) BEP (c) Profit when sales amounted Rs.12,00,000 (d) sales required to earn a profit of Rs.2,00,000. 5. Given: Fixed costs Rs.4,000. Break even sales Rs.20,000. Profit Rs.1,000. Selling price per unit Rs.20. Calculate (a) sales and marginal cost of sales (b) new break even point if selling price is reduced by 10 %.

6. Find the margin of safety if profit is Rs.20,000 and MCSR is 40 %. 7. Calculate Break even sales and margin of safety. Given Sales Rs.10,00,000.Fixed costs Rs.3,00,000 and Profit Rs.2,00,000. 8. Given Sales Rs.20,000. Total Costs Rs.16,000 and Variable Costs Rs.12,000. Compute Break even sales, Margin of safety and sales to earn a profit of Rs.4,000.
12.18 Answer Self Assessment Questions

1. Variable cost , fixed cost 2. Period cost 3. Variable 4. CVP analysis

5. In total 6. Cost drivers 7. Per unit, total 8. Gross margin 9. Contribution

Answer for Terminal Questions

1. Contribution = S V = 250 200 or Rs.50 Therefore BEP = FC / Contribution = 12,000 / 500 or 240 units. 2. Contribution = S V = 18 2 = 16. BEP in units = Fixed costs / contribution per unit = 1,60,000 / 16 = 10,000 units. Margin of safety = Actual Sales (-) Break-even sales Actual sales = Fixed Cost + Desired profit / contribution per unit = 1,60,000 + 36,000 / 16 or 12,250 units. Margin of safety = 12,250 -10,000 units or 2,250 units.

3. Contribution per unit = S V or 20 2 = 18. BEP in Units = FC / CPU = 1,80,000 / 18 = 10,000 units. Break even sales = BEP in units x Selling price = 10,000 x Rs.20 = Rs.2,00,000 Sales required to earn a profit of Rs.36,000 Sales ( required) = Total Fixed overheads + Profit desired] / CPU = 1,80,000 + 36,000 / 18 = 216000/18 =12,000 units Sales ( in Rs) = 12,000 x Rs.20 = Rs.2,40,000. Margin of safety = Actual sales Break even sales = Rs. 2,40,000 Rs.2,00,000 = Rs.40,000 in terms of units = 12,000 10,000 units = 2,000 units. 4. MCSR = Contribution / Sales x 100 = Contribution = S V or Rs.4,00,000. = 4,00,000 / 10,00,000 x 100 or 40% Break even point = FC / MCSR = 3,00,000 / 0.4 = Rs.7,50,000 Profit when sales amounted to Rs.12,00,000. Contribution 40% Therefore total contribution 12,00,000 x 40 % = Rs.4,80,000

Less fixed costs Rs.3,00,000 Profit = Rs.1,80,000 Sales to earn a profit of Rs.2,00,000 = FC + Desired profit / MCSR = 3,00,000 + 2,00,000 / 40 % = Rs.12,50,000. 5. BEP = Fixed costs / MCSR 20,000 = 4,000 / MCSR MCSR = 4,000 / 20,000 x 100 = 20% Contribution = Fixed cost +Profit = 4,000 + 1,000 = Rs.5,000. MCSR = Contribution / Sales x 100 20 = 5,000 / Sales x 100 = Rs. 25,000 Marginal cost of sales = Sales Contribution or 25,000 5,000 = Rs.20,000 If selling price is reduced by 10 % New selling price = 20 2 = Rs.18 Variable cost = Rs.16 (20 20% of 20) Contribution = Rs.2 New MCSR = 2 /18 x 100. New break even sales= FC /SR or 4,000 / 100

= Rs.36,000. 6. Margin of safety = Profit / 40 % = Rs.50,000 7. MCSR = Contribution / sales = 5,00,000 / 10,00,000 = 50 %. Break even sales = Fixed costs / MCSR = 3,00,000 / 50 % = Rs.6,00,000 Margin of safety = Profit / MCSR = 2,00,000 / 50 % = Rs.4,00,000 8. Sales 20,000, Variable cost Rs.13,000, Total Cost Rs.16,000. Therefore, Fixed cost = Total cost variable cost = 16,000 12,000 = Rs.4,000. Profit = Contribution Fixed Cost = 8,000 4,000 = 4,000. MCSR = Contribution / sales = 8,000 / 20,000 = 40 %. BEP in units = FC / MCSR = 4,000 / 40% = 10,000. Margin of safety = Profit / MCSR = 400 / 40 % = 10,000 Sales to earn a profit of Rs.4,000 = FC + Desired profit / MCSR = 4,000 + 4,000 / 40 % = Rs.20,000. Copyright 2009 SMU Powered by Sikkim Manipal University
.

MB0041-Unit-13-Decisions Involving Alternative Choices


Unit-13-Decisions Involving Alternative Choices Structure: 13.1 Introduction Objectives 13.2 Decision Making 13.3 Types of Costs 13.4 Types of Choices Decisions 13.5 Make or Buy Decisions 13.6 Addition / Discontinuance of a Product line 13.7 Sell or Process Further

13.8 Operate or Shut down 13.9 Exploring New Markets 13.10 Maintaining a desired level of profit 13.11 Summary 13.12 Terminal Questions 13.13 Answers to SAQs and TQs 13.1 Introduction In the previous unit we learnt about Marginal Costing. Marginal costing is the ascertainment of marginal cost and of the effect on profit of changes in volume by differentiating between fixed costs and variable costs. Marginal cost is the amount at any given volume of output by which aggregate costs are changed if the volume of output is increased or decreased by one unit. Marginal costing is a very useful tool for management because of its applications. It is used in providing assistance to the management in vital decision-making both short term and long term. Differential analysis is the process of estimating the consequences of alternative actions that a decision maker may take. It is used both for short term and long term decisions. Short term decisions relates to fixing price for the product, selecting a suitable product mix, diversification of the product etc while long term deals with capital budgeting decisions. Objectives After studying this unit, you should be able to: Explain the steps involved in decision making process Know various types of decision choices Analyze and interpret various decision choices 13.2 Decision Making Decision making is the process of evaluating two or more alternatives leading to a final choice known as alternative choice decisions. Decision making is closely associated with planning for the future and is directed towards a specific objective or goal. Decision model contains the following decision-making steps or elements: 1. Identify and define the problem 2. Identify alternative as possible solutions to the problem.

3. Eliminate alternatives that are clearly not feasible 4. Collect relevant data (costs and benefits) associated with each feasible alternative 5. Identify cost and benefits as relevant or irrelevant and eliminate irrelevant costs and benefits from consideration. 6. Identify to the extent possible, non-financial advantage and disadvantage about each feasible alternative. 7. Total the relevant cost and benefits for each alternative 8. Select the alternative with the greatest overall benefits to make a decision 9. Implement or execute the decision 10. Evaluate the results of the decision made. 13.3 Types of Costs A decision involves selecting among various choices. Non routine types of decisions are crucial and critical to the firm as it involves huge investments and involve much uncertainty. Short term decision making is based on relevant data obtained from accounting information. Relevant Cost are costs which would change as a result of the decision. Opportunity costs are monetary benefits foregone for not pursuing the alternative course. When a decision to follow one course of action is made, the opportunity to pursue some other course is foregone. Sunk costs are historical cost that cannot be recovered in a given situation. These costs are irrelevant in decision making. Avoidable costs are costs that can be avoided in future as a result of managerial choice. It is also known as discretionary costs. These costs are relevant in decision making. Incremental / Differential costs are costs that include variable costs and additional fixed costs resulting from a particular decision. They are helpful in finding out the profitability of increased output and give a better measure than the average cost. Self Assessment Questions: 1. Relevant Costs are costs which would _________as a result of the decision. 2. ___________ are historical cost that cannot be recovered in a given situation.

3. Opportunity costs are _________________for not pursuing the alternative course 4. ____________ is also known as discretionary cost. 13.4 Types of Choices Decisions The application of incremental / differential costs and revenues for decision making is known as decision situations or types of choice decisions. Make or Buy decisions Selection of a suitable product mix Effect of change in price Maintaining a desired level of profit Diversification of products Closing down or suspending activities Alternative course of action Own or Lease Retain or Replace Change or Status quo Export or Local sales Expand or Contract Take or Refuse order Place special orders Select sales territories Sell at split-up point or process further. 13.5 Make or Buy Decisions Make or buy decisions arise when a company with unused production capacity consider the following alternatives

a) To buy certain raw materials or subassemblies from outside suppliers b) To use available capacity to produce the items within the company. c) The quality and type of item which affects the production schedule d) The space required for the production of item e) Any transportation involved due to the location of production facility f) Cost of acquiring special know how required for the item. Illustration 1: The Anchor Company Ltd produces most of its electrical parts in its own plant. The company is at present considering the feasibility of buying a part from an outside supplier for Rs. 4.5 per part. If this were done, monthly costs would increase by Rs. 1,000 The part under consideration is manufactured in Department 1 along with numerous other parts. On account of discontinuing the production of this part, Department 1 would have somewhat reduced operations. The average monthly usage production of this part is 20,000 units. The costs of producing this part on per unit basis are as follows.

Material Labour (half-hour) Fixed overheads Total costs Solution

Rs. 1.80 2.40 0.80 5.00

The company should continue the practice of producing the part in Department1. Illustration 2: ABC ltd plans utilize its idle capacity by making components parts instead of buying them from suppliers. The following are the data available for decision to make or buy:

Direct Material Direct Labour Variable manufacturing overhead

Unit cost 12.5 8.0 5.0

The company purchases the part at a unit cost of Rs.30. The company has been operating at 75% of normal capacity. Fixed manufacturing cost is 17 lakhs. The cost to manufacture 50000 units is:

Unit cost Direct material Direct labour Variable manufacturing o/h Total incremental cost Cost to purchase part Net advantage in parts production 12.5 8.0 5.0 25.5 30.0 4.5

Total cost 6,25,000 4,00,000 2,50,000 12,75,000 15,00,000 2,25,000

Inference: The total incremental cost by producing the part in-house is Rs. 25.50 while the cost incurred on purchase of the part from suppliers is Rs. 30.00. There is a clear advantage to the company to produce the part in-house. 13.6 Addition or Discontinuance of a Product line or Process The decision to add or eliminate an unprofitable product is a special case of product profitability evaluation. When a firm is divided into multiple sales outlets, product lines, divisions, departments it may have to evaluate their individual performance to decide whether or not to continue operations of each of these segments. Illustration 3: The Hi-tech Manufacturing Company is presently evaluating two possible processes for the manufacture of a toy, and makes available to you the following information: Process A Particular Process B

Rs. Variable cost per unit Sales price per unit Total fixed costs per year Capacity (in units) 12 20 30,00,000 4,30,000

Rs. 14 20 21,00,000 5,00,000 4,00,000

Anticipated sales (next year, in units) 4,00,000 You are required to suggest:

i) Which process should be chosen? Substantiate your answer. ii) Would you change your answer as given above if you were informed that the capacities of the two processes are as follows: A 6, 00,000 units; B 5, 00,000 units? Why? Substantiate your answer. Solution Comparative Profitability Statement Process A Particular (i) Selling price per unit Variable cot per unit Contribution per unit Rs. 20 12 8 Process B Rs. 20 14 6 24,00,000

Total annual contribution (as per anticipated 32,00,000 sales) Total fixed costs per year Total Income 30,00,000 2,00,000

21,00,000 3,00,000

Process B may be chosen Total contribution (if utilized to present capacity and sold) Less : Fixed costs Total Income Process B may be chosen (ii) Total contribution (if capacity of A of 6,00,000 units and of B 5,00,000 units) Less : Fixed costs Total Income Process A may be chosen. Illustration 4: Addition of second shift Ulfa Ltd produces a single product in its plant. This product sells for Rs. 100 per unit. The standard production cost per unit is as follows: 48,00,000 30,00,000 34,40,000 30,00,000

30,00,000 4,40,000

21,00,000 9,00,000

30,00,000 18,00,000

21,00,000 9,00,000

Raw materials (5 kgs @ Rs. Direct labour (2 hours @ Rs. 5) Variable manufacturing overheads Fixed manufacturing overheads

Rs. 40 10 10 20 80

The plant is currently operating at full capacity of 1, 00,000 units per years on a single shift. This output is inadequate to meet the projected sales manager has estimated that the firm will lose sales of 40,000 units next years if the capacity is not expanded Plant capacity could be doubled by adding a second shift. This would require additional out-ofpocket fixed manufacturing overhead costs of

Rs. 10,00,000 annually. Also, a night work wage premium equal to 25 per cent of the standard wage would have to be paid during the second shift. However, if annual production volume were 1,30,000 units or more, the company could take advantage of 2 per cent quantity discount on its raw material purchases. You are required to advise whether it would be profitable to add the second shift in order to obtain the sales volume of 40,000 units per year? Solution Decision analysis

Particulars Sales revenue Less: variable costs: Raw materials (Rs 39.20 x 1,40,000) Direct labour Variable manufacturing overhead Contribution Less : fixed costs (Rs. 1,00,000 x 20) Net Income

Profit without expansion

Profits with expansion

Rs. 1,00,00,000 Rs. 1,40,00,000

40,00,000 10,00,000 10,00,000 40,00,000 20,00,000 20,00,000

54,88,000 15,00,000 14,00,000 56,12,000 30,00,000 26,12,000

Yes, it would be profitable to add the second shift as it would increase profits by Rs. 6, 12,000. Illustration 5: Assume a company is considering dropping product B from its line because accounting statements shows that product B is being sold at a loss.

Income Statement Product Sales revenue A 50,000 B 7,500 C 12,500 Total 70,000

Cost of sales: D. Material D. Labour 7,500 15,000 1,000 2,000 1,000 1,500 2,500 1,250 10,000 19,500 9,750

Indirect manufacturing cost 7,500 (50% of Direct labour) Total Gross margin On sales Selling & Admn Net income Additional information: 30,000 20,000 12,500 7,500

4,000 3,500 4,500 (1,000)

5,250 7,250 4,000 3,250

39,250 30,750 21,000 9,750

a) Factory Overhead cost is made up of fixed cost of Rs.5850 and variable cost of Rs.3900. b) Variable cost by products are: A Rs 3000, B Rs 400 and C Rs 500 c) Fixed costs and expense will not be changed if product B is eliminated d) Variable selling and administrative expenses are to the extent of Rs.11000 can be traced to the product: A-Rs.7,500; B- Rs.1500 and C- Rs.2000 e) Fixed selling and admn expense are Rs.10000 Solution:

If the sale of product B were discontinued, the marginal contribution would be lost and the net income would be reduced by Rs.2,600. Assume that after dropping product B, the sales of product A increased by 10%. The total profit of the firm will not increase by this sales increase. Product A makes only a marginal contribution of 34% (17000/50000)

Sales revenue of Product A Variable cost of Product A Marginal contribution of Product A

50000 33000 17000

100% 66% 34%

On additional sales of Rs. 5000 the marginal contribution would be Rs.1700

Sales revenue 10% of 50000 Variable cost 66% Marginal contribution (34%)

5000 3300 1700

This contribution is less than Rs. 2,600 now being realized on the sales of product B. it would take additional sales of product A of approximately Rs. 7,647 to equal the marginal contribution of Rs. 2,600 mow being made by product B: = Rs. 7,647 It is possible that dropping product B may result in reduction in some of the fixed costs. Products B now contributes Rs. 2,600 towards recovery of fixed costs and expenses. Only if the fixed costs and expenses can be reduced by more than this amount, it will be advisable to drop product B. 13.7 Sells or Process Further A firm is frequently faced with the problem of continuing with the existing policies or plans or change to new ones. Such change could be in the form of selling a partially processed product (semi finished) or process further. While taking a decision about such matters, the management must keep in mind the long term consequence and the interest of the firm. Illustration 6: A firm sells semi finished product at Rs. 9 per unit. The cost to manufacture the semi finished product is Rs. 6. Further processing can be done at an additional cost of Rs.3 per

unit and the final product can be sold at Rs. 15 per unit. The firm can produce 10,000 units. The analysis is shown below:

Sell Sales revenue (10,000 units) Less : Manufacturing costs Profit Rs. 90,000 60,000 30,000

Process & Sell 1,50,000 90,000 60,000

There is a net advantage of Rs. 30,000 in processing the product further. The market value of the partially processed product (Rs. 90,000) is considered to be opportunity cost of further processing. The figure of net advantage of Rs. 30.000 can be arrived at in the following manner also:

Revenue from sale of final product (10,000 x 15) Less : Additional processing cost (10,000 x 3 ) 30,000 Revenues from sale of intermediate product Net advantage in further processing 13.8 Operate or Shutdown 90,000

Rs. 1,20,000

1,20,000 Rs. 30,000

Various factors both external and internal affect the functioning of the firm. In such situations it becomes necessary for a firm to temporarily suspend or shutdown the activities of a particular product, department or a unit as a whole. Illustration 7: A company operating below 50% of its capacity expects that the volume of sales will drop below the present level of 10,000 units per month. Management is concerned that a further drop in sales volume will create a loss and has under consideration a recommendation that operation be suspended, until better market conditions prevail and also a better selling price. The present operation income statement is as follows:

Rs Sales revenue (10,000 units @ Rs. 3.00)

Rs 30,000

Less : Variable costs @ Rs. 2.00 per unit 20.000 Fixed costs Net Income 10,000 0

The following income statements have been prepared for sales at different capacities:

It would appear that shutdown is desirable when the sale volume drops below 6,000 units per month, the point at which operating losses exceed the shutdown cost. 13.9 Exploring New Markets Decisions regarding entering new markets whether within the country or other the country should be taken after considering the following factors: Whether the firm has surplus capacity to meet the new demand? What price is being offered by the new market? Whether the sale of goods in the new market will affect the present market for the goods? Illustration 8: The following figures are obtained from the budget of a company which is at present working at 90% capacity and producing 13,000 units per annum. 90% Rs. Sales Fixed Expenses Semi- Fixed Expenses 15,00,000 3,00,500 97,500 100% Rs. 16,00,000 3,00,600 1,00,500 1,49,500

Variable Overhead Expenses 1,45,000

Units made

13,500

15,000

Labour and material costs per unit are constant under present conditions. Profit margin is 10 per cent. a) You are required to determine the differential cost of producing 1,500 units by increasing capacity to 100 per cent. b) What would you recommend for an export price for these 1,500 units taking into account that overseas prices are much lower than indigenous prices? Solution

Basic Calculation: Sales at 90% capacity Less: Profit 10% Cost of Goods sold

Rs. 15,00,000 1,50,000 13,50,000

Less : Expenses (Fixed, semi-variable and variable) 5,43,000 Cost of Material and Labour Labour and Material at 100% capacity = 8,07,000 Rs. 8,07,000 x 100/90 = 8,96,667 Differential cost analysis can now be done as follows:

Capacity levels Production (Units) Material and Labour

90% 13,500 8,07,000

100% 15,000 8,96,667 1,49,500

Different cost 1,500 89,667 4,500

Variable overhead expenses 1,45,000

Semi-variable expenses Fixed expenses

97,500 3,00,500 13,50,000

1,00,500 3,00,600 14,47,267

3,000 100 97,267

a) Different Cost = Rs. 97,267 (Rs. 14,47,267 13,50,000)

b) Minimum price for export =

= Rs. 64.84 per unit

At this price, there is no addition to revenue; any price above Rs. 64.84 per unit may be acceptable. Note: It has been presumed that i) No capital investment is necessary ii) No export charges are incurred and iii) The export price will have no effect on the home market where the product will continue to be sold at the old price. It has also been assumed that necessary precaution have been taken to ensure that the product is not dumped back. 13.10 Maintaining a Desired level of profit When deciding between alternative courses of actions the criterion should be to select the project which yields the greatest contribution. Illustration 9: A company is considering expansion. Fixed costs amount to Rs. 4, 20,000 and are expected to increase by Rs. 1, 25,000 when plant expansion is completed. The present plant capacity is 80,000 units a year. Capacity will increase by 50 per cent with the expansion. Variable costs are currently Rs. 6.80 per unit and are expected to go down by Rs. 0.40 per unit with the expansion. The current selling price is Rs. 16 per unit and is expected to remain same under either alternative. What are the break- even points under either alternative? Which alternative is better and why? Solution

The profitability after expansion is very good and hence it is better to expand. Illustration 10: Disposal of inventories ABC Ltd has on hand 5,000 units of a product that cannot be sold through regular sales. These were produced at a total cost of Re. 1, 50,000 and would normally have been sold for Rs. 40 per unit. Three alternatives are being considered. i. Sell the items as scrap for Rs. 2 per unit ii. Repackage at a cost of Rs. 20,000 and sell them at Rs. 8 per unit iii. Dispose them off at the city dump at removal cost of Rs. 500. Which alternative should be accepted? Solution Exhibits the decision analysis

Alternative II should be accepted. 13.11 Summary Decision making is the process of evaluating two or more alternatives leading to a final choice known as alternative choice decisions. Decision making is closely associated with planning for the future and is directed towards a specific objective or goal. A decision involves selecting among various choices. Non routine types of decisions are crucial and critical to the firm as it involves huge investments and involve much uncertainty. Short term decision making is based on relevant data obtained from accounting information.

Relevant Cost are costs which would change as a result of the decision. Opportunity costs are monetary benefits foregone for not pursuing the alternative course. When a decision to follow one course of action is made, the opportunity to pursue some other course is foregone. Sunk costs are historical cost that cannot be recovered in a given situation. These costs are irrelevant in decision making. Avoidable costs are costs that can be avoided in future as a result of managerial choice. It is also known as discretionary costs. These costs are relevant in decision making. Incremental / Differential costs are costs that include variable costs and additional fixed costs resulting from a particular decision. They are helpful in finding out the profitability of increased output and give a better measure than the average cost. 13.12 Terminal Questions 1. Avon garments Ltd manufactures readymade garments and uses its cut-pieces of cloth to manufacture dolls. The following statement of cost has been prepared.

Particulars

Readymade garments Rs. 80,000 13,000

Dolls

Total

Direct material Direct labour

Rs. 6,000 1,200 2,800 3,000 13,000 12,000 (1,000)

Rs. 86,000 14,200 19,800 27,000 1,47,000 1,82,000 35,000

Variable overheads 17,000 Fixed overheads Total cost Sales Profit (loss) 24,000 1,34,000 1,70,000 36,000

The cut-pieces used in dolls have a scrap value of Rs 1,000 if sold in the market. As there is a loss of Rs. 1,000 in the manufacturing of dolls, it is suggested to discontinue their manufacture. Advise the management.

2. The ABC Company Ltd produces most of its own parts and components. The standard wage rate in the parts department is Rs. 3 per hour. Variable manufacturing overheads is applied at a standard rate of Rs. 2 per labour hour and fixed manufacturing overheads are charged at a standard rate of Rs 2.50 per hour. For its current years output, the company will require a new part. This part can be made in the parts department without any expansion of existing facilities. Nevertheless, it would be necessary to increase the cost of product testing and inspection by Rs. 5,000 per month. Estimated labour time for the new part is half an hour per unit. Raw materials cost has been estimated at Rs. 6 per unit. The alternative choice before the company is to purchase part from an outside supplier at Rs 9 per unit. The company has estimated that it will need 2,00,000 new parts during the current years. Advise the company whether it would be more economical to buy or make the new parts. Would your answer be different if the requirement of new parts was only 1,00,000 parts? 13.13 Answers to SAQ and TQs Answer to SAQ 1. Change 2 Sunk cost 3. Monetary benefits foregone 4. Avoidable cost Answers to TQs: 1. Discontinue manufacture of dolls

Readymade garments Total cost Profit (loss) 134000 36000

Dolls 13000 (1000)

Total 147000 35000

2. Decision analysis : 200000 units The company is advised to make the new part. The differential costs favouring the decision of making the component is Rs40000

Decision analysis : 100000 units The company is advised to buy from an outside supplier. Total cost to manufacture 100000 units is Rs.9,10,000. Copyright 2009 SMU

MB0041-Unit-14-Budgetary Control
Unit-14-Budgetary Control Structure: 14.1 Introduction Objectives 14.2 Meaning of a Budget 14.3 Budgetary control 14.4 Objectives of budgetary control 14.5 Merits of budgetary control 14.6 Essential features of Budgetary Control 14.7 Steps in budgetary Control 14.8 Types of Budgets 14.9 Cast Budget 14.10 Flexible Budget 14.11 Limitation of Budget Control 14.12 Summary 14.13 Terminal Questions 14.14 Answers to SAQs and TQs 14.1 Introduction In a competitive environment, the effective operation of a concern resulting into the excess of income over expenditure fully depends upon as to what extent the management follower proper

planning, effective coordination and dynamic control. For all these aspects, it has become necessary that management should plan for the future financial and physical requirements. These are the basic criteria that a firm has to adopt to maintain its profitability and productivity. The procedure for preparing plan in respect of future financial and physical requirements is generally called Budgeting. It is a forward planning exercise. It involves the preparation in advance of the quantitative as well as the financial statements to indicate the intention of the management in respect of the various aspects of the business. In a broader sense, it is essentially an economic service. Budgeting requires a deeper understanding of the economic system of the environment in which the business concern operates. Objectives: After studying this unit, you should be able to: 1. Explain the meaning of budget and budgetary control. 2. Analyze the merits, demerits, essential features of budgetary control. 3. Note the steps involved in the preparation of budgets. 4. Acquaint with various types of budgets. 5. Prepare cash and flexible budgets. 14.2 Meaning of a Budget Cost and Management Accountants, England has defined a budget as a numerical statement expressing the plans, policies and goals of an enterprise for a definite period in the future. Budgets are not actual but are estimated. It is therefore a financial and / or quantitative statement prepared and approved prior to a definite period of time, of the policy to be pursued during that period for the purpose of attaining a given objective. 14.3 Budgetary Control It is applied to a system of management accounting control by which all operations and output are forecasted far ahead as possible and actual results when known are compared with the budget estimates. 14.4 Objectives Budgeting is a forward planning. It basically serves as a tool for management control. The objectives of budgeting may be taken as: To forecast and plan for future to avoid losses and to maximize profits. To help the concern in planning the activities both physical and financial.

To bring about coordination between different functions of the enterprise. To control actual actions by ensuring that actual are in tune with targets. Budgeting and Planning: The planning normally deals with long term and short goals and operations. The goals can be for the entire organization or department-wise or group wise or segment wise to achieve the maximum results and operational efficiency. After setting up objectives in terms of plans, it becomes imperative to organize the factors of production to convert into a reality and workable preposition. In budgeting, planning refers to the preparation of budgets in respect of sales, advertisement, production, inventory, materials cost and requirements, labor cost and requirements, expenses, research, capital expenditures, financial plans. Planning through budgets brings together all segments of the concern in a cooperative way and they are compelled to think seriously about the planning. The views get enlarged than getting into contraction. Internal refinement, broad indexation of activities, concentrated details is the essential features in planning. All the staff must be involved in the planning function to make it more successful and purposeful. Budgeting and Coordination: It deals with the combined efforts of all the people involved from the shop floor to the top management. Individual and collective wisdom should be considered in the preparation of budgets at all levels to make it a workable document for translation into reality. For this adequate communication at all levels should be established. It is very important that each member of management is having perfect and clear cut knowledge. There must be continuity to coordination. Budget may help us to evaluate and examine whether the members of the management are working in a cooperative way or not. Budgeting and control: When one relates control function to budget, we find a system what is generally termed as budgetary control. Control signifies such systematic efforts which help the management to know whether actual performance is in line with predetermined goal, policy and plans. It is basically a measurement tool. Yardsticks should be laid down. Standards must be set up. Therefore, the objectives can be summarized as follows: To conform to good business practice by planning for the future. To coordinate the various divisions of a business. To establish divisional and departmental responsibilities. To forecast operating activities and financial position. To operate most efficiently the divisions, departments and cost center. To avoid waste, to reduce expenses and to obtain the income desired. To obtain more economical use of capital available for the efficient operation.

To provide more definite assurance of earning the proper return on capital employed. To centralize management control. To show the management where action is needed to remedy a situation. To help in controlling cash. To help in obtaining better inventory control and turnover. Self Assessment Questions: 1. Budgets are not actual but ______. 2. After setting up objectives in terms of plans, it becomes imperative to organize the factors of production to convert into a ____________and ______________ 3. _______signifies such systematic efforts which help the management to know whether actual performance is in line with predetermined goal, policy and plans. 4. Internal refinement, broad indexation of activities, concentrated details is the essential features in _______. 14.5 Merits In order to help in planning, coordinating and control, budgets need to be prepared for every organization to get the maximum benefit. Broadly, the merits are as follows: 1. It forces basic policies to initiatives 2. The budgetary control aims at the maximization of profits 3. Budgets fix the goals and targets without which operations lack direction 4. Reduction in cost and elimination of inefficiencies 5. Budgetary control facilitates to make ordered effort and brings about overall efficiency in results. 6. Budgetary control ensures that the capital employed at a particular level is kept at a minimum level 7. Budgetary control enables the management to decentralize responsibility without losing control

8. It is a good guide to the management for making future plans. Based on budgetary control realistic budgets can be drawn. 9. Budgetary control facilitates an intelligent and planned forecast of the future 10. Budgetary control acts as a safety signal for the management. It prevents all types of wastages. 11. Budgetary control brings to light the inefficiencies and weakness on comparing actual performance with budget. Management can take timely remedial measures. 12. Financial crisis can be avoided since budget provides advance information. 13. It is a guide to the management in the field of research and development in future. 14.6 Essential Features of Budgetary Control An effective budgeting system should have essential features to get best results. In this direction, the following may be considered as essential features of an effective budgeting. Business Policies defined: The top management of an organization strives to have an action plan for every activity and for each department. Every budget should reflect the business policies formulated from time to time. The policies should be precise and the same must be clearly defined. No ambiguity should enter the document. Clear knowledge should be provided to all the personnel concerned who are going to execute the policies. Periodic suggestions should be called for. Forecasting: Business forecasts are the foundation of budgets. Time and again discussions should be arranged to derive the most profitable combinations of forecasts. Better results can be anticipated based on the sound forecasts. As far as possible, quantitative techniques should be made use of while forecasting Formation of Budget Committee: A budget committee is a group of representatives of various important departments in an organization. The functions of committee should be specified clearly. The committee plays a vital role in the preparation and execution of budget estimated. It brings coordination among other departments. It aids in the finalization of policies and programs. Non-financial activities are also considered to make it a wholesome affair. Accounting System: To make the budget a successful document, there should be proper flow of accurate and timely information. The accounting adopted by the organization should be proper and must be fine-tuned from time to time Organizational efficiency: To make the budget preparation and its subsequent implementation a success, an efficient, adequate and best organization is necessary a budgeting system should always be supported by a sound organizational structure. There must be a clear cut demarcation

of lines of authority and responsibility. There must also be a delegation of authority from top to bottom line. Management Philosophy: Every management should set a healthy philosophy while opting for the budget. Management must wholeheartedly support the activities which developing a budget. Encouragement should flow from top management. All the members must be involved to make it a workable preposition and a dream-driven document. Reporting system: Proper feedback system should be established. Provision should be made for corrective measures whenever comparative measures are proposed. Availability of statistical information: Since budgets are always prepared and expressed in quantitative terms, it is essential that sufficient and accurate relevant data should be made available to each department. Motivation: Since budget acts as a mirror, the entire organization should become smart in its approach. Every employees both executive and non-executives should be made part of the overall exercise. Employees should be persuaded than pressurized to appreciate the benefits of the budgets so that the fruits can be shared by all the members of the organization. Self Assessment Questions: 5. Budgetary control acts as a ________ for the management. It prevents wastages of all types 6. Business forecasts are the foundation of ________ 7. _______ must wholeheartedly support the activities which developing a budget. 14.7 Steps in Budgetary Control The procedure to be followed in the preparation and control of budget may differ from business to business. But, a general pattern of outline of budget preparation and control may go a long way to achieve the end results. The steps are as follows: Formulation of policies: The business policies are the foundation stone of budget construction. Function policies should be formulated in advance. Long-range policies with short term projections should be made for the functional areas such as sales, production, inventory, cash management, capital expenditure. Preparation of forecasts: Based on the formulated policies, forecast should be made in respect of each function. Activity based concepts should be introduced at the micro level for each function Forecasts should not be considered as a mere estimates. Scientific methods should be adopted for forecasting. Analysis of various factors based on past, and present, future forecast should be made.

Preparation of budgets: Forecasts are converted into written codified document. Such written documents can be used for coordination purposes. Function budgets will act as guidelines for implementation. Forecast combinations: While developing the budgets, through a Master Budget various permutations and combination processes are considered and developed. Based on this, establishment of the most preferred one which will yield optimum benefits should be considered. All the factor components should be identified which are likely to cause disturbances while implementing the budgets 14.8 Types of Budgets The budgets are normally classified according to their nature. They are: (a) fixed budget. (b) Flexible Budget. (c) Functional Budget Fixed Budget: It is also known as static budgets. It is prepared for a fixed or standard volume of activity. They do not change with change in the volume of activity. They are prepared well in advance Due to this, there are bound to be variances at the time of comparison. Hence, the budget targets become unsuitable for the purpose of comparison. Wide deviations are noticed due to changes in the volume of activity. Flexible Budget: It is prepared with a view to take into account the periodic changes in the level of activity attained. In this case, the revenues and costs targets are set in respect of different levels of activity say from zero to 100 % of the production volume. Such mechanism helps to change revenues and cost targets for the actual level of activity and thus makes the comparison more logical and scientific. Functional Budget: These are also known as subsidiary budgets. These are prepared on the basis of approved forecasts for individual department. Since departments are created based on the functions, they are known as functional budgets. The functional budgets may vary in number from business to business. The functional budgets include sales budget, Production budget, selling and distribution overhead budget, plant budget, research and development budget, overheads budget, financial budget such as cash budget and capital expenditure budget. Self Assessment Questions: 8. ______ is also known as static budget because it is prepared for a fixed or standard volume of activity. 9. Types of budgets are _____________. 10. In _______ budget the revenues and costs targets are set in respect of different levels of activity say from zero to hundred percent of the production volume. 14.9 Cash Budget

A proper control over cash is very essential. Cash is an important component in any activity. The control becomes inescapable. If cash is not properly managed or if it is mismanaged, the ultimate result would be disastrous. In many times and in many business situations, business failures are noticed due to the lacunae found in the cash management. Hence cash budgeting occupies a pivotal place in the study of Financial Management. Cash budgeting is the process of forecasting the expected receipts known as cash inflows, and expected payments known as cash outflows to meet the future obligations. The written statement of receipts and payments is known as the cash budget. It is a crystal ball which enables one to observe the future movements in cash position. It is a mere forecast of cash position of an undertaking for a definite period of time. The period may be daily, weekly, monthly, quarterly, semi-annually, or annually. The major two components of cash budget would be forecast first the cash receipts and then second forecasting the cash disbursements. The receipts of cash are formatted as follows: 1. Opening balance of cash in hand and cash at bank 2. Cash sales 3. Collection from debtors to whom sales are effected on credit basis 4. College from Bills received 5. Interest and advances and loans granted 6. Dividends received from investments 7. Sale proceeds from capital assets 8. Proceeds from issue of shares and debentures 9. Other sources. After determining the various sources, the quantum of receipt should be estimated. Past analysis will help to identify the problem areas for effecting collection of cash. Illustration 1: A large retail stores makes 25% of its sales for cash and the balance on 30 days net. Due to faulty collection practice, there have been losses from bad debts to the extent of 1 % of credit sales on average in the past. The experience of the store tells that normally 60 % of credit sales are collected in the month following the sale, 25% in the second following month and 14 % in the third following month. Sales in the preceding three months have been January 2007 Rs.80,000, February Rs.1,00,000 and March Rs.1,40,000. Sales for the next three months are estimated as April Rs.1,50,000, May Rs.1,10,000 and June Rs.1,00,000. Prepare a schedule of projected cash collection.

Solution:

Forecasts of cash payments: The items of expenditures differ from business to business. The normal items which come under the lists are: 1. Cash purchases 2. Payment to creditors or suppliers 3. Payments to Bills payable 4. Payment to employees in the nature of wages, salaries 5. Manufacturing, selling and distribution and administration expenses 6. Repayments of bank load and special obligations such as bonus, donations, advances 7. Interest and dividend payments 8. Capital expenditures for acquiring assets of enduring benefit 9. payment of tax liability 10. other expenses of periodic nature The quantum of amount likely to be spend on the above each item is generally determined with reference to functional budgets of the concerns. The policy of the management will also play a crucial role. It is the policy which determines the ratio of cash purchases and credit purchases. In many cases, the time lag affects the amount of expenditures to be incurred in a particular period. The formula adopted for the expenses payable in next month is : months amount x time lag Illustration 2:

The following are the forecasts relating to wages and factory expenses.

July Wages 32,000

Aug 32,000 5,000

Sept 32,000 5,000

Oct 40,000 5,000

Nov 32,000 5,000

Factory expenses 5,000

One eighth of wages, half of factory expenses are paid in the succeeding month. Estimate the amount of wages and factory expenses payable in September, October and November.
Solution Statement showing the disbursements of cash

Particulars Wages: Aug Sept Oct Nov 32,000 32,000 40,000 32,000

Sept 4,000 28,000 32,000

Oct 4,000 35,000 39,000

Nov 5,000 28,000 33,000

Factory expenses Aug Sept Oct Nov 5,000 5,000 5,000 5,000 2,500 2,500 5,000 Illustration 3: 2,500 2,500 5,000 2,500 2,500 5,000

The following information is provided in respect of Rashmi Ltd. Prepare a Cash Budget for April, May and June 2007.

Months Jan Feb March April May June

Details Actual Actual Actual Budget Budget Budget

Sales 80,000 80,000 75,000 90,000 85,000 80,000

Purchases Wages 45,000 40,000 42,000 50,000 45,000 35,000 20,000 18,000 22,000 24,000 20,000 18,000

Expenses 5,000 6,000 6,000 7,000 6,000 5,000

Additional information: a. 10% of the purchases and 20% of sales are in cash b. The average collection period of the company is 1/2 month and the credit purchases are paid regularly after one month. c. Wages are paid half monthly and the rent of Rs.500 included in expenses is paid monthly. Other expenses are paid after one month lag. d. Cash balance on April 1, 2007 may be assumed to be Rs.15,000. Solution:

Illustration 4:

Hindustan Ltd. is to start production on January 1, 2008. The prime cost of a unit is expected to be Rs.40 (Rs.16 per material and Rs.24 for labor). In addition, variable expenses per unit are expected to be Rs.8 and fixed expenses per month Rs.30,000. Payment for materials is to be made in the month following the purchases. One-third of sales will be for cash and the rest on credit for settlement in the following month. Expenses are payable in the month in which they are incurred. The selling price is fixed at Rs.80 per unit. The number of units to be produced and sold are expected to be : January 900, February 1,200, March 1,800, April 2,000, May 2,100 and June 2,400. Draw a cash budget indicating cash requirements. Solution

Illustration 5: Ranjini Ltd. intends to approach her Bankers for temporary overdraft facility for three months from 1st June to 31st August, 2007. Prepare a Cash budget for the above period.

Months April May June July Aug

Sales 3,60,000 3,84,000 2,16,000 3,48,000 2,52,000

Purchases 2,49,600 2,88,000 4,86,000 4,92,000 5,36,000

Wages 24,000 28,000 22,000 20,000 30,000

(a) The entire sale is on credit basis out of which 50% is realized in succeeding month and balance in the second month following sales. (b) Creditors are paid in the month following purchase. (c) Estimated cash as on 1st June is Rs.50,000 Solution

Illustration 6: Prepare a cash budget from January to April.

Expected Purchases Jan Feb Mar April 48,000 80,000 81,000 90,000

Expected Sales 60,000 40,000 45,000 40,000

Wages paid Rs.5,000 per month. Cash balance on 1st January Rs.8,000. Management decides that: a) In case of deficit upto of Rs.10,000, arrangement can be made with the bank. b) In case of deficit exceeding Rs.10,000 but within Rs.42,000, debentures to be issued. c) In case of deficit exceeding Rs.42,000, equity shares to be issued. Solution CASH BUDGET

Particulars RECEIPTS Opening balance Cash sales Total, say A PAYMENTS Purchases Wages Total, say B Closing Balance A B

Jan

Feb

March

April

8,000 60,000 68,000

15,000 40,000 55,000

(30,000) 45,000 15,000

(71,000) 40,000 (31,000)

48,000 5,000 53,000 15,000

80,000 5,000 85,000 (30,000)

81,000 5,000 86,000 (71,000)

90,000 5,000 95,000 (1,26,000)

The total deficit of Rs.1,26,000 should be raised from the issue of Equity Shares. 14.10 Flexible Budget According to I.C.M.A., London, a flexible budget is a budget which is designed to change in accordance with the level of activity actually attained. The basic idea of a flexible budget is that there shall be some standard of cost and expenditures. Thus, a budget prepared in a manner to give budgeted costs for any level of activity is, known as flexible budget. Such budget is prepared after considering the variable and fixed elements of costs and the changes which may be expected for each item at various levels of operations. The main focus of flexible budget is to re cognize the difference in behavior pattern of fixed and variable costs in relation to fluctuations in production and sales. The flexible budget is, hence, designed to change appropriately with such fluctuations. In flexible budget, data relating to costs and expenses may progressively be changed in any month in accordance with actual output achieved. Costs and estimates are made in advance based on standards. A maximum and a minimum level of operation are made. Comparison of budgeted with actual are made. Budgeted activities are taken as basis.

The principles of flexible budgeting concepts are applied to functional budget, master budgets. Popularly, the flexible budget is adopted for production cost budget. A detailed classification is adopted such as variable, fixed and semi-variables. Adopting micro-level classifications, it is intended to pin-point the various effects on each class of overheads. Illustration 7: Draw a flexible budget for the level of operation at 70 %, 80 % and 90 %. Variable overheads: at 80% capacity. Indirect labor Rs.12,000. Stores and spares Rs.4,000. Semi-variable overheads at 80% capacity. Power (30% fixed) Rs.20,000. Repairs and maintenance at 60% fixed Rs.2,000. Fixed overheads: at 80% Depreciation Rs.11,000. Insurance Rs.3,000. Salaries Rs.10,000. The estimated direct labor hours 1,24,000. Solution:

Divide the grand total by estimated Labour hours. 14.11 Limitations of Budgeting The main limitations of budgeting are as under:

Budget plan: Since budget plans are based on estimates, the success or otherwise depend on the accuracy of basic estimates or forecasts. Due to this while making estimates, judgmental decision may accrue. The results need to be interpreted very cautiously. Rigidity: Since the estimates are quantitative expression of all relevant data, there is likely that finality attachment may become very clear. Such consideration may result in rigidity. Rigidity may become a setback for the changing business conditions. Replacement: Budgeting is not a substitute for management. It is essentially a tool of management. Under no circumstances, it should be concluded that the budgeting is alone sufficient to ensure success and to guarantee future profits. Costly: The installation of budgeting system to an organization involve too much of costs. Its scientific approach will definitely call for huge cost allocation. Small concerns cannot afford to take over huge costs for the establishment of business systems. Since the costs and revenues and operational activities do not match in many occasions, the entire exercise will become costly. The system should be adopted only when benefits exceed the costs. 14.12 Summary
Budgets are not actual but estimated ones. It is therefore a financial and / or quantitative statement prepared and approved prior to a definite period of time, of the policy to be pursued during that period for the purpose of attaining a given objective. Budgeting is a forward planning. It basically serves as a tool for management control.

Planning through budgets brings together all segments of the concern in a cooperative way and they are compelled to think seriously about the planning. The views get enlarged than getting into contraction. Internal refinement, broad indexation of activities, concentrated details is the essential features in planning. All the staff must be involved in the planning function to make it more successful and purposeful. Control signifies such systematic efforts which help the management to know whether actual performance is in line with predetermined goal, policy and plans. It is basically a measurement tool. Yardsticks should be laid down. Standards must be set up.
14.13 Terminal Questions

1. What are the merits of budgets? 2. Describe the essential features of budgetary control. 3. What are the steps in budgetary control?

4. What are the limitations of budgeting? 4.14 Answer Self Assessment Questions 1. Estimates 2. Reality and workable preposition 3. Control 4. Planning 5. Safety signal 6. Budgets 7. Management 8. Fixed budget 9. Fixed , flexible, functional 10. Flexible Copyright 2009 SMU Powered by Sikkim Manipal University
. Answer for Terminal Questions

1. Refer to unit 14.5 2. Refer to unit 14.6 3. Refer to unit 14.7 4. Refer to unit 14.11

MB0041-Unit-15 -Standard Costing


Unit-15 -Standard Costing Structure 15.1 Introduction Objectives 15.2 Definition of Standard Costing 15.3 Meaning 15.4 Difference between Standard cost and Budgetary Control 15.5 Establishment of standards 15.6 Variance analysis 15.7 Material cost variance 15.8 Material price variance 15.9 Material usage variance 15.10 Material Mix variance 15.11 Material Yield variance 15.12 Direct labor variance 15.13 Labor Efficiency Variance 15.14 Labor Rate variance 15.15 Labor mix variance 15.16 Labor Yield Variance 15.17 Summary

15.18 Terminal Questions 15.19 Answers to SAQs and TQs 15.1 Introduction Standard costing is a very important system of cost control. It is a system which seeks to control the cost of each unit or batch through determination before hand of what should be the cost and then its comparison with actual cost. Through planned accounting procedures, the difference between the actual and pre-determined costs are analyzed and then promptly reported upon to managers. Based on this, it is possible to take corrective and preventive action as well as employ the data for planning, coordination and control.

Objectives: After studying this unit, you should be able to : 1. Define the standard costing. 2. Understand the meaning. 3. Differentiate between standard cost and budgetary control. 4. Acquaint with establishment of standards. 5. Practice the variance analysis. 15.2 Definition of Standard Costing Standard costing may be defined as a technique of cost accounting which compares the standard cost of each product or service with the actual cost to determine the efficiency of the operation so that any remedial action may be taken immediately: Brown and Howard. According to J. Batty standard costing is a system of cost accounting which is designed to show in detail how much each product should cost to produce and sell when a business is operating at a stated level of efficiency and for a given volume of output. 15.3 Meaning The meaning of standard costing is focused on the method of financial control. It compares the predetermined and actual costs. It is normally associated closely with budgetary control. Many organizations use both the systems although one can be used without the other. Standard costing is mainly applied to products and processes.

Therefore, it is a technique that is more commonly used in manufacturing organization, though it may also be useful in service industries. As in budgetary control, it allows the comparison of predetermined costs and income with the actual costs and income achieved. Any difference can then be investigated. 15.4 Difference between Standard Cost and Budgetary Control Both are closely interrelated. They both aim at the improvement of the system of managerial control. They both achieve the same objective of maximum efficiency and cost control by establishing pre-determined standards. They compare actual performance with the predetermined standard. They take necessary steps to improve the situation wherever necessary. Both techniques are forward looking. However, the following are some of the differences identified. 1. The scope of budgetary control is wider. It is integrated plan of action, a coordinated plan in respect of all functions of an enterprise. The scope of standard costing on the other hand is limited to the operating level. Here too, it is further linked to costs. Budgetary control is extensive whereas standard costing is intensive in its application 2. Budgetary control deals with costs and revenues. But standard costing restricts only with costs. 3. Budgetary control takes into account all activities such as production, sales, purchases, finance, capital expenditure, personnel whereas standard costing is restricted to deal with only costs. 4. Budgetary control targets are based on past actual adjusted to future trends. In standard costing, standards are based on technical assessment. 5. At the approach level, budgeted targets work as the maximum limit of expenses above which the actual expenditure should not normally exceed. Under standard costing, standards are attainable level of performance. 6. Budget is projection of final accounts. Standard costs are projection of only cost accounts. 7. Budgetary control emphasizes the forecasting aspect of the future operations. Standard 8. Costing scope and utility is limited to only operating level of the concern. 9. In budgetary control, the degree of variance analysis tends to be much less and variances are not revealed through the accounts but are revealed in total. But in standard costing, variances are analyzed in details according to their originating causes and are revealed through different accounts.

10. Budgetary control is possible even in parts of expenses according to the attitude of management. A standard costing system can not be operated in parts. All items of expenditure included in cost units are to be accounted for.

Self Assessment Questions: Standard costing is defined as a technique of cost accounting which compares the _____________ of each product or service with the actual cost . Standard cost focuses on _________. Standard cost and budget control _________. 15.5 Establishment of Standards Under standard costing system, there is a need to determine the standard costs for each element of cost. The standards are fixed for three main elements of cost namely direct material, direct labor and overhead. Standards should be fixed for each of them separately. Direct Material: Standard material cost for each product should be pre-determined. This will require the determination of material quantity standard and material price standard. The standard quantity of each type of materials is determined by the engineering department on the basis of past records, experience and chemical and engineering tests. While setting such standards, an allowance should be made for the normal wastage of materials. The standard price for each item of material is established after carefully studying the market conditions and forecasting the trend of prices for a future period. This is done by cost accountant with the help of purchase officer. Direct labor: The standard labor time and standard labor rate should be established. Standard time for each grade of labor is fixed by the production engineering department on the basis of time and motion study. In fixing the standard time due allowance should be made for fatigue, tool setting, receiving instructions and normal idle time. The standard rates of pay for each category of labor are fixed by the cost accountant with the help of personnel department. Overheads: These are aggregate of indirect materials, indirect labor and indirect expenses. Separate standards must be established for variable and fixed overheads. As variable overheads per unit or per hour remain constant at each level of output / sales but total amount of variable overheads tend to vary directly with volume of output / sales. Therefore, it is sufficient to calculate only a standard variable overhead rate per unit or per hour. This is done by dividing the total variable overheads for the budget period by the budgeted

output. In respect of fixed overheads standards are set for total fixed overheads for the budget period and the budgeted output and standard fixed overhead rate is computed by dividing the budgeted fixed overheads with budgeted output. 15.6 Cost Variance Analysis The distinctive feature of standard costing system is variance analysis. By definition, the term variance means the variation or deviation of the actual from the standard. In standard costing, it implies the difference between the actual cost and standard cost. Variances indicate the extent to which standards set have been achieved. If properly recorded and analyzed, these variances become very important and useful tool for managerial control. Variances by themselves are not the end. They are computed to know the reasons and fix the responsibility for any deviations of actual performances from pre determined targets. Based on this corrective measures are proposed for adoption in future. Therefore, variance analysis is the process of analyzing variances by sub-dividing the total variance in such a way that management can assign responsibility for off standard performance It is hence a very useful means for interpreting operating results and spotting situations calling for correction. Variances are interpreted as favorable and unfavorable variances. Each variance is interpreted. Interpretation helps in deciding whether a variance is favorable or unfavorable. When the actual cost is less than the standard cost, the difference is termed as favorable or credit variance. On the other hand, when actual cost exceeds the standard cost, the difference is termed as unfavorable, adverse or debit variance. Ordinarily, a favorable variance is a sign of efficiency of the organization whereas an unfavorable variance a sign of inefficiency. But in variance analysis, this general logic may prove to be untrue. Therefore, all variances should be interpreted in relation to each other and only the net result be reported to the management for corrective action. Controllable and Uncontrollable variances: The controllable variance can be identified as the primary responsibility of a specified person or a department. The variance is due to the factors beyond the control of the concerned person or department, it is said to be uncontrollable. No person or department can be held responsible for uncontrollable variances. Actually revision of standards is required to remove such variances in future. Self Assessment Questions: Standards are established for _________, __________ and _______________. Variation refers to _____________. When the actual cost is less than the standard cost, the difference is termed as ____________ or credit variance Variances are interpreted as __________ and _____________.

15.7 Material Cost Variance It is the difference between the standard cost of materials allowed for the actual output and the actual cost of materials used. It may be expressed as:

Or standard quantity of material for actual output x standard price per unit of material. A favorable variance would result if actual cost is less than the standard cost and vice versa. The material cost variance is the sum total of material price variance and material usage variance. Illustration 1: Bombay Dyeing Ltd. has decided to extend its range to include Denim Jackets. One jacket requires a standard usage of 3 meters of direct material which has been set at a standard price of Rs.2.20 per meter. In the period, 80 jackets were made and 260 meters of material consumed at a cost of Rs.1.95 per meter. Calculate the direct materials total variance. Solution: Calculate the standard quantity of materials for the actual level of production

The difference indicates that them company has spent less on materials than planned for the level of production. 15.8 Material Price Variance Under Material price variance, the price paid for materials is different from the pre-determined price. It is calculated by multiplying the actual quantity of materials used with the difference between standard and actual prices. The formula is :

A favorable variance would result if the actual price is less than the standard price and vice versa. Illustration 2: Calculate the direct material price variance from the data of Bombay Dyeing Ltd above. Solution:

It is favorable because the company has paid less for the materials than planned for that level of production

15.9 Material Usage Variance It is also known as material quantity variance or efficiency variance. It is that portion of material cost variance which measures the difference in material cost arising from higher or less a consumption of materials than the standard material consumption for the actual output. It is calculated by multiplying the standard price with the difference between the standard and actual quantitative of materials:

A favorable variance would result if the actual quantity is less than the standard quantity and vice versa. Self Assessment Questions: The formula for Material Cost variance is ______________. The formula for Material price variance is __________. The formula for Material usage variance is _______. Illustration 3: Calculate the direct material usage variance from the data of Bombay Dyeing Ltd above. MUV = (SQ AQ) SP = (240 260 meters) x Rs. 2.20 per unit = Rs. 44 (ADV) It is adverse because the company has used more materials than planned for that level of production. Illustration 4: It is observed that one unit of product X requires 3 kgs of material M at Rs.2 per kg. During January 2008, 200 units of product X were produced consuming 620 kgs of material M, all of which was purchased at Rs.1.80 per kg. Compute material cost variances.

Solution:

Illustration 5: For producing a commodity, the standard quantity of material was fixed 10 kgs and standard price was fixed at Rs.2 per kg. The actual quantity was consumed 12 kgs and the actual price was Rs.1.90 per kg. Calculate the material variances. Solution:

Illustration 6: Calculate the material variance.

Standard Material for 80 kgs Finished products 100 kgs Price per kg Rs.0.80 paid

Actual Output 1,65,000 kgs Material used 2,00,000 kgs Actual cost Rs.1,70,000

Solution: Calculation of standard quantity

Illustration 7:

It is estimated that in the manufacture of a product, for each ton of materials consumed 100 units should be produced. The standard price per ton of materials is Rs. 10. During the first week of January, 100 tons of materials were issued to the Production Department. The purchase price of which was Rs. 10.50 per ton. The actual output for the period was 10,250 units. Calculate the cost variances. Solution:

Illustration 8: A factory works on standard costing system. The standard estimates of material for the manufacture of 1000 units of a commodity are 400 kg at Rs. 2.50 per kg. When 2000 units of a commodity are manufactured, it is found that 820 kgs of material is consumed at Rs. 2.60 per kg. Calculate the material variance Solution: First calculate the standard quantity and standard cost.

15.10 Material Mix Variance This variance arises only when more than one type of materials are used in manufacturing the product and the quantities of materials issued to production are not in proportion of standard mix. It is defined as that portion of the direct materials usage variance which is due to difference between the standard and actual, composition of a mixture. For calculating the mix variance, first calculate the quantities of revised standard mix. This is calculated by dividing the total quantities of actual mix in standard mix proportion. In the terminology of standard costing, quantities of revised standard mix are referred to as revised standard quantity. Mix variance is obtained by multiplying the standard price of materials with the difference between revised standard quantity and actual quantity for each material. It may be expressed as follows: Material Mix Variance = Revised standard quantity for each material actual MMV Quantity for each material) x standard price.

Where RSQ = standard quantity for each material / total of standard quantity of all types of materials x actual mix total RSQ = Total weight of actual mix / total weight of standard mix (x) standard quantity. A favorable variance would result if actual quantity is less than revised standard quantity and vice versa. Illustration 9: The following extracts are extracted from the books of DR Ltd.

Standard Mix Material Qty X Y Total 300 200 500

Actual Mix Rate 5 10 Total 1500 2000 Qty 280 220 500 Rate 5 10 Total 1400 2200

Calculate the material mix variance Solution: Revised standard quantity = Total weight of actual mix / total weight of standard mix x standard quantity

15.11 Material Yield Variance This variance arises only when the rate of output is known. It is that portion of the direct material usage variance which is due to the difference between standard yield specified and actual yield obtained. It measures the loss or saving due to wastage of materials. This variance is calculated as follows:

where standard rate = standard cost of standard mix / standard output from standard mix standard yield = standard output from standard mix / standard mix total x actual mix total MYV is an output variance and hence a favorable variance would result if actual yield is more than standard yield and vice versa. Self Assessment Questions: The formula for Material mix variance is __________. The formula for Material yield variance is __________. Illustration 10: DR manufactures a product X. It is estimated that for each ton of material consumed, 100 articles should be produced. The standard price per ton of material is Rs. 10. During the first week of January 2008, 100 tons were issued to production, the price of which was Rs.10.50 per ton. Production during the week was 10,200 articles. Compute the variances Solution:

Illustration 11 : The standard mix of product MS is as follows

Materials A B C

Qty in Kg 50 20 30

Price/kg 5 4 10

The standard loss in production is 10 % of input. There is no scrap value. Actual production for a month was 7,240 kgs of MS from 80 mixes. Actual purchases and consumption of materials are:

Materials

Qty in Kg

Price/kg

A B C Solution

4160 1680 2560

5.50 3.75 9.50

Statement showing standard input requirements of 80 mixes of product MS

Material SQ A B C Total Less std.loss Final output 4,000 1,600 2,400 8,000 800 7,200

Standard SR 5 4 10 SC AQ

Actual AR 5.50 3.75 9.50 AC 22,800 6,300 24,320 53,500

20,000 4,160 6,400 1,680

24,000 2,560 50,400 8,400

50,400

15.12 Direct Labour Variances The same principles apply to the calculation of Direct Labor variances as for the direct material variances. Standards are established for the rate of pay to be paid for the production of particular products and labor time taken for their production. The standard time taken is expressed in standard hours or minutes and become the measure of output. By comparing the standard hours

allowed and the actual time taken, labor efficiency can be assessed. In practice, standard times are established by work, time and method study techniques. Direct Labor Variance: It is the difference between actual labor cost and the standard labor cost of production achieved. It is calculated as :

Illustration 12: The management of DR Ltd decides that it takes 6 standard hours to make one Denim jacket and the standard rate paid to labor is Rs. 8 per hour. The actual production is 900 units and this took 5,100 hours at a rate of Rs.8.30 per hour. Calculate the direct labor total variance. Solution: Calculate the standard labor hour for 900 jackets. For one Jacket production, the standard hour is 6. Therefore, for producing 900 units, the standard hour is 900 x 6/1 = 5,400 hours. DLV = (5,400 x (5,100 x 8.30) = Rs. 870 favorable.

A favorable variance would result when actual cost is less than standard cost and vice versa. Labor cost variance is the sum total of labor rate variance, labor efficiency variance, rate variance, idle time and labor calendar variance 15.13 Labour Efficiency Variance It is that portion of labor cost variance which is due to the difference between the standard labs or hours specified for the activity i.e. output achieved and the actual labor hours worked. It is calculated by multiplying standard rate of wages with the difference between standard hours and actual hours worked. LEV = Standard hours actual hours worked) x standard rate Shorten = (SH AH ) SR A favorable variance would result when actual hours worked are less than standard hours and vice versa. Illustration 14: From the data above, calculate the labor efficiency variance; Solution: (5400 standard hours 5100 actual hours) x Rs. 8 standard rate Rs. 2,400 FAV

15.14 Labour Rate Variance It is that portion of labor cost variance which is due to the difference between the standard rate specified and actual rate paid. It is calculated by multiplying the actual hours paid with the difference between standard rate specified and actual rate paid. Labor Rate variance = (Standard Rate Actual Rate) x actual hours paid. Shorten = (SR AR) AH

Self Assessment Questions 5: The formula for Direct labor variance is ___________. The formula for Labor efficiency variance to _________________. The formula for Labor rate variance is __________. Illustration 15: Citing DR example, calculate the direct labor rate variance Solution:

llustration 16: The production of a certain unit is assumed to require 18 hours labor at a rate of Rs. 1.25 per hour. On completion of a unit, it was found that the time taken was 16 hours, the wage rate being Rs. 1.50 per hour. Calculate labor variances. Solution:

Working:

Labor Idle Time Variance: It is that portion of labor cost variance which is due to abnormal idle time of workers. This variance is calculated to show separately the effect of abnormal causes

affecting production such as failure of power supplies, machine break down, waiting for materials, waiting for instructions, strike, lock-outs. It is calculated as: Labor idle time variance = Idle hours x standard hourly rate This variance is always an adverse one. Illustration 17: In the ma manufacture of a product, 200 employees are engaged at a rate of 50 paise per hour. A five day week of 40 hours is worked and the standard performance is set at 250 units per hour. During the first week in January, six employee were paid at 45 paise an hour and four at 56 paise an hour, the remaining employees were paid at standard rates. The factory stopped production for one hour due to power failure. Calculate variances. Solution:

15.15 Labour Mix Variance This variance arises only when different types of workers (women and men workers, trained, semi-trained and untrained workers, are employed in manufacturing. If actual working force of different grades of workers is not in the pre-determined ratio, then the mix variance will occur. The variance shows to the management as to how much of the labor cost variance is due to the changes in the composition of labor force. It is calculated as follows: LMV = (Revised standard hours actual hours worked) x standard hourly rate Shorten (RSLH ALH) x SR Where revised standard hour = total time of actual worker / total time of standard workers x standard labor rate.

Illustration 18: The labor budget for a week is as follows: 40 skilled men at Rs.1.50 per hour for 80 hours 80 unskilled men at Re.1 per hour for 80 hours Actual labor force was used are given below;

60 skilled men at Rs.1.50 per hour for 80 hours 60 unskilled men at Re.1 per hour for 80 hours Calculate labor mix variance. Solution:

15.16 Labour Yield Variance This is due to the difference in the standard output specified and the actual output obtained. The formula is as follows:
LYV = (Actual output Standard output) x standard cost per unit

Illustration 19: Actual output 460 units. Standard output 500 units. Standard rate of wages is Rs.9 per hour. The Standard time is 2 hour per unit. Solution:

Illustration 20: Calculate (a) Labor rate variance (b) labor efficiency variance (c) labor cost variance Standard: Labor rate 0.24 paise per hour. Labor hours 3 per unit. Actual: Units produced 250. Labor rate 0.25 paise per hour. Hours worked 800. Solution:

15.17 Summary Standard costing is a system of cost accounting which is designed to show in detail how much each product should cost to produce and sell when a business is operating at a stated level of efficiency and for a given volume of output

Budgetary control takes into account all activities such as production, sales, purchases, finance, capital expenditure, personnel whereas standard costing is restricted to deal with only costs.
The term variance means the variation or deviation of the actual from the standard. In standard costing, it implies the difference between the actual cost and standard cost.

Under standard costing system, there is a need to determine the standard costs for each element of cost. The standards are fixed for three main elements of cost namely direct material, direct labor and overhead. 15.18 Terminal Questions 1. How is standard costing related to budgetary control? 2. How do you fix standard for direct material, direct labor and direct overheads? 3. Write short note on : a) Material Price Variance b) Material Mix Variance c) Material Yield Variance. 4. Briefly describe labor mix variance and yield variance. 5. Compute price, usage, cost and mix variance

Material

Standard Quantity Price Total Qty

Actual Price Total

A B

6 2

1.50 3.5

9 7

5 1

2.40 6

12 6

6. Data given below pertain to DR Ltd.

DEPARTMENT A Actual gross wages Standard hours produced Standard rate per hour Actual hours worked Calculate labor variances 15.19 Answer to SAQs and TQs Standard cost Financial control Closely interrelated 8. SC AC 9. (SP AP) x AQ 10. SQ AQ x SP 2,000 8,000 0.30 8,000 B 1,800 6,000 0.35 5,800

Direct martial, direct labor, overheads 11. RSQ AQ x SP Deviation Favourable Favorable and adverse 12. SY AY x SR 13. SC-AC 14. SH AH x SR 15. SR AR x AH Terminal Questions 1. Refer to unit 15.4

2. Refer to unit 15.5 3. Refer to unit 15.8, 15.10 and15.11 4. Refer to unit 15.15 and15.16 5. Price variance = (SR AR) AQ for A = Rs. 4.50 ADV For B = Rs.2.50 ADV total Rs.7 ADV Usage variance: (SQ AQ ) SP for A = 1.50 FAV For B = 3.50 FAV Total Rs. 5 FAV Cost Variance = MUV + MPV = 5 7 or 2 ADV Mix variance = (RSQ AQ) SP where RSQ = A 6 /8 x 6 or 4.5 kgs B 6 /8 x 2 or 1.5 kgs For A (4.5 5) x 1.5 or 0.75 ADV For B (1.5 1) x 3.5 or 1.75 FAV Therefore total 1 FAV 6. Standard labor cost; Dept A 8000 x 0.30 = 2,400 B 6,000 x 0.35 = 2,100 Actual rate A 2000. / 8000 = 0..25 B 1800 / 5800 or 900 /29 paise LRV = (SR AR ) AH Dept A = (0.30 0.25) 8000 = 400 FAV B = (.35 900 /25) x 5800 = 230 ADV LEV = (SH AH) SR for A = (8000 8000) x 0.30 Nil B = (6000 5800) 0.35 70 ADV LCV : (SLC ALC For A (2400 2000) 400 FAV B (2100 1800) 300 ADV Reference Books:

1. Accounting for Managers by Jawahara Lal. 2. Financial Accounting by S. N. Maheswari. 3. Financial Accounting for Managers by R. Narayana Swamy. 4. Introduction to Management by Anthony Reece. 5. Management Accounting by Manmohan and Goel. 6. Cost and Management Accounting by Horugren etal. Copyright 2009 SMU Powered by Sikkim Manipal University

Vous aimerez peut-être aussi